Step 2 Clinical Cases

अब Quizwiz के साथ अपने होमवर्क और परीक्षाओं को एस करें!

13 yo female w/ acne over her forehead and chin that began 4 mo ago and is now painful w/more acne over face and upper chest. She hasn't had her period and has grown 4in in past yr (height and weight in 70th percential). There is no breast bud development. External pelvic exam shows the clitoris protruding from the clitoral hood and bilateral masses in the labia majora. What is the most likely diagnosis?

5-alpha-reductase deficiency - patients are unable to convert testosterone into DHT; therefore they have no male external genitalia development, have undescended testes (eg, bilateral labial masses), and appear phenotypically female at birth (46, XY) - at puberty, increased testosterone levels cause virilization (acne and clitoromegaly) **patients w/ nonclassical adrenal hyperplasia (21-hydroxylase deficiency) can have virilization; however, they are genotypically femal and therefore don't have undescended testes (eg, bilateral labial masses)

A cohort study is conducted to assess relationship between high saturated fat diet and occurrence of colorectal cancer. The study showed that women with a high baseline saturated fat consumption have 4 x the risk of colorectal cancer in a 7 yr period, compared to low fat consumption (RR= 4.0 95% CI = 1.5-6.5). What percent of colorectal carcinoma w/ women w/ high fat consumption could be attributed to diet?

75% Attributable risk percent = (risk of exposed - risk of unexposed)/risk in exposed (4-1)/4 = .75 or 75%

16 yo boy comes to ED due to femur fracture and reveals packed RBC transfusion. 5 min aft it begins he develops chills and severe flank pain. Temp is 102.4, BP is 88/40, pulse is 130, and resp 30. PE shows bleeding from IV catheter side. Breath sounds are normal. Most likely cause?

ABO incompatibility **cytokine release from transfused blood products causes febrile nonhemolytic reaction (most common), which presents w/fever and chills 1-6 hrs aft transfusion

A 58-year-old woman who is 6 days postop from a gastrojejunostomy for PUD presents with postprandial RUQ pain, floating stools, and nausea. She reports that vomiting relieves her suffering. Think:

Afferent loop syndrome tx: endoscopic balloon dilation

59 yo woman falls from a standing height and is found to have displaced fractures of the left posterior ninth and tenth ribs on xray with normal breath and heart sounds, left shoulder pain, and tachycardia. What would be the best next test to do?

CT scan of abdomen left lower ribs injuries are capable of injuring the spleen tachycardia could be a sign of early hemorrhagic shock and left shoulder pain could be referred pain from the left diaphragm

42 yo comes to ED w/severe headache that has rapidly worsened. She also has neck pain and episode of vomiting. Has HTN. Works at a daycare. Temp 100.9, BP 154/92, and pulse 102. Patient has no focal weakness or numbness. Fundoscopy shows no papilledema. There is decreased resistance to passive neck flexion, and it also elicits pain. Next step in management?

CT scan of brain (subarrachnoid hemorrhage) - neck pain and low grade fever common CT is done before lumbar puncture (looks for xanthrochromia if CT unremarkable)

48 in ED due to a wk of progressive blurred vision and floaters in R eye. No ocular pain, fever, or headache. Has AIDs and is noncompliant with therapy, was recently hospitalized for pneumocystis pneumonia. Vision is decreased in R eye and fundoscopic exam shows several yellow-white exudates immediately adjacent to fovea and retinal vessels. Most likely dx?

CMV retinitis - most common end-organ manifestation of CMV in pts w/AIDS - usually in CD4 counts <100 - tx w/ valganciclovir **HIV retinopathy is marked by cotton-wool retinal lesions that are rarely hemorrhagic and tend to resolve over weeks to months - doesn't cause floater or blurred vision

16 yo boy fell on outstretched hand while playing soccer and has left shoulder and hand pain. Crunching sound was heard immediately following the injury. BP 105/72 and pulse 85/min. There is bruising around left shoulder and small hematoma at base of neck. He has intact sensation of left with slightly decreased radial pulse. Xray reveals displaced fracture of clavicle with a normal cardiac silhouette. Next best step in management?

CT angiogram - middle 1/3 of clavical overlies brachial plexus and the subclavian artery and vein in the thoracic outlet - "soft signs" of arterial injury like stable hematoma and reduced pulse require vascular imaging to confirm and localize site of injury - hard signs like distal limb ischemia or absent distal pulse require immediate surgery

A 60-year-old male presents with a history of cutaneous flushing, diarrhea, wheezing, and an unintentional 15-lb weight loss. Think:

Carcinoid syndrome; the wheezing is a clue that the lesion may be endobronchial. Order a 24-hour urine 5-HIAA level to confirm the diagnosis.

A 33-year-old female smoker presents with burning epigastric pain that is improved after eating a meal. Think:

Duodenal ulcer

You are asked to see a patient with bleeding from an angiogram puncture site. She has an oozing, pulsatile expanding mass in her groin at the puncture site. Think:

Expanding hematoma. Maintain direct pressure for 30 minutes. If bleeding continues, wound exploration may be indicated.

60 yo man w/squamous cell carcimoma has a right pneumonectomy planned. What is most likely to provide best assessment of postop pulmonary morbidity?

FEV1 and diffusion capacity of the lung for carbon monoxide (DLCO), obtained by preop pulmonary function testing **measurement of oxygen sat during exercise is used to evaluate need for home supplemental oxygen therapy in patients w/COPD

A 52-year-old woman presents due to 3 months of early satiety, weight loss, and nonbilious vomiting. Think:

Gastric outlet obstruction

A 45-year-old Japanese male smoker presents with weight loss and epigastric pain exacerbated by eating. Think:

Gastric ulcer.

43 yo man w/worsening pedal edema of last several weeks. Has fatigue, but no dyspnea or chest pain. HIV+ w/ inconsistent antiretroviral therapy and w/pneumocystis pneumonia 4 mo ago. Normal heart and lung sounds and nontender abdomen. BUN 28, creatinine 2.4 and urinalysis w/3+ proteinuria. Serum creatinine was normal at hospital discharge 4 mo ago and Cd4 was 220 1 mo ago. What is the most likely cause of his condition?

HIV-associated nephropathy - thought to be caused by direct infection of the renal tubular and glomerular cells by HIV virus - presents w/heavy proteinuria and rapidly progressive renal failure - dx confirmed w/ renal biopsy that demonstrates collapsing focal segmental glomerulosclerosis

35 y.o. HIV+ homosexual male complains of deterioration of vision over past wk. He initially experience pain and mild conjunctivitis, followed by rapidly progressive vision loss. Exam of eyes reveals marked keratitis. Fundoscopy shows widespread, pale, peripheral retinal lesions and central necrosis of the retina. Most likely causative organism?

Herpes simplex - in HIV patients, HSV and VZV can cause severe, acute retinal necrosis associated with pain, keratitis, uveitis, and fundoscopic findings of peripheral pale lesions and central necrosis CMV retinitis is painless, not usually associated with keratitis or conjunctivitis, and characterized by fundoscopic findings of hemorrhages and fluffy or granular lesions around the retinal vessels

patient presenting with LGIB and a mid-systolic murmur in the right second intercostal space, think

Heyde syndrome—angiodysplasia (causing bleeding from colon) with aortic stenosis.

62 yr old w/ acute right sided headache, ocular pain, and nausea leading to decreased vision in left eye. In ED she has episodic retching and right eye has red conjunctival flushing and right pupil is mid-dilated and nonreactive to light. Next step in management?

IV acetazolamide (acute-closure glaucoma and you want to decreas intraocular pressure) can also use topical timolol, pilocarpine and apraclonidine (all together) **high dose steroids are for anterior ischemic optic neuropathy in giant cell (temporal) arteritis

75 yo w/ advanced prostate cancer presents with subacute low back pain that has now profoundly changed in severity. Examination shows lower extremity weakness, hyperreflexia, and bladder dysfunction. What is the best initial step of management in this patient?

IV glucocorticoids (concern for epidural spinal cord compression (ESCC)) - glucocorticoids decrease vasogenic edema (caused by obstructed epidural venous plexus), help alleviate pain, and may restore neurological function MRI should be performed after steroids are given **a radionuclide bone scan can detect metastatic disease but doesn't provide useful information about compression of the thecal sac

2 yo boy w/ sickle cell anemia brought to ED due to pallor and decreased responsiveness. This morning looked pale and parents had difficulty waking him . He has no fever, vomiting, diarrhea, or rashes. Exam reveals a 3/6 systolic ejection murmur throughout precordium and clear breath sounds. He has marked splenomegaly and grimaces w/ palpation of LUQ. Capillary refill > 3 seconds, and extremities are cool and cyanotic to touch. Labs?

Increased reticulocytes and decreased platelets (splenic sequestration crisis as seen by enlarged spleen and signs of shock) to compensate for anemia, reticulocytes increase and there is thrombocytopenia because platelets are trapped in the spleen **aplastic crisis occurs after parvovirus B19 and there is arrest of erythrocyte production due to severe anemia (low retics and normal platelets)

32 yo w/ difficulty getting pregnant. Has recurrent vaginal candidiasis for which she uses over the counter vaginal suppository. Vitals are normal and thyroid isn't enlarged. She has thick, dark, velvety plaques under her axillae. Pelvic exam reveals normal external genitalia; a small, mobile uterus w/no cervical motion tenderness; and bilateral enlarged ovaries. TSH, LH, and prolactin are normal. What is the most appropriate therapy for her infertility?

Letrozole (PCOS) Letrozole is an aromatase inhibitor, that inhibits the conversion of androgens to estrogens - decreased estradiol secretion causes positive feedback to the pituitary gland, increasing LH and FSH - LH surge results in ovulation Letrozole is first-line in PCOS due to higher live birth rates compared w/clomiphene citrate **cyclic progesterone can be administered for endometrial protection against uncontrolled endometrial proliferation due to elevated estrogen levels in PCOS. However, progesterone doesn't facilitate ovulation and isn't used for infertility

19 yo comes in w/ persistent sore throat, fever, and mild lower abdominal discomfort for past few days. No changes in bowel or bladder fxn and no other pmh. She lives in dorms, is sexually active and has 2 new partners (on ocps). Temp is 101, bp 120/70, pulse 68. The pharyngeal arches are erythematous w/out exudate. There is bilat nontender cervical lymphadenopathy. Abdomen is tender to deep palpation in the lower quadrants. UPT is neg. Pathogen that is most likely responsible?

Neisseria gonorrhoeae - pharyngitis w/ fever and lower abdominal pain in young, sexually active woman is suggestive of gonococcal pharyngitis w/PID **EBV mono can present w/exudative pharyngitis and TENDER cervical lymphadenopathy, which aren't seen in this pt. In addition, typical presentation includes fatigue, rash, and splenomegaly

37 yo g2p1, who has not received prenatal care and is at 24 wks gestation, precipitously delivers a male fetus w/no cardiac activity, edematous, peeling skin, no dysmorphic features, tense and fluid-filled abdomen w/thickened and edematous placenta. Maternal lab results: Hgb 11.2, O-, negative indirect Coombs test, HIV negative. What is the most likely cause of the fetal demise?

Parvovirus B19 infection leading to nonimmune hydrops fetalis (sequela of high-output HF characterized by excessive fluid accumulation in interstitial spaces) 3rd spacing of fluid leads to ascites (tense fluid-filled abdomen), generalized skin edema (and subsequent peeling) **RH(D) alloimmunization is very unlikely w/ negative indirect coombs

75 yo man comes in for physical w/ PMH of osteoarthritis, HTN and hyperlipidemia. Current meds are hydrochlorothiazide and atorvastatin. Prostate diffusely enlarged without nodules. The distal interphalangeal joints are enlarged. Ca2+ is normal, PSA is normal, ALP is elevated at 420. What is the likely cause for this?

Paget disease of the bone (most patients asymptomatic with elevated ALP and normal Ca2+ and phosphorus)

A patient with known PUD presents with sudden onset of severe epigastric pain. Physical exam reveals guarding and rebound tenderness. Think:

Perforation

62 yo postmenopausal, nulligravid woman has right adnexal enlargement found on routine pelvic exam. US revealed 5cm ovarian cyst. Pt has no vaginal spotting or bleeding and has never had any surgeries. She has no family hx of ovarian or breast cancer. Had a normal mammogram 2 yrs ago. What is the most appropriate initial course of action?

Serum CA-125 - biomarker of epithelial ovarian cancer - elevations are also caused by common gynecologic conditions (eg, leimyomata, endometriosis) that are more likely in premenopausal women; therefore, specificity of CA-125 is much greater in postmenopausal women If Ca-125 is normal w/small ovarian mass, postmenopausal pts can be observed w/periodic US **knowledge of Ca-125 is important in guiding surgical exploration (laparoscopy), so should be done first

21 yo female w/ continuous vomiting, diarrhea, headache and high fever for a day. She spent 3mos in Southeast Asia and returned yesterday on an international flight. She is currently on the 5th day of her menstrual period and has had to use both tampons and pads due to heavy bleeding. Temp is 103, BP 80/40, and pulse is 124/min. PE shows diaphoretic woman dry mucous membranes. No nuchal rigidity. There is an erythematous macular rash covering entire body including palms and soles. What organism is most likely responsible?

Staph aureus (Staphylococcal toxic shock syndrome) **secondary syphilis can present w/ maculopapular rash involving hands and soles but patient would typically have indolent course rather than acute-onset hypotension and tachycardia (shock)

A 24-year-old male with chronic constipation complains of intense anal pain. He has a tender, swollen, bluish lump at the anal orifice. What is the treatment?

Surgical excision if the pain has been present for <48 hours or is persistent. Pain typically subsides after 48 hours, and treatment is symptomatic. history of constipation and physical exam findings are classic for a thrombosed external hemorrhoid.

A patient with a history of duodenal ulcers presents with massive GI bleeding. Upper GI endoscopy shows an ulcer that penetrates the posterior wall of the duodenal bulb (first part of the duodenum). What is the most likely vascular structure responsible for the bleeding? Think:

The gastroduodenal artery courses behind the first part of the duodenum and is the most likely culprit.

3 yo girl with bp of 156/96 mm Hg and US revealing 12-cm, heterogenous, solitary mass arising from superior pole of the right kidney

Wilms tumor (nephroblastoma) **neuroblastoma usually in kids <2, arise in adrenal glands, and HTN is less common

57 yo w/2 episodes blood in urine and fatigue and fever over last 4 wks. 50 pack year hx. Father died from blood disorder. Has left-sided varicocele that fails to empty when he is recumbent. Labs: Hgb 18, WBC 7400, Platelets 580,000 and blood on urinalysis. Most appropriate diagnostic procedure?

abdominal CT - he most likely has renal cell carcinoma (RCC) - varicocele that fails to empty when recumbent due to tumor obstruction of gonadal vein where it enters renal vein - ectopic production of EPO producing polycythemia

An 84 yo is brought to ED w/ left-sided flank pain and nausea over last hr. Also has syncope but no chest pain, bowel symptoms, or bladder symptoms. PMH: HTN, sigmoid diverticulitis, and TIA 5yrs ago. 50-pack yr hx. BP is 90/60 and pulse is 112 and regular. Pulse ox is 94% on RA. Patient is anxious, pale and diaphoretic. Diffuse abdominal tenderness and left CVA tenderness. Bowel sounds are present and there is no rigidity or guarding. Peripheral pulses are decreased. Dx?

abdominal aortic aneurysm rupture - severe flank pian, syncope, and hemodynamic instability w/smoking hx **acute renal infarction presents w/abdominal or flank pain accompanied by HTN (not hypotension) due to increased renin secretion. Nausea is also uncommon

3 wk old boy in neonatal intensive care w/several episodes of bilious, non-bloody vomiting. Born at 28 wks and weighed 2 lb. Required mechanical ventilation initially. On nasogastric feeds, has gained .07lb every day. Has small, yellow, seedy stools, but no bowel movement in 24 hrs. Temp 96, bp 75/45, pulse 165 resp 70. Appears lethargic, w/ tense, distended abdomen and hypoactive bowel sounds. Labs: leukocytosis and metabolic acidosis. What are the likely imaging findings?

abdominal x-ray w/ air in the bowel wall and portal veins - necrotizing enterocolitis (bilious emesis, abdominal distension, leukocytosis, and metabolic acidosis in a premature infant exposed to bacteria from enteral feeds)

A 32-year-old female presents with nausea, vomiting and right upper quadrant pain that radiates to her back. History reveals episodes of similar pain after a fatty meal. Her vitals show a temperature of 37ºC (98.6ºF), blood pressure of 110/64 mmHg, and heart rate of 72/min. Physical examination reveals mild tenderness in the right upper quadrant and a negative Murphy's sign. An ultrasound is obtained revealing a stone within the common bile duct. The most likely additional finding upon obtaining a hepatobiliary scan is:

absent filling of the intestines after 1 hr - in patients w/ choledocholithiasis, a HIDA scan will depict normal filling time of the liver, gallbladder, and CBD, but the tracer won't make its way into the small intestine

56 yo w/fever on postop day 3 after CABG requiring 4 grafts. Has DM2, hypercholesterolemia, and HTN. Has coarse breath sounds b/l w/normal heart sounds. Patient grimaces during palpation of RUQ. Bowel sounds are decreased. Hgb 10.8, WBC 17,000, ALP 100, AST 32, ALT 34, Amylase 110. Xray reveals postop changes but no new infiltrate. What is the most likely diagnosis?

acalculous cholecystitis - unexplained fever, RUQ pain, leukocytois (can have normal liver function tests) **subphrenic abscess (rare) can cause fever and abdominal pain. However, it usually develops due to peritonitis (eg, perforated ulcer, appendicitis, abdominal surgery) and is unlikely in pt who didn't undergo abdominal surgery

37 yo has severe lower abdominal pain and vomiting. She developed sudden-onset, RLQ pain after waking this am. The pain continues to increase and she now has persistent vomiting. She has been taking clomiphene to increase fertility and LMP was 5wks ago. Temp is 100, BP 140/80, and pulse is 92. She has tenderness over RLQ w/ guarding and rebound tenderness. There is tenderness over right adnexa, but no palpable masses or cervical motion tenderness. WBC 16,000, B-hCG is 2840. Transvaginal US shows singleton gestation w/in the uterine cavity and normal b/l adnexa w/ normal doppler flow. dx?

acute appendicitis - nausea, vomiting, RLQ pain, and peritoneal signs **ovarian hyperstimulation syndrome is an exag, abnormal response to ovulation induction (eg, clomiphene) caused by overexpression of VEGF in the ovaries. VEGF overexpression results in B/L enlarged, cystic ovaries w/ increased vascular permeability (eg, increased doppler flow), which causes 3rd spacing (eg, ascites, pulmonary edema)

56 yo w/ 2 days of dull, persistent abdominal pain in LLQ that has progressively worsened and nausea. She has no appetite and vomited once today. She also has mild urinary urgency w/no dysuria or hematuria. Hasn't had a bowel movement but is passing flatus. Has HTN, DM2, and ovarian cyst. Temp is 100.4, BP 134/82, and pulse is 94. PE shows LLQ tenderness w/ no rebound or guarding. Bowel sounds normoactive. Urin dipstick is positive for leukocyte esterase and negative for nitrite. Dx?

acute diverticulitis - LLQ pain, nausea and vomiting, bladder symptoms and sterile pyuria (eg, pos leukocyte esterase, negative nitrite/bacteria) due to bladder irritation from adjacent sigmoid colon inflammation **ovarian torsion usually causes pelvic pain, nausea/vomiting and (occasionally) fever. However, the pain is often acute and severe, and most pts have palpable adnexal mass

24 yo who has been acting oddly since beginning a fasting diet 4-5 days ago. He is irritable and moody and says that he is being haunted by large, shadowy figures. He also has abdominal pain and tingling in his fingertips. BP 140, pulse 110, resp 16. Abdomen is soft w/ decreased bowel sounds. He has decreased sensation in fingertips and mild weakness in right arm. Dx?

acute intermittent porphyria - should be expected w/sudden-onset abdominal pain, neuropsychiatric symptoms, and family hx (AD) - elevated urine porphobilinogen during attack is diagnostic

Pt w/ new-onset epigastric pain, gestational hypertension, and proteinuria at 37 wks gestation is at an increased risk of what complication?

acute ischemic stroke (preeclampsia w/ severe features) endothelial cell damage in the brain can precipitate an acute stroke via the following mechanisms: - activation of the coag system, platelet aggregation, and vascular microthrombi formation, leading to cerebral vessel occlusion ischemic stroke - dysregulated cerebral blood flow that causes inappropriate cerebral vasospasm, severely elevated profusion pressure, and ruptured intracerebral vessels (hemorrhagic stroke) **subarachnoid hemorrhage typically occurs due to ruptured intracranial saccular (berry) aneurysm; risk factors include cig smoking and chronic HTN. Preeclampsia isn't a risk factor

65 yo man w/ COPD cor pulmonale is tx in hospital with steroids, bronchodilator nebulization, and furosemide. Lab studies have changed from time of admission to now w/ BUN going from 28 to 60 and creatinine going from 1.1 to 2.4 and patient has anion-gap metabolic acidosis. What is the likely cause of these changes?

acute kidney injury from diuretic therapy loop diuretics are often used in cor pulmonale to lower right ventricular filling volumes and reduce peripheral edema, but the can lead to hypovolemia, low cardiac output, and renal hypoperfusion Prerenal azotemia/acute kidney injury can develop an is reflected in elevated creatinine, BUN: creatinine ratio > 20, and elevated anion gap acidosis likely from uremia **glucocorticoids elevate glucose and elevate BUN do to their catabolic effect on body proteins, but are unlikely to cause creatinine elevation

5 yo girl fell onto living room floor one week ago and has had persistent leg pain since then that is worse at night. She is now refusing to walk. She has had a mild, nonproductive cough for the past mo. Temp is 100.9. She is pale, liver is palpated 3cm below costal margin, there is pain w/ palpation of b/l anterior proximal tibias. There are scattered bruises across the chest and back. Hgb 8, PLT 30,000, WBc 3000. Dx?

acute lymphoblastic leukemia leukemic infiltration of the bone marrow causes bone pain, pancytopenia, pallor (due to anemia), and bruising (due to thrombocytopenia) - also explains pt's leukopenia (although ALL is classically assoc w/ leukocytosis) hepatosplenomegaly can occur w/ extramedullary leukemic spread **aplastic anemia is characterized by pancytopenia but, in contrast to hypercellular bone marrow of ALL, the hypocellular marrow doesn't cause pain - also wont have hepatosplenomegaly

60 yo has worn hearing aids for years but recently got fitted for new molds. Over past 2 days, he has drainage from R ear that causes aid to become clogged and ear canal is swollen, itchy and painful. R ear exam elicits pain and canal is erythematous w/purulent drainage. There is no granulation tissue and the tympanic membrane is clear. CN fxn is normal. Most likely dx?

acute otitis externa (AOE) - hearing aids can cause retention to moisture, which makes patients more prone to getting aOE **new hearing molds may raise concern for allergic contact dermatitis, but pt would typically show scaly appearance at all sites of contact (ie, both ears) rather than a unilateral, edematous draining ear canal

What is the best initial step in treating hyperosmolar hyperglycemic state (HHS)?

aggressive fluid replacement w/isotonic normal saline, which replenishes extracellular volumes, lowers plasma osmolality, and increased tissue perfusion and responsiveness to insulin **then can give insulin (should be IV regular insulin because hypovolemia and systemic vasoconstriction in HHS can lead to erratic absorption of long-acting insulin)

29 yo male gets multiple boluses of IV fluid and is started on packed RBC transfusion after MVC w/unstable pelvic fx. A few minutes after transfusion starts, he has SOB and an episode of nonbloody emesis. BP 88/40, pulse 140, resp 30. Puls Ox 86% on 100% O2 nonrebreather mask. Lung exam shows decreased breath sounds bilaterally. Pulse is weak. Urinalysis is normal? What is likely cause of his respiratory distress?

anaphylactic reaction - stop transfusion and give IM epi **transfusion-associated circulatory overload is characterized by pulmonary edema due to volume overload. Pts have respiratory distress and hypoxemia, but hypotension wouldn't be suspected (many pts hypertensive)

patient w/ fever, abdominal pain, tachypnea, and tachycardia 5 days after Roux-en-Y gastric bypass

anastomotic leak get abdominal CT scan with oral contrast, followed by urgent surgical repair if a leak is demonstrated

65 yo has had 3-4 episodes of dark maroon-colored stools in last 2 wks, w/normal intervening bowel mvmnts. She has no abdominal pain, nausea, or vomiting. Has hx of HTN, DM1, and hypercholesterolemia. PE reveals 3/6 systolic ejection murmur in R 2nd ICS. Carotid pulses are delayed b/l. Hgb 11.1, MCV 90, BUN 34, Cr 1.6. Colonoscopy 6 mos ago was unremarkable but was somewhat limited in ascending colon due to suboptimal bowel prep. Most likely cause of sx?

angiodysplasia - painless bleeding from dilated submucosal veins and AV malformations that can be missed on colonoscopy due to poor bowel prep or location behind haustral fold - common in pts over 60 w/ vW deficiency, renal disease, and aortic stenosis **diverticulosis is unlikely to be missed on colonoscopy and bleeding is freq arterial and typically results in bright red blood

76 yo w/ incidental finding of leukopenia. Has had 2 episodes of skin infections over past 3 mo. She has also had pain and swelling in her hand and knee joints for several years. She has moderate splenomegaly, generalized lymphadenopathy, and swollen small joints of hands. Hgb 11.8, MCV 92, PLT 280,000, WBC 1800 (20% neutrophils), ESR 68. Pt would likely have what additional finding?

anticitrullinated peptide antibodies (Felty syndrome) - rheumatoid arthritis - vasculitis (mononeuritis multiplex, necrotizing skin lesions) - neutropenia - splenomegaly **aplastic anemia would have significant reduction in all 3 cell lines (not normal platelet) and splenomegaly would be rare

22 yo woman w/ 3 wk hx of rash on face and upper back, that first appeared during beach vacation. During vacation, she had flu-like illness and she continues to experience fatigue and aching pain in joints. She is sexually active w/ multiple partners. Temp is 100.4, BP is 142/80, and pulse is 92 and reg. PE shows erythematous, maculopapular rash on face and upper back. Nontender, generalized lymphadenopathy is present. There is faint diastolic murmur and mild splenomegaly. Hgb 10.4, MCV 92, PLT 110,000, WBC 3200. Most appropriate next step in dx?

antinuclear antibody assay (SLE) common presenting symptoms of SLE include constitutional sx (eg, flu-like sx), arthralgias, and photosensitive rash (on classic sun-exposed areas following beach vacation) - new murmur and HTN indicate heart and kidney involvement - can produce hematologic manifestations (eg, mild thrombocytopenia, anemia, leukopenia), lymphadenopathy, and splenomegaly

13 yo boy has poor weight gain and is shortest kid in his class. He has occasional abdominal pain and bloating but has a good appetite. He has no nausea, vomiting, diarrhea, hematochezia, or melena. He has DM1 and insulin was recently reduced due to episodes of hypoglycemia. PE shows anicteric sclera and moist mucous membranes. Abdomen is nondistended, soft, and nontender w/ no organomegaly. Labs: Hgb 10.8, MCV 72, PLT 435,000, WBC 9,000 HgbA1c 5.5%, TSH 2.4. Next step in eval?

antitissue transglutaminase antibody assay this adolescent w/type 1 DM is in a low height and weight percentile, and the development of recurrent hypoglycemia and reduced insulin requirements suggest malnutrition despite adequate oral intake - given his abdominal discomfort and microcytic anemia, this presentation raises suspicion for celiac disease

11 yo boy w/sickle cell disease is brought to ED w/worsening SOB, weakness, and fatigue over past 3 days. He has no fever, cough, or chest pain. He is pale and tired-appearing w/tachycardia, a 3/6 systolic murmur heard throughout precordium. Lungs are clear to auscultation. No hepatosplenomegaly. Hgb 4.5, reticulocytes 0.1%. What is the most likely cause?

aplastic crisis - acute drop in hemoglobin accompanied by low reticulocyte count (<1%) without splenomegaly (sudden halt in RBC production) - most common cause is parvovirus B19 infection **splenic sequestration can cause severe acute anemia (vasoocclusion and pooling of RBCs in spleen), but there would be elevated reticulocyte count and rapidly enlarging spleen

28 yo w/ left-sided headache and neck pain after a fall while skiing yesterday. Today she also has transient leg weakness. Her father had a stroke. Temp is 99, BP is 134/86, and pulse is 90. PE shows mild ptosis and miosis of left eye. No carotid bruit is present. Neurologic exam shows normal strength, DTRs and sensation in UE and LE. Most likely cause of her current condition?

arterial dissection - unilateral headache, neck pain, and partial Horner syndrome following a fall and likely also a TIA suggests internal carotid artery dissection (a common cause of stroke in young patients injury to the arterial intima allows blood to flow into the vessel wall, leading to the formation of a false lumen, aneurysm, or most commonly, an intramural hematoma. this can lead to arterial obstruction or thromboembolic events; cerebral ischemia (eg, stroke, TIA) is a common complication. distension of sympathetic fibers that travel along the internal carotid artery lead to partial horner syndrome ptosis and miosis w/out anhidrosis (sympathetic fibers responsible for facial diaphoresis travel along external carotid a)

30 yo w/ right-sided headache that began 30 min ago and that has gradually worsened to severe pain over the next several minutes. He has had episodic headaches in the past. He has had nausea and vomiting today and is somnolent and difficult to arouse. BP1 150/90, pulse 64, resp 14. He withdraws all extremities to painful stimuli, but left-sided DTRs are increased. No neck rigidity. What is the most likely underlying cause of his condition?

arteriovenous malformation patient w/ right-sided headache who developed nausea, vomiting, and decreased consciousness, suggests intracerebral hemorrhage (ICH) left-sided hyperreflexia suggest possible right hemispheric lobe hemorrhage ICH in a young person is most often due to arteriovenous malformation (AVM), which occurs occurs when an artery directly anastomoses with veins without interposed capillary bed - high pressure system predisposes patients to aneurysm and/or spontaneous bleeding

32 yo woman w/absent menses after giving birth 4 mos ago. Postpartum course complicated by a postpartum hemorrhage, requiring blood transfusion and emergency suction and curettage. Was started on OCPs 2 mos after birth and has had no bleeding on placebo week. Patient has increased fatigue since returning to work and is bottle-feeding. She has no headaches, galactorrhea, or hot flushes. Pelvic exam shows clear vaginal discharge throughout vault and well-rugated vagina. Uterus is small and anterverted, and there are bilateral small, nontender ovaries. UPT negative and FSH and TSH are normal. What is the most likely diagnosis?

asherman syndrome - formation of intrauterine adhesions that can follow intrauterine surgery - because of complete or partial obliteration of the endometrium, patients typically develop very light menses or secondary amenorrhea that doesn't respond to the progesterone challenge (evidenced by no withdrawal bleeding during placebo week) - hysterectoscopy for dx and lysis of adhesions **sheehan syndrome can occur as complication of postpartum hemorrhage and causes amenorrhea, fatigue, and inability to breastfeed, but FSH and TSH would be low due to panhypopituitarism

17 yo girl w/ acute abdominal pain, nausea, and vomiting. Has irregular menstruation and isn't sexually active. Mom had dvt following knee replacement and takes apixaban. Temp 101. PE shows RLQ tenderness. CT reveals acute appendicitis. Preop labs show PLT 400,000, PT 12, INR 1.2, PTT 52, thrombin time 15 (norm: 14-19). Mixing pt's plasma sample w/ norm plasma fails to correct coag abnormalities. What best explains coag studies?

autoantibodies against phospholipids pt's PTT doesn't correct when mixed w/ norm plasma - normal plasma provides coag factors; therefore, it corrects prolonged PTT in the setting of coag factor defic but fails to correct PTT in setting of coag inhibitor one of the most common inhibitors is lupus anticoagulant (LA), a type of antiphospholipid antibody - LA binds to phospholipids used in most PTT tests and prevents them from inducing coag (prolonged PTT) - not improved w/ addition of plasma, but resolves w/ addition of phospholipids (will eventually overwhelm all antibody binding sites **PTT is sometimes prolonged in VWF defic but a mixing study would correct the PTT

70 yo w/ parkinson disease who has occasional dizziness and syncope when standing. He takes levodopa and carvidopa. Seated BP is 120/70 and HR is 66. Standing is 89/60 and HR is 68. Most likely cause of his dizziness?

autonomic insufficiency - Neurogenic orthostatic hypotension (NOH) is a common complication of Parkinson disease - NOH occurs due to autonomic dysfunction that may result from degenerative changes in the autonomic ganglia or CNS nuclei; there is impaired release of Norepinephrine and consequent failure of vasoconstriction and increase HR

3 yo girl w/ apnea and cyanosis. Mother reports multiple episodes over last 48 hrs of gagging and gasping for air. During the last event, the patient started coughing harshly during a feeding and stopped breathing (cyanotic for 12 sec and quickly returned to normal). 1 wk hx of clear rhinorrhea. Pt is is well developed and active w/nasal congestion and clear rhinorrhea. Lungs are clear except for referred upper airway congestion. Most appropriate tx?

azithromycin (pertussis) **Rantidine is for tx of GERD. emesis associated w/ GERD is postprandial rather than posttussive, and GERD doesn't cause rhinorrhea or congestion

19 day old girl who comes to ED due to difficulty feeding. Seemed healthy until 2 days ago when she stopped waking for feedings and seems sleepier and fussier. Lives at home w/parents and 3 cats. Temp 95.2, BP 78/52, pulse 150, resp 62. Infant has full fontanelle, jaundice of chest, mild scleral icterus, and dry mucous membranes. Clear lung fields and no murmur present. She is lethargic w/decreased tone. Total and direct bilirubin and CBC w/diff are pending. Next step in management?

bacterial cultures and antibiotics (neonatal sepsis) - should be considered in in any baby under 28 days w/decreased activity and poor feeding **a peripheral smear and retic count should be obtained if hemolysis is the suspected cause of jaundice. However, lethargy and hypothermia make neonatal sepsis a much more probable cause of scleral icterus

3 yo boy is refusing to walk. Has had runny nose and fever intermittently for 2 mos since starting day care. Temp 100.4, pulse 90, resp 24, puls ox 100%. Has clear rhinorrhea and several soft and mobile anterior cervical lymph nodes approx 1cm in diameter. Liver edge is palpable and hips are flexed bilaterally. During movement of the left hip, the patient wails and resists. Normal neurological exam. Hgb 11.5, Plt 380,000, WBC 15,000 (80% neutrophils), ESR 30. Next step in management?

bilateral hip US most common cause of hip pain and limp in children is transient synovitis (self-limiting, inflammatory condition often preceded by a viral illness) transient synovitis must be distinguished from septic arthritis (potentially life threatening and requires emergency antibiotics) bilateral hip US performed because joint effusion in septic arthritis is unilateral, whereas 25% of pts w/ transient synovitis have bilateral hip effusions (even with unilateral symptoms) - if hip US shows unilateral effusion, arthrocentesis is required

A 40-year-old male with past medical history of diabetes mellitus and orange skin pigmentation presents with melena. Likely dx?

bleeding esophageal varices - dilation of venous supply secondary to portal HTN from liver cirrhosis

16 yo has right knee swelling and mild stiffness. over last several mos, he has had fatigue and occasional flulike illnesses and pain occurring in variable joints. He spent the summer in Maine. He is sexually active w/ gf. exam of right knee shows warmth and effusion and is tender to palpation, but can bare weight on it. He has decreased flexion of right knee. Aspiration yields yellow, translucent fluid w/ WBC of 20,000 (50% neutrophils) and no organisms on gram stain. Cause of knee swelling?

borrelia burgdorferi infection (lyme arthritis, hallmark of late lyme disease) early symptoms (eg, erythema migrans, fever, fatigue) are often overlooked or misdiagnosed, making joint swelling the presenting symptom **gonorrhea can cause acute, afebrile, monoarticular arthritis and chronic polyarticular athralgias. However, gram-neg diplococci are present, and disseminated infection often presents w/ tenosynovitis and pustular skin lesions

7 wk old boy w/ persistent hypoxia. Was born at 29 wks gestation and has been in NICU since birth. 6 hrs aft delivery, he developed tachypnea, grunting and cyanosis. Chest x-ray revealed ground-glass opacities. Surfactant was administered and mechanical ventilation was initiated. Ever since ventilation was discontinued, he has remained hypoxic and requires O2 admin. Resp are 50, S1 and S2 are normal, a 1/6 systolic murmur is present a lower left sternal border. Scattered rhonchi and rales are present in both lungs. Chest x-ray appears hazy bilat and demonstrated decreased lung volumes. Dx?

bronchopulmonary dysplasia (BPD, chronic lung disease of the neonate) - persistent O2 requirement w/ tachypnea, rhonchi, and haziness on xray and decreased lung volumes who had RDS - results from repeated insult to neonatal lung from factors such as mechanical ventilation, prolonged O2 exposure, and inflammation - most pts w/ BPD improve over 2-4 months; some develop pulmonary artery HTN **bronchiectasis is a result of airway remodeling and thickening due to chronic inflammation - generally assoc w/cystic fibrosis, not RDS

34 yo after motorcycle accident w/ head CT showing contusion and swelling w/out subdural or epidural hematoma. On 2nd day, ventriculostomy is placed for invasive ICP monitoring. On 3rd day, pressure is still high despite sedation, elevation of head of bed, and removal of CSF. Hyperventilation would decrease ICP how?

by causing cerebral vasoconstriction paCO2 is a potent regulator of cerebral blood flow (CBF) (much greater than paO2) as levels of cerebral paCO2 rise, so does blood flow lowering cerebral paCO2 through hyperventilation results in rapid vasoconstriction and a consequent decrease in ICP

27 yo was found unconscious at the scene of a house fire. She has black soot near nares and mouth. Capillary refill time is 4 secs, HR is 122 and regular, and respirations are 24. pH is 7.15, PaO2 is 114, PaCO2 is 33, bicarb is 12, and lactic acid is 20. What should the pt be empirically tx for?

carbon monoxide (CO) and hydrogen cyanide (HCN) poisoning - 2 major products of combustion in closes spaces - HCN is produced from combustion of nitrogen-containing synthetic polymers (eg, foam, cotton, paint, silk) - HCN is a fast-acting poison and blood levels can't be measured rapidly enough to confirm diagnosis prior to tx acid-base disturbance in this pt is likely due to reduced oxygen utilization by tissues (switch to anaerobic metabolism, leading to lactic acid formation) **metheglobinemia occurs after exposure to oxidizing agents (eg, dapsone, nitrates, topical/local anesthetics)

28 yo had heavy concrete slab fall on legs. Radiographs reveal multiple bilat lower extremity fxs. Aft pt is cleared of other major trauma, he is taken for surgical stabilization and fixation of lower extremity fxs. Succinycholine is considered for rapid-sequence intubation. With use of succinylcholine, the patient is at greatest risk of what?

cardiac arrhythmia due to electrolyte derangment succinylcholine is a depolarizing neuromuscular blocker that works by binding postsynaptic acetycholine receptors to trigger influx of sodium ions and efflux of potassium ions thru ligand-gated ion channels, leading to depolarization and temporary paralysis - in some patients, causes life-threatening arrhythmia due to sever hyperkalemia

65 yo w/ tingling, numbness, and burning pain in left hand for several weeks. He has similar symptoms in right hand but to lesser extent. He has ESRD and notices that symptoms worsen during dialysis. What is the most likely diagnosis?

carpal tunnel syndrome (the most common mononeuropathy in pts on hemodialysis) the most common cause is likely dialysis-related amyloidosis, in which inflammation stimulates the formation of beta-2-microglobulin. This is inadequately cleared and deposited as amyloid within the carpal tunnel **hypocalcemia can result in hand cramping and paresthesia , but typically occurs in assoc w/ perioral numbness and tetany (wouldn't be more severe unilaterally or worsen w/dialysis)

2 yo boy refuses to walk on left leg, resists all passive ROM of knee, and knee is warm and edematous. Knee is aspirated and synovial fluid reveals WBC of 53,000 (sent for culture). Pt is admitted to receive IV vancomycin. After 2 days, symptoms haven't improved. What medication is most appropriate to add to pt's regimen?

ceftriaxone most septic arthritis is causes by staph aureus, which vanco has good coverage against, however, gram-negative bugs like kingella kingae are also common and vanco doesn't cover gram-negatives ceftriaxone has excellent coverage against gram-pos and gram-neg

10 mo girl hasn't begun to stand. She can pull herself along the floor using her arms and drags her legs behind her. She was born at 32 wks aft mother devel preeclampsia w/ severe features. She has mild hypertonia of the UE; B/L LE are hypertonic w/ significant resistance to passive extension. Patellar reflexes are brisk, b/l feet have equinovarus deformities. Babinski reflex is present b/l. Dx?

cerebral palsy (CP) - greatest risk factor is prematurity and first sign is delayed gross motor milestones - spastic CP, as seen in this pt, is most common type and pts will present w/ hypertonia and hyperreflexia that predominately involve lower extremities - feet may point down and inward (ie, equinovarus deformity) - commando crawl is common (dragging legs) **spinal muscular atrophy is AR disorder that may present w/ delayed gross motor milestones, but pts have proximal muscle weakness and hypotonia, not hypertonia

22 yo woman at first gynecologic exam. She became sexually active a few months ago w/bf and uses condoms as contraception. Hx of yeas infection at 18. Received all recommended vaccinations except HPV. Pelvic exam shows no abnormalities. In addition to first pap smear, what is the best recommendation for the patient?

cervical swab for chlamydia and gonorrhea - 2 most common STIs and infections have no symptoms but eventually can cause infertility and ectopic pregnancy due to adnexal scarring **HPV cotesting isn't recommended from age 21-29 because most women at this age spontaneously clear infection

32 yo w/ 3 days of fever, malaise and cough productive of clear sputum. No nasal congestion, rhinorrhea, sore throat, or chest pain. Temp is 100 and puls ox is 96%. Oropharynx is normal and he has no cervical lymphadenopathy. Lung auscultation reveals crackles at the right lung base w/ occasional expiratory wheezing. Next step in management?

chest x-ray (possibly community acquired pneumonia (CAP)) **symptomatic tx alone in setting of consolidation is insufficient due to risk of secondary bacterial pneumonia - empiric antibiotics would likely be administered if CAP (even if a viral etiology is still possible)

6 yo boy w/new-onset chest pain. A week ago, he developed a low-grade fever w/ nasal congestion and a dry cough. Over the past few days he has had paroxysms of coughing that are worse at night and chest pain. Temp is 99.1, pulse is 130, and resp are 36. There is clear rhinorrhea. Posterior oropharynx and tonsils are erythematous. There is mild swelling of the neck and crepitus over the anterior chest. Lungs are clear b/l. Next step in eval?

chest x-ray (spontaneous pneumomediastinum(SPM)) patient has subcutaneous emphysema and chest pain after a consistent cough, consistent w SPM first step in eval is chest x-ray to confirm mediastinal gas and rule out a life-threatening pneumothorax that may require emergency needle thoracostomy

30 yo w/ 3 UTIs over past 4 mos that have had complete resolution with short courses of antibiotics. Sexually active and has pain with ejaculation, but no urethral discharge or fever. Smokes pack of cigs daily. Abdomen soft and nontender. No CVA tenderness. Rectal exam reveals smooth, nontender prostate. What is most likely cause of frequent UTIs?

chronic bacterial prostatitis - recurrent UTIs that transiently improve w/antibiotics and pain w/ ejaculation (discharge of prostatic fluid is irritative) - can have prostatic swelling and tenderness or normal prostate exam - need at least 6wks of antibiotic therapy to eradicate pathogen (eg, fluoroquinolone) **chronic infectious epididymitis can present w/painful ejaculation but focal tenderness over epididymitis (posterior testis) would be expected; bacteriuria and transient improvement w/antibiotics wouldn't be expected

60 yo w/ cough and dyspnea on exertion that has worsened over past 2 yrs. Cough is productive of a small volume of mucoid sputum. 40 yr pack hx and worked in as shipyard for 10 yrs. Chest x-ray reveals prominent bronchovascular markings and mild diaphragmatic flattening. FEV1 67% of predicted, FVC is 95% of predicted, FEV1/FVC ratio: 0.65. Diffusion capacity of lung for carbon monoxide (DLCO) is 100% of predicted value. Likely cause of symptoms?

chronic bronchitis - given pt w/ long smoking hx, productive cough for greater than 3mos over 2 consecutive years, prominent bronchobascular markings on x-ray, decreased FEV1/FVC ratio, and normal DLCO - emphysema has decreased DLCO **in bronchiectasis, sputum production is more prominent, chest x-ray reveals dilated conducting airways, and FVC is often low (<80% of predicted) due to airway destruction

54 yo Vietnamese immigrant who was treated for TB 10 yrs ago complains of fatigue, productive cough with blood-tinged sputum and weight loss. Has right-sided rhonchi and crackles. He has cavitary lesion on CT. What is the most likely diagnosis?

chronic pulmonary aspergillosis (risk factor of lung disease/damage)

53 yo w/ schizoaffective disorder had seizure at group home w/brief jerking movements of arms. He regained consciousness after 1-2 mins. A 1-cm laceration is noted on right side of tongue but physical exam is otherwise normal. What med most likely contributed to his presentation?

clozapine - causes neutropenia/agranulocytosis, seizures, and myocarditis - highest risk of seizures out of antipsychotics **carbamazepine wouldn't cause seizures (anticonvulsant) - can cause nausea, vomiting, rash, pruritis, drowsiness, and blurred vision

34 yo male is a military pilot and presents w/hearing loss. He denies pain, tinnitus, or vertigo. He takes no medications. He has clear, intact tympanic membranes bilat w/no middle ear effusion. Audiometry reveals mild, bilat, high-freq hearing loss. What is the most likely cause?

cochlear hair cell damage loud noises may induce sensorineural hearing loss (SNHL) via both mechanical damage and metabolic overload of the cochlear hair cells (results in bilateral, high-frequency SNHL)

6 wk old baby w/poor feeding. Mom says aft a few minutes of breastfeeding he begins sweating and grunting and stops nursing. He was born at home and has down syndrome. Weight is < 5th percentile. Auscultation reveals fixed split S2 and a grade 2/6 systolic ejection murmur heard best at left upper sternal border. Bibasilar crackles are present. What congenital heart defect is likely present?

complete atrioventricular septal defect (most common congenital heard defect in kids w/ down syndrome) - failure of endocardial cushions to merge results in VSD and ASD - a common AV valve also occurs due to poor mitral and tricuspid valve development HF results from mixing of blood between the chambers and severe AV valve regurg, leading to volume overload and excessive pulmonary blood flow Auscultation can reveal any of the following: - Fixed split S2 due to delayed pulmonary valve closure from flow across ASD - Systolic ejection murmur from increased flow across pulmonary valve due to left-to-right shunt across the ASD - holosystolic murmur of VSD that may be soft or absent if defect is large - holosystolic apical murmur depending on degree of AV valve regurg

2 day old infant w/ length in 10th percentile and weight and head circumference in the 25th percentile. Is alert w/normal tone. Pt found to have ambiguous genitalia. Abdomen is soft w/no palpable masses. There is severe curvature of an underdeveloped phallus. The urethral meatus is at the base of the structure, flanked by labioscrotal folds; the gonads arent palpable. Exam of mother shows no acne. Na+ 138, K+ 4, glucose 70, testosterone 850, karyotype 46, XX. Dx?

congenital adrenal hyperplasia - electrolyte abnormalities due not occur until 1-2 wks **sertoli-leydig cell tumors produce (eg, 17-OHP, testosterone) and cause virilization in female patients; however, presentation is typically in early adulthood, not the newborn period

2 yo who had abrupt onset of dysphagia, drooling and vomiting after eating sausage 2 wks ago. she had urgent esophagoscopy, which showed complete occlusion of the esophageal lumen w/ impacted food. There were no mucosal lesions or strictures at site of impaction. Barium esophagography revealed a deep impression on the posterior aspect of the esophagus at T4 level. This was her 2nd episode of foot impaction. PMH of frequent respiratory infections, including 2 episodes of middle lobe pneumonia last yr. She has no problem with liquids, but has frequent choking w/solid foods. Her weight was in 50th percentile until 9 mos ago and now it is at the 15th percentile. Most likely etiology of condition?

congenital vascular malformation (vascular ring) vascular rings encompass congenital malformations of the aortic arch system that encircle the trachea and/or esophagus and cause compressive symptoms pt was gaining weight well until introduced to solid foods around 6mos she has had recurrent food impactions at the site of esophageal compression, seen on fluoroscopic esophagography at around the level of the aortic arch (T3-T4) Her episodes of middle-lobe pneumonia were possibly due to aspiration

66 yo w/ SOB that began a week ago w/ dry cough and exertional dyspnea. Has PMH of HTN and recent stenting for double-vessel CAD. Was hospitalized for pneumonia 6 mos ago and has 35yr pack hx. BP is 160/90 and pulse is 90 and regular. He is in mild respiratory distress and has decreased breath sounds and lung bases, bilateral crackles, and occasional wheezes. pH 7.46, pO2 73, pCO2 31. Most likely explanation for symptoms?

congestive heart failure exacerbation - Hx of HTN, smoking, and CAD w/ bibasilar crackles and worsening SOB **pulmonary embolism presents w/ acute-onset dyspnea, tachypnea, and pleuritic chest pain. Pts have hypoxemia and respiratory alkalosis w/ widened A-a gradient - bilateral crackles and progressive course make CHF exacerbation more likely in this pt

Best contraceptive for a woman w/ antiphospholipid syndrome

copper IUD not condoms and spermicide and nothing with estrogen

69 yo man w/ 2 days of increased SOB, cough, and lower-extremity edema. Drinks half bottle of vodka a day and has 45-pack-year hx. Has sedentary lifestyle and hasn't seen doctor in 20 yrs. O2 90%, cardiac exam shows faint heart sounds, JVP 11, decreased breath sounds, liver span is 18 cm and ascites is present, 3+ lower-extremity edema. Xray shows enlarged central pulmonary arteries without evidence of vascular congestion. Likely diagnosis?

cor pulmonale - right heart failure from pulmonary HTN, most commonly due to COPD

52 yo w/ irritation, grittiness and foreign-body sensation in her eyes. No skin rashes or joint tenderness, but the lips are cracked, and the oral mucosa is dry. What is a potential eye complication associated with this patient's condition?

corneal ulceration (Sjogren syndrome) decreased tear volume leads to hyperosmolar state on the surface of the eye, resulting in irritative symptoms and promoting inflammatory response. Potential complications include decreased visual acuity, superficial infection, corneal ulceration, and, occasionally corneal perforation

21 yo has tonic clonic seizure and is brought to ED. Has hx of seizure disorder, but not complaint w/meds. Was given lorazepam, thiamine, and glucose en route but continued sizing. He is unresponsive and cyanotic and clonic jerks of all extremities are observed. BP 96/54 and pulse 152. He is at highest risk for what?

cortical necrosis (from status epilepticus (seizures lasting over 5 mins)) at increased risk of developing permanent injury due to excitatory cytotoxicity cortical laminar necrosis is the hallmark of prolonged seizures anc can lead to persistent neurologic deficits and recurrent seizures

5 yo boy w/ recent clumsiness that began since last visit (running into things around the house). He has also been drinking water and urinating "all the time". PE shows dry mucous membranes and loss of peripheral vision. On CT there is mass seen in suprasellar region that appears calcified. What is likely diagnosis?

craniopharyngioma - presents w/bitemporal hemianopsia and pituitary hormone deficiencies (eg, diabetes insipidus, growth failure) **pituitary adenomas secrete prolactin. Can also present w/bitemporal hemianopsia but don't calcify **Rathke cleft cysts can cause visual field defects and endocrinopathies if large, but don't calcify

A patient with Hemophilia A is scheduled for an elective sigmoidectomy for treatment of his diverticular disease. Which of the following is the most appropriate blood product to have available in case of intraoperative bleeding?

cryoprecipitate - rich in factor VIII and fibrinogen and will help coagulation in hemophilia A patients

43 yo w/ acute pancreatitis develops respiratory distress on day of of admission He is transferred to ICU and intubated. Ideal body weigh is 65kg. Ventilator settings: fraction of inspired O2: 0.8 (80%), RR 14, Tidal volume: 380mL, positive end-expiratory pressure: 7cm H2O. Ten minutes later, BP is 110/70 and HR is 90. ABGs are: pH 7.42, paO2 105, paCO2 37. Best next step in management?

decrease the fraction of inspired oxygen (ARDS) mechanical ventilation improves oxygenation by providing an increased fraction of inspired oxygen (FiO2) and positive end expiratory pressure (PEEP) to prevent alveolar collapse the goal is to maintain arterial partial pressure of oxygen (PaO2) at 55-80 mmHg (O2 sat of 88-95%) immediately following intubation, a high FiO2 (> 60% or 0.6) is usually provided, and ventilator settings can be adjusted based on ABG the PaO2 is influenced mainly by FiO2 and PEEP. The PaCO2 is a measure of pulmonary minute ventilation and is affected mainly by RR and tidal volume this patient had normal PaCO2 and more than adequate oxygenation - prolonged, high FiO2 can lead to oxygen toxicity as it can lead to formation of proinflammatory oxygen free radicals and predispose to atelectasis as alveolar nitrogen is displaced, resulting in worsening oxygenation - need to decrease this pt's FiO2

38 yo female g1p1 who comes into office due to abnormal uterine bleeding. She underwent menarche at age 15 and irregular menses that normalized with contraceptives. 4 yrs ago she had tubal ligation after cesarean delivery. Over the last year her menses have become increasingly irregular and now occur every 2-3mos w/ heavier bleeding and passage of clots. She has difficulty concentrating at work due to increased irritability and depressed mood. She has no headaches or hot flashes and vitals are normal. Skin is cool and dry. Pelvic exam reveals well-rugated vagina, mobile uterus, and small, nontender ovaries. Pregnancy test is negative. What would her lab findings likely be?

decreased FSH, increased TSH and prolactin (she has hypothyroidism) **elevated FSH with normal TSH and prolactin would be seen in menopause

36 yo female comes in to ED due to severe headache and vision disturbance. Symptoms began an hr ago and she feels weak, dizzy, and nauseated. Had prolactinoma dx a month ago and is on cabergoline. BP is 80/50 supine and 60/40 standing and pulse is 110. She has mild ptosis and impaired adduction to right. Bitemporal hemianopsia is present. Strength is normal, neck veins are flat, heart and lung sounds are normal, and there is no abdominal tenderness or extremity edema. Most likely cause of her hemodynamic changes?

decreased adrenal hormone levels (pituitary apoplexy) - suspect in patient w/ hx of pituitary adenoma w/ severe headaches, ptosis and vision changes pituitary apoplexy typically involves sudden hemorrhage due to enlarged pituitary adenoma (eg, prolactinoma) - bleeding into tight space of sella trucica causes severe headaches, visual disturbances, eye dysfunction and typically loss of all pituitary fxn (loss of ACTH leading to adrenal crisis) **loss of central sympathetic output (neurogenic shock) would cause hypotension, but also bradycardia. Extremely unlikely in pt w/ normal strength and sensation

38 yo comes to ED w/ high-grade fever, shaking chills, productive cough, and SOB. He's had 2 admissions for alcohol withdrawal seizures in the past yr, but continues to drink. Temp 103.5, resp 20 and pulse is 110. Skin and mucous membranes are dry. PE shows crackles and bronchial breath sound in right lower lung field. Cardiac exam is normal. X-ray reveals right lower lobe consolidation. Started on IV antibiotics and 12 hrs later develops significant SOB. Resp 38 and O2 sat is 80% on nonrebreather mask. He is intubated emergently and chest x-ray shows bilateral alveolar infiltrates. What was most likely present in patient just prior to intubation?

decreased lung compliance (ARDS) ARDS is an inflammatory condition that can develop in setting of infection (eg, sepsis, pneumonia), trauma, or other conditions (eg, massive transfusion, pancreatitis) Lung injury causes release of proteins, inflammatory cytokines, and neutrophils into alveolar space. This results in leakage of bloody and proteinaceous fluid into the alveoli, alveolar collapse due to loss of surfactant, and diffuse alveolar damage. As a result: - gas exchange is impaired due to V-Q mismatch - lung compliance (ability to expand) is decreased (stiff lungs) due to both loss of surfactant and increased elastic recoil of edematous lungs - pulmonary arterial pressure is increased (pulmonary HTN) due to hypoxic vasoconstriction, destruction of lung parenchyma, and compression of vascular structures from positive airway pressure in mechanically ventilated patients **PaO2 decreases and leads to an increased fraction of inspired oxygen (FiO2) requirement (decreased PaO2/FiO2 <300)

30 yo man presents w 2 days of right arm pain, swelling, and heaviness that is worse w/ activity and improves w/ arm elevation and rest. He likes to lift weights and pitched 6 innings several hours prior to onset of symptoms. Vitals are normal. He has swelling and redness of the right arm from elbow to shoulder. Capillary refill is normal. What is most likely cause of symptoms?

deep vein thrombosis -due to thoracic outlet abnormality that compressed or injured axillosubclavian vein - probably from hypertrophy of scalene/subclavius muslces from weight lifting or damage of the vein from repetitive overhead throwing **superficial thrombophlebitis is generally marked by localized pain and swelling along the pain (edema and heaviness of extremity unlikely)

3 yo girl w/severe oral pain that is causing her to refuse any oral intake. Had a similar episode over past yr. Has had 2 sinus infections and numerous episodes of cellulitis w/ S aureus and S pyogenes. PE shows periodontal inflammation w/ ulceration and necrosis. Hgb 11.8, WBC 55,000 w/90% neutrophil. What abnormality is likely present?

defective leukocyte adhesion (leukocyte adhesion deficiency) - lack of neutrophil migration causes recurrent skin and mucosal infections and poor wound healing - exam will show inflammation and lack of purulence - marked neutrophilia, particularly during infection **in chronic granulomatous disease children have infections w/catalase + ogranisms (s pyogenes in not) and wouldn't have neutrophilia

56 yo man w/new-onset lethargy and confusion w/hx of cirrhosis secondary to alcoholism. Is on spironolactone and furosemide. Temp is 100.4, pulse is 112, and resp 24. PE reveals flapping tremor, labored breathing but clear lungs, and shifting dullness and diffuse abdominal tenderness. He has pitting edema in LE. WBC 12,000, Tbili 2.5, AST 110, ALT 52, ALP 125, Creatinine 1.4, and Albumin 2.1. Abdominal x-ray reveals gas in the small and large bowels w/out air-fluid levels. Next step in management?

diagnostic paracentesis (spontanous bacterial peritonitis and heptatic encephalopathy) - pt w/cirrhosis accompanied by fever, abdominal pain, and lethargy

2 yo boy w/right eye mildly deviated toward the nose, and asymmetric pupillary red reflexes with no proptosis in intact, nonpainful extraocular movements. What is the the best next step in management?

dilated fundoscopic exam (pt has strabismus, or ocular misalignment) - used to assess for secondary causes, particularly retinoblastoma, that can be visualized on fundoscopy as a nodular, cream-colored retinal mass **MRI of brain and orbits can confirm diagnosis of retinoblastoma if a retinal mass is seen on dilated fundoscopic exam

35 yo g2p1 at 31 wks comes to physician with severe heartburn and RUQ pain w/ nausea and vomiting this morning. BP was 130/80 at last visit and is now 160/90, Hgb 9.7, PLT 80,000, ALP 170, AST 112, ALT 124, 2+ protein in urine. What is the most likely cause of her upper abdominal pain?

distension of liver capsule (HELLP syndrome) serious liver problems include centrilobular necrosis, hematoma formation, and thrombi in the portal capillary system - these processes can cause liver swelling and distension of the hepatic (Glisson's) capsule, leading to RUQ or epigastric pain

32 yo man has 2 day hx of fever, headache, malaise, myalgias, and slight confusion after being bit by a tick 2 weeks ago in Arkansas. He's febrile and tachycardic. No lymphadenopathy, rash, or focal neurologic deficits. Hgb 14, Plt 78,000, WBC 2500 (10% monocytes), ALP 110, AST 98, ALT 105. What is next best step in management?

doxycycline (human monocytic ehrlichiosis) - presents with flu-like symptoms, confusion, leukopenia and thrombocytopenia, and elevated LFTs and lactate dehydrogenase

62 yo woman w/ weeklong hx of brief episodes of sharp chest pain that worsens w/deep inspiration. She also has fatigue and stiffness and pain in the hands and knees. PMH of obesity, HTN, and non-ischemic cardiomyopahty w/ LVEF of 40%. Meds include carvedilol, furosemide, hydralazine, and isosorbide mononitrate. Pt developed angioedema from lisinopril 6mos ago. Temp is 100, BP is 134/78, and pulse is 74 and reg. No JVD or heart murmurs, but inspiratory rub is present on lung auscultation. There is moderate swelling and tenderness of hand and knee joints b/l. No rash is present. Most likely cause of symptoms?

drug adverse effects (drug-induced lupus erythematous from hydralazine) - common symptoms: arthralgia, fever, or serositis (seen in this patient w/ new pleuritic chest pain and pleural rub) - rash is less common than in non-drug-induced SLE

67 yo man w/abdominal discomfort and distension. 5 days ago, began having large-volume watery stools 6-8x/day after eating at a new restaurant. His wife at there too, but has no symptoms. He began to consistently improve a few days ago but then his bowel movements abruptly stopped yesterday, and diffuse abdominal pain and distension worsened. PMH HTN, CAD, HF w/diuretics. BP is 116/72 and pulse is 102, afebrile. Abdomen is distended, tympanic, and mildly tender to palpation w/no guarding or rebound tenderness. Bowel sounds are decreased. Abdominal CT shows colonic dilation w/nondilated small bowels. What likely contributed to condition?

electrolyte abnormality (acute colonic pseudoobstruction (Olgilvie syndrome)) - abdominal distension w/colonic dilation on CT - diuretic use and recent diarrhea (due to foodborne gastroenteritis) likely caused an electrolyte imbalance (eg, hypokalemia, hypomagnesemia) that precipitated it

75 yo diabetic w/ fevers, chills, abdominal pain, nausea, and vomiting for past 2 days. Abdomen mildly distended. There is marked tenderness over epigastrium and RUQ w/ guarding. Bowel sounds are decreased. Blood glucose 340. Abdominal US reveals thickening of gallbladder wall, pericholecystic fluid, and multiple hyperechoic calculi. Air is noted within the gallbladder wall, and the common bile duct isn't dilated. IV fluids, insulin, and broad spectrum antibiotics are initiated. What is the best next step in management?

emergent cholecystectomy (gas in gall bladder wall = emphysematous cholecystitis

31 yo had pap testing last week that revealed high-grade squamous epithelial lesion (HSIL). Pt has regular menstrual periods and LMP was a week ago. Colposcopy reveals a nulliparous cervix w/no raised or ulcerated lesions. The entire squamocolumnar jxn can't be visualized. Next best step?

endocervical curettage HSIL on pap is concerning for underlying high-grade cervical intraepithelial neoplasia (CIN) Pt's colposcopy revealed no raised ulcerated lesions, but, entire jxn not visualized. Because most at risk area cant be seen in entirety, eval is unsatisfactory, and occult CIN or cancer could be missed. Therefore, best next step is endocervical curettage, which can sample tissue from transformation zone to assess for lesions **endometrial biopsy is for pts w/ abnormal uterine bleeding (sign of endometrial cancer)

42 yo woman presents for wellness exam and pap test shows atypical glandular cells. Next step in management?

endometrial biopsy - atypical glandular cells on pap testing may be due to either cervical or endometrial cancer - all patients over 35 require eval for endometrial cancer in addition to cervical pathology

56 yo obese female comes in for regular physical. Speculum exam shows no cervical abnormalities. Pap smear shows endometrial cells and no cervical intraepithelial lesions. Next best step in management?

endometrial biopsy - women over 45 shouldn't have endometrial cells reported on pap smear - in this age group, endometrial shedding may be due to endometrial hyperplasia/cancer, regardless if the pt has no other symptoms (eg, abnormal uterine bleeding) - if woman was under 45, these endometrial cells would be a normal, benign finding

38 yo nulligravid woman comes to the office due to persistent abnormal uterine bleeding for the past 8 mos. She has had instermenstrual spotting and bleeding that have occurred at varying intervals and last 3-7 days. She started OCPs 4 mos ago but they haven't helped bleeding pattern. Speculum exam shows dark red blood in the posterior vaginal vault but no cervical or vaginal lesions. Hgb 12.2, prolactin 5, and TSH 1.8. UPT is neg. Pelvic US shows an anteverted uterus and no adnexal masses. Next step?

endometrial biopsy in women under 45, the absolute risk of endometrial hyperplasia/cancer is low; therefore, they can be started on combined OCPs w/out eval of endometrium - estrogen component regulates the menstrual cycle and builds the endometrium; the progestin component sheds the endometrium this patient failed medical management. the endometrial lining is likely too thick for progestin to completely shed it during menstruation; as a result, the unshed endometrium continues to undergo dysregulated proliferation, which leads to an increased risk of hyperplasia/cancer and warrants and endometrial biopsy **a progresterone withdrawal test is used to eval secondary amenorrhea (no menses >6 mos) to determine if the amenorrhea is from low estrogen levels (ie, no bleeding after progesterone)

77 yo w/ 2 wk hx of fevers and generalized weakness. Had pyelonephritis 2mos ago and was hospitalized, an episode of rheumatic fever as a child, and Hodgkin's lymphoma tx w/chemo 15 yrs ago. Recently underwent cystoscopy for eval of persistent dysuria. Temp 100, BP 150/86, pulse 98. He is slightly diaphoretic, has new II/VI holosystolic murmur at apex and tender erythematous lesions affecting several fingertips. What bacteria is most likely responsible for his illness?

enterococci - enterococci, especially enterococcus faecalis, are a common cause of endocarditis associated w/ nosocomial UTIs **strep pyogenes causes pharyngitis, cutaneous infections (eg, pyoderma cellulitis), and complications associated w/ strep infection (eg, acute rheumatic fever, glomerulonephritis) - doesn't cause infective endocarditis

21 yo has painful blisters on hands that began after playing golf yesterday. She also gets blisters during hiking and competitive sports on her feet. When she was an infant, she had oral ulcers that resulted in poor feeding. PE shows bullae and erosions on both palms. Skin of soles is moderately thickened. Most likely dx?

epidermolysis bullosa - inherited disorder characterized by epithelial fragility (eg, bullae, erosions, ulcers) triggered by minor trauma **porphyria cutanea tarda causes blisters and skin fragility, but mostly on sun-exposed areas (oral mucosa not involved)

A 22-year-old male presents to the emergency department with a right-sided head pain after being hit in the head with a blunt object by his roommate. Vital signs reveal a blood pressure of 138/92 mmHg, heart rate of 88/min, and respiratory rate of 12/min. Physical examination reveals mild confusion, left upper and lower extremity weakness. The most likely diagnosis is:

epidural hematoma (rupture of middle meningeal artery, a branch of maxillary artery) - commonly occurs following a fracture or heavy blow to the temporal bone

34 yo man w/difficulty swallowing and blurred vision starting last night and mild SOB that started today. Was on 1wk long Caribbean vacation where he snorkeled, at locally cured fish, and hiked in forest. Had mild URI 2 wks ago that resolved spontaneously. Oral mucosa is dry and speech is slurred. Pupils are dilated and react sluggishly to light. Neck muscles are weak and he has difficulty holding head up. 1/5 strength b/l UE. No sensory abnormalities. What is the most appropriate next step in management?

equine antitoxin therapy (foodbone botulinism from cured fish) - symmetric descending weakness

64 yo woman comes to ED w/ headache and nausea over last 2 days. She has ESRD due to advanced diabetic nephropathy and recently started hemodialysis. Is taking erythropoietin and iron for anemia due to CKD. PMH of HTN, osteoporosis, and hypothyroidism. BP is 210/121, pulse 76. Ophthalmoloscopic exam shows bilat retinal hemorrhages. Glucose is 186. Most likely cause of current condition?

erythropoietin therapy (hypertensive crisis) - up to 30% of patients on EPO therapy develop new or worsening HTN - must monitor BP of those on EPO - pts who receive large doses or experience a rapid rise in Hgb are at highest risk **fibrinoid necrosis of renal arterioles is.a sequela rather than a cause of severe uncontrolled HTN

42 yo man was hit by car while crossing intersection. Intubated due to respiratory distress and given IV fluids. There is diffuse bruising across the chest and CT scan reveals bilateral mildly displaced rib fxs w/underlying pulmonary contusions and a right sided pneumothorax. Chest tube is placed. One day later chest tube is draining turbid green fluid. Repeat x-ray shows resolution of pneumothorax but there is a new right-sided pleural effusion. What is the most likely diagnosis?

esophageal perforation - blunt thoracic trauma can rapidly increase intraesophageal pressure enough to rupture esophagus - pleural effusion analysis typically reveals unusual color (eg, green), low pH, and high amylase

72 yo from South Korea comes into office with upper abdominal pain and weight loss. Pain is worse when eating food and sometimes relieved with over-the-counter antacids. He hasn't had dysphagia, melena, or rectal bleeding. BMI is 172, mucous membranes are dry, and cardiopulmonary exam is normal. Epigastric fullness and tenderness is present, but there is no hepatosplenomegaly. What is the best next step in management?

esophagogastroduodenoscopy (suspicious for gastric cancer) **H pylori is major risk factor for gastritis, gastric ulcer, and gastric cancer. However, EGD is required prior to H pylori stool testing to establish underlying stomach pathology

60 yr old g3p3 w/SOB over past 6 mos. She has difficulty taking deep breaths and can't fit into any of her pants lately despite a decreased appetite and nausea. Sister had BRCA+ breast cancer. Pelvic exam shows firm, nodular, non-mobile mass in the left adnexa. What is the most appropriate best step in management in this pt?

exploratory laparotomy (epithelial ovarian carcinoma) exploratory laparotomy with cancer resection and inspection of the entire abdominal cavity for metasases (ie, surgical staging) is required with high level of suspicion

26 yo w/ seizure disorder and medication nonadherence has seizure and is found to have displaced right femure fx. Levetiracetam is restarted and he undergoes intramedullary nailing of fx w/no intraoperative complications. Next day he is confused, temp is 99, bp 142/86, pulse 102, and resp 28. O2 sat is 90%. Pt has no focal weakness or sensory loss. No skin rashes or hematomas. Hgb 12.4, plt 103,000, glucose 118, pH 7.47, paO2 58 and paCO2 34. CT pulmonary angiography shows bilateral scattered ground-glass opacities but now filling defects w/in pulmonary vasculature. Most likely cause of current condition?

fat embolism syndrome - fx of femur (marrow-containing bone) - neurologic dysfunction - respiratory distress - petechial rash only present in half of cases - thrombocytopenia may occur due to platelet adherence and aggregation to circulating fat globules - pts typically develop pulmonary edema aft 24-48 hrs (bilateral ground glass opacities) **cardiogenic pulmonary edema explains respiratory distress; however, if would not explain the confusion unless cardiogenic shock is present - lack of hypotension rules out shock

6 mo boy coming in for follow-up after UTI. Renal US performed 2 days ago showed mild right hydronephrosis; voiding cystourethrogram revealed dilation of right ureter with blunting of the right renal calyces. PE shows circumcised male w/normal abdominal exam. If untreated, what is the most likely long term complication of the patient's condition?

fibrosis of renal interstitial space (vesicoureteral reflux (VUR) = retrograde flow of urine from bladder into ureter +/- renal pelvis) Patients w/ VUR are at risk of developing frequent UTIs and pyelonephritis due to reflux of nonsterile urine Bacterial penetration of the renal parenchyma induces inflammatory response that can lead to ischemia and subsequent renal scar formation - scar is focal and manifests as wedges of fibrotic tissue due to chronic interstitial inflammation

58 yo man w/ 3 mo hx of right ear pain. He is a welder and has smoked a pack of cigs daily for 40 yrs and drinks a case of beer each weekend. Exam of ear shows no abnormalities. Palpation of the temporomandibular joint elicits no tenderness or crepitus. Oral exam shows poor dentition, put no ulcers. There is a nontender 2-cm lymph node on the right side of his neck. What is the next best step in management?

flexible laryngopharyngoscopy otalgia in the setting of normal ear exam is likely referred pain and the most common causes of referred otalgia are dental disease and TMJ. however, referred otalgia is commonly a presenting symptom of mucosal head and neck squamous cell carcinoma, especially in older patients w/ smoking hx, alcohol use, occupational exposure to welding fumes, and cervical lymphadenopathy

58 yo man w/ 40 pack yr hx, cough, hemoptysis, weight-loss and temporal wasting has Na+ of 124. no other sx. Best initial step in tx?

fluid restriction (SIADH due to small cell lung cancer) **hypertonic saline reserved for severe hyponatremia (seizures, coma) **isotonic saline would result in net free water retention and worsening of hyponatremia in those w/ SIADH

66yo comes into ED w/worsening abdominal pain. Has had lower abdominal pain, nausea, anorexia, and constipation over past wk. This morning has severe, sudden lower abdominal pain accompanied by light-headedness and vomiting. Pain initially improved but then gradually intensified to involve whole abdomen. PMH CAD and remote appendectomy. Temp 100.9, BP 110/54, pulse 108, resp 20. Bowel sounds are diminished and the abdomen is diffusely tender w/ guarding and rebound tenderness. Imaging would likely show what?

free air in peritoneal cavity (diverticular perforation after week of diverticulitis) sudden, severe abdominal pain that temporarily improved and then spread to entire abdomen with fever, peritonitis (eg, rebound tenderness, guarding) is most consistent w/diverticular perforation **dilated small bowel w/transition point would be seen in SBO. More likely to have crampy pain in midabdomen and increased, high-pitched bowel sounds without peritonitis **embolic occlusion of mesenteric a causes acute-onset, severe abdominal pain (acute mesenteric ischemia), but pain is typically out of proportion to benign abdominal exam and risk, like a fib, is often present

3 day old boy whose arms and legs began rhythmically jerking around 20 mins ago for one minute duration. He has also had persistent vomiting, especially after feeding, and an episode of loose, nonbloody stool. Has been exclusively breastfed. Exam shows lethargic newborn w/ diffuse hypotonia as well as jaundice. Liver edge is palpable. Glucose 30, Tbili 9.3, Dbili 4, urinalysis is negative for glucose w/ 3+ reducing substances. Dx?

galactosemia - galactose (in breast milk and standard formula) cannot be reduced to glucose - presents in first few days of life after ingestion of galactose or its precursor lactose - effected newborns develop jaundice and conjugated hyperbilirubinemia due to liver disease; some also have unconjucated hyperbilirubinemia due to hemolysis - seizure cause by hypoglycemia may be the presenting symptom and vomiting is also common - exam reveals hepatomegaly (due to galactose accum), lethargy, and hypotonia - nonglucose reducing substances suggest galactosuria **glycogen storage disorders can present w/ hepatomegaly and seizures, but typically dont occur until 3-6 mos because they are causes by periods of fasting, which begin when infant sleeps longer throughout the night

62 yo woman w/ DM2 and poorly controlled HTN has symptoms of dysuria and urinary frequency and urinalysis shows hematuria, leukocytes, positive nitrites, and 2+ proteinuria. She is tx and symptoms subside, but urinalysis still reveals 2+ protein. What is most likely responsible for her persistent proteinuria?

glomerular basement membrane changes (diabetic nephropathy) glomerular basement membrane changes include: - glomerular hyperfiltration - glomerular basement membrane fibrosis and thickening - interstitial fibrosis, mesangial thickening and nodules (kimmelstiel-wilson lesion) **atherosclerotis vascular disease can lead to renal artery stenosis, the most common cause of secondary HTN in adults; however, this patient's persistent proteinuria is an uncommon finding

A patient has been NPO for 48 hours after a small bowel obstruction. Which of the following is most likely depleted in this patient?

glycogen After 24-48 hours of starvation, the glycogen stores are depleted and the body must switch to alternative sources of energy. The majority of glycogen is found in skeletal muscle, but some is also stored in the liver.

60 yo who wen thru menopause at 53 has noticed some postcoital bleeding over the past 4 mos. Over past 3 mos has had daily vaginal spotting and sometimes bleeding similar to 1st day of menstrual period. Speculum exam shows small amount of blood in the vaginal vault and normal cervix. Bimanual exam reveals right adnexal fullness and slightly enlarged uterus. Hgb 11.8. US shows 11-cm solid ovarian mass and no free fluid in pelvis. Endometrial bx shows complex hyperplasia w/out atypia. Dx?

granulosa cell tumor - postmenopausal woman w/ large ovarian mass and bleeding

45 yo w/ increasing difficulty sleeping, decreased concentration, and fatigue. She has had recent 10lb weight gain recently. No mood changes, hair loss, weakness, or changes to bowel and bladder fxn. Menstrual periods have been irregular for over 2 yrs; lmp was 3 mos ago. She is sexually active. She is obese. On bimanual exam, the uterus is symmetrically enlarged. There are no palpable adnexal masses. Next step in management?

hCG level - insomnia, amenorrhea, weight gain, and an enlarged uterus are consistent w/ pregnancy **TSH levels for people w/suspected hypothyroidism - pt doesnt have a lot of typical symptoms like mood changes, hair loss, weakness, and changes in bowel function - thyroid disorders don't cause uterine enlargement

16 yo boy w/ pain and limited motion of right knee that began 6 mos ago. Has had previous episodes of sudden swelling of knee w/out injury. Pain was initially controlled w/NSAIDs but has worsened over last few mos. Had an episode of prolonged bleeding after tooth extraction several years ago. Sexually active and uses condoms intermittently. Right knee is swollen and boggy w/small effusion and can't go into full extension. Most likely cause of findings?

hemosiderin deposition and fibrosis (Hemophilia)

55 yo w/abdominal distension. BP of 152/87, w/ grossly enlarged and nontender abdomen and shifting abdominal dullness. Trace pitting edema is present in bilateral lower extremities. Paracentesis reveals bloody ascitic fluid. Repeat paracentesis from another site yields similar bloody fluid. What is most likely responsible for presentation?

hepatocellular carcinoma (HCC) - persistently bloody ascites found on multiple paracenteses suggests underlying malignancy - HCC is most common malignancy to present w/bloody ascites due to tumor growth disrupting and eroding nearby blood vessels **portal vein thrombosis doesn't commonly cause ascites because the obstruction occurs proximal to the hepatic sinusoids **hepatic vein thrombosis (budd-chiari syndrome) can cause ascites, but typically causes non-bloody, straw-yellow ascites in association w/fever and abdominal pain

55 yo w/ 2 yrs of productive cough with yellow sputum. First episode lasted nearly 3 wks w/chest congestion, cough productive of purulent sputum, and SOB. She was diagnosed w/ bronchitis and given antibiotics. Has 6 more episodes since then w/yellow sputum (sometimes w/blood) and sinus congestion. She has no fevers, chills, chest pain, pets, recent travel and has never smoked cigarettes. Has diffuse rhonchi and wheezes with coarse crackles bilaterally in lung bases. X-ray shows linear atelectasis in mid and lower lung fields bilaterally. What is the most appropriate step in confirming diagnosis?

high resolution CT of chest (bronchiectasis) - will show bronchial dilation, lack or airway tapering, and bronchial wall thickening **PFTs can reveal obstructive pattern but are non-specific

fetus w/ ascites (eg, echolucent abdominal fluid), skin edema, polyhydramnios (single deepest pocket > or = 8), and placental thickening (reflecting intravillous edema)

hydrops fetalis - could be due to Rh alloimmunization or parvovirus B19 - in fetus w/hydrops fetalis and no associated dysmorphic features (eg, normal parietal diameter and femur length) , most likely cause is alpha-thalassemia major

18 yo black man w/ excessive urination. He voids every few hrs during the day and wakes 2-3 times per night to urinate despite restricting evening fluid intake. Urine appears clear w/no visible blood. He takes antihistamine for allergies. He's sexually active and doesn't use condoms. His mom has sickle cell disease and died from a stroke at age 32. Hematocrit is 41% and sodium is 138. Urine specific gravity is 1.001 (norm: 1.010-1.030) w/ normal pH and not blood, nitrites, protein, or glucose. Most likely cause of polyuria?

hyposthenuria (inability of kidneys to concentrate urine) - likely that patient has sickle cell trait and pts commonly have hyposthenuria - in response to hypoxic, hyperosmolar conditions of renal medulla, RBCs sickle in the vasa recta, impairing free water reabsorption and countercurrent exchange - pts typically have polyuria w normal serum sodium - typically doesn't require tx (RBC transfusion if severe) **Central DI causes polyuria and hyposthenuria due to insufficient ADH production. However, thirst mechanism is impaired and serum sodium is elevated

Infant w/ weight in 5th percentile, height in 5th percentile, and head circumference in 10th percentile is at risk for having (or developing) which complications?

hypoxia, polycythemia, hypoglycemia, hypothermia, and hypocalcemia (SGA infant) Hypoglycemia is due decreased glycogen stores

35 yo g3p0 comes in for recurrent preg loss. She has regular, monthly menses w/ 4-5 days of moderate bleeding and cramping. She has no chronic medical conditions. On pelvic exam, uterus is small, anteverted, and nontender. There are no adnexal masses. UPT is negative. Transvaginal US reveals a 2-cm submucosal fibroid at the uterine fundus. Thrombophilia workup is neg. Most appropriate tx for recurrent preg loss?

hysteroscopic myomectomy - pregnancy loss is likely due to submucosal fibroid, a benign smooth muscle (myometrial) tumor that can cause abnormal intrauterine anatomy by extending into the cavity or impinging on (ie, disrupting) the endometrium. this compromises embryo implantation or growth, likely due to decreased endometrial thickness and reduced vascularity of the fibroid (relative to the myometrium) **vaginal progesterone is used in pts w/ an incidental shortened cervix (< 25mm) on US to decrease risk of preterm delivery. it doesn't reduce risk of recurrent preg loss

68 yo man w/ worsening fatigue and exertional dyspnea. He was diagnosed w/ early-stage leukemia a yr ago but hasn't required tx (no significant sx). PE shows generalized lymphadenopathy and hepatosplenomegaly. Hgb is 7.1, reticulocytes 6%, plt 210,000, WBC 44,800 (80% lymphs). Most likely cause for anemia?

immune-mediated hemolysis (warm agglutinin disease) - high reticulocyte count suggests that there are adequate levels of iron, folate, and B12 to generate new erythrocytes and that the bone marrow is responding appropriately to anemia (bone marrow infiltration w/leukemic cells is unlikely cause of anemia) Patients w/ chronic lymphocytic leukemia often have significant immune dysregulation, which triggers formation of IgG autoantibodies against erythrocyte membranes (warm agglutinins) - RBCs coated in IgG are subsequently identified by the Fc receptor on splenic macrophages and partially or wholly phagocytized, leading to extravascular, immune-mediated hemolysis (ie, autoimmune hemolytic anemia)

45 yo man w/ several wks of easy fatigability and exertional dyspnea. Patient had subtotal gastrectomy for nonhealing gastric ulcer 5 yrs ago. He takes daily iron. PE shows shiny tongue and pale palmar creases. Labs Hgb 7.8, WBC 3800, Bili 2.3, Dbili 0.4, LDH 190. Patient's condition involves what pathophysiologic mechanism?

impaired DNA synthesis - loss of intrinsic factor leading to B12 deficiency - B12 is required cofactor in formation of thymidylate and purine molecules for DNA synthesis - increased intramedullary hemolysis of megaloblasts leads to direct hyperbilirubinemia and increased LDH

34 yo brought to ED due to several hrs of confusion after fever, malaise and cough for past 2 days. A year ago, he required prolonged hospitalization and extensive surgery for multiple gunshot wounds to the abdomen. Temp is 104.5, BP 80/50, pulse 110, and resp 32. Dullness to percussion and crackles over the left chest are present. Has normal 1st and 2nd heart sounds and bounding peripheral pulses. IV fluids and broad spectrum antibiotics are initiated. Next day, blood culture shows gram-pos cocci. Most likely underlying mechanism leading to clinical presentation?

impaired antibody-facilitated phagocytosis - likely has a splenectomy during operation for multiple gun shot wounds - high fever, hypotension, tachypnea, and tachycardia in the setting of bacteremia w/ gram-positive cocci, suggesting overwhelming Strep pneumoniae infection - encapsulated organisms are largely eliminated via the humoral immune response w/ antibody-mediated phagocytosis (opsoninization) and antibody-mediated complement activation in spleen

4 mo boy had generalized tonic-clonic seizure 30 mins ago. Since birth he has had 3 episodes of URI and otitis media. His parents and siblings have no PMH. He is lethargic and is under 5th percentile in weight. Anterior fontanelles is flat, pupils are reactive w/ normal fundi. He has round cheeks and doll-like face. Abdomen is protuberent and liver is palpable 4cm below the right costal margin. Extremities are thin. sodium 136, bicarb 16, glucose 38 lactic acid 24 (norm: 6-16). Urine is pos for ketones and serum triglycerides and uric acid levels are high. Likely cause?

impaired glycogen to glucose conversion (glucose-6-phosphatase deficiency (type 1 glycogen storage disease, von Gierke disease) deficiency of g6p in liver, kidneys and intestinal mucosa leads to impaired conversion of glycogen to glucose, leading to glycogen accumulation in the affected organs pts typically present at 3-4 mos w/ hypoglycemia (seizures) and lactic acidosis (due to buildup in the liver) - hyperuricemia and hyperlipidemia also seen - doll-like face, thin extremities, and hepatomegaly

45 yo w/ fatigue, weakness and diffuse bone pain w/ hx of celiac sprue and diet noncompliance. Labs: Hgb 12.2, Ca2+ 9, phosphate 2.6, PTH 122, ALP 234. What is responsible for condition?

impaired osteoid matrix mineralization (osteomalacia) - celiac sprue is common cause - typically due to severe vitamin D deficiency, which leads to decreased intestinal calcium and phosphorus absorption w/resultant secondary hyperparathyroidism - typical lab findings are hypophosphatemia, hypocalcemia, and elevated ALP

34 yo has been experiencing irregular periods for a year and now they have stopped completely. She has no headaches, visual changes, galactorrhea, hair loss, or fatigue. Has hx of non-Hodgkin lymphoma and received chemo 5 yrs ago (no recurrence). PE reveals nonenlarged thyroid w/out masses. Pelvic exam reveals dry vaginal mucosa and small, anteverted, mobile uterus w/ no adnexal masses. UPT neg. Likely lab findings?

increased FSH, increased LH, normal prolactin, normal TSH pt w/ cancer, amenorrhea, and signs of estrogen deficiency (eg, vaginal dryness) is consistent w/ ovarian failure secondary to chemo chemo and radiation target rapidly dividing cancer cells, but also affect proliferating granulosa and theca cells of the ovary ovarian failure results in decreased estrogen, thereby decreasing negative feedback to hypothalamus and pituitary resulting in increased FSH and LH

42 yo man comes to ED w/worsening anal pain in for the past 2 days. Was only hurting during defecation, but now is constant and sever pain. He also has anal pruritis, but no drainage. He has had anoreceptive sex in the past. He's febrile and has an erythematous and tender 1-cm fluctuant mass near the anal orifice with induration of the overlaying skin. What is the most likely cause of his current condition?

infection of an occluded anal crypt gland (perianal abscess) - indurated, erythematous mass near anal orifice associated with sever, constant anal pain and low-grade fever **HPV warts rarely cause pain **thrombosis of external hemorrhoids can cause extreme pain and tenderness. however, association with fluctuant mass or fever is unusual

A 95-year-old female has been in the ICU for 10 days with ventilator-dependent respiratory failure following a community acquired pneumonia. She has congestive heart failure with an ejection fraction of 15%. An abdominal sonogram was obtained during the workup for a fever of unknown origin and is consistent with cholecystomegaly, anterior wall thickening, and pericholecystic fluid. Which of the following is the most appropriate management?

insertion of percutaneous cholecystectomy tube - percutaneous insertion of cholecystectomy tube for decompression is a safe way to manage acute cholecystitis in a pt who is otherwise a poor surgical candidate due to medical comorbities

58 yo patient tx w/chemotherapy for Burkitt lymphoma w/ PMH of HTN, stable angina, and hyperlipidemia has nausea and generalized weakness. K+ is 6.8 and peaked T waves are seen on EKG. What intervention should be employed to rapidly lower serum potassium?

insulin and glucose (insulin moves potassium into cell in minutes and glucose is given to avoid hypoglycemia) **Inhaled beta agonists are also rapidly acting agents that shift potassium intracellularly. However, their use in patients w/ active coronary disease (stable angina) can cause tachycardia and precipitate angina

45 yo woman w/ 3mo hx of progressive exertional dyspnea and nonproductive cough. She also has intermittent episodes of difficulty swallowing and hx of HTN, GERD, and raynaud phenomenon. Skin over face, hands, and arms is thick and firm. Scattered crackles are heard on lung auscultation. Most likely cause of her current symptoms?

interstitial lung disease (Systemic sclerosis) **restriction of chest movement affecting respiration is most commonly seen in patient with obesity hypoventilation syndrome

64 yo w/ 2 days of fever, chills, productive cough, and left-sided pleuritic chest pain. Temp is 102, BP is 110/60, pulse is 100, and resp 22. O2 sat is 95% on right side, but drops to 89% when he lies on left. Dullness to percussion and bronchial breath sounds are present on the left side. What pathophysiologic mechanism in the left lung is most responsible for the drop in oxygen saturation after the change in position?

intrapulmonary shunting (pneumonia) alveolar consolidation in pneumonia causes hypoxemia due to right-to-left intrapulmonary shunting positional changes that make the consolidation more gravity dependent worse V/Q mismatch, increase intrapulmonary shunting, and lead to worsened hypoxemia

A 20-year-old male presents to the emergency department with colicky abdominal pain that comes and goes every 30-60 minutes. History reveals that his mother has Peutz-Jeghers syndrome. Physical examination reveals multiple small macular spots around the mouth and brown pigmentation of the buccal mucosa. His heart has a regular rate and rhythm, the chest is clear to auscultation bilaterally, and the abdomen is moderately tender in the right lower quadrant with associated rebound. The most likely diagnosis is:

intussusception (about 50% of patients w/ Peutz-Jeghers syndrome will have intussusception during their lifetime and it is most common in the small bowel, often at ileocecal jxn)

30 yo w/ low back pain and stiffness for 2mos that improves w/activity. He has also had intermittent diarrhea for last 3 mos. No travel outside of US and has has one sexual partner over last 2 yrs. No other medical conditions. PE shows limited spine flexion and tenderness in lower back. Has anemia and thrombocytosis. Stool cultures are negative. Plain radiograph reveal SI joint inflammation. What is most likely cause of symptoms?

irritable bowel disease complicated by spondyloarthritis **reactive arthritis may follow infectious diarrhea but typically occurs w/in 2-3 wks of onset of diarrhea and usually also causes urethritis, conjunctivitis/uveitis, malaise, and cutaneous findings

75 yo undergoes surgical repair for AAA. Postoperatively, he develops left lower quadrant abdominal pain followed by bloody diarrhea. PMH of DM2, HTN, and hypercholesterolemia. He eats a low-fiber diet and recently quit smoking. He has low grade fever and blood is visible on rectal exam. CT shows thickening of colon at rectosigmoid jxn. On colonoscopy, ulcerations are seen in same area, but colon above and below the lesions is completely normal. Dx?

ischemic colitis - abdominal pain and blood diarrhea following vascular procedure - contributing factors from AAA repair include loss of collateral circulation, manipulation of vessels w/surgical instruments, prolonged aortic clamping, and impaired blood flow through inferior mesenteric artery

75 yo man is found unresponsive by son. Has hx of CAD, DM, HTN, and stroke. Was hospitalized 2mos ago for pneumonia and received IV antibiotics and transfusion of 4 units packed RBCs. Temp is 102.4, BP 70/40, pulse 120, resp 32, and O2 sat 79%. Elevated WBC and has new right upper lobe infiltrate. Given normal saline, antibiotics, vasopressors, and mechanical ventilation. Next day Hgb 9.5, platelet 120,000, AST 3720 ALT 3250 ALP 162. What is the most likely cause of abnormal liver function panel?

ischemic hepatic injury from septic shock due to pneumonia - in patients who survive the underlying cause of their hypotension, liver enzymes typically return to normal within a few weeks **acute viral hepatitis can lead to large AST and ALT elevations, but is typically accompanied by significant hyperbilirubinemia, nausea, and vomiting

4 mo boy has noisy breathing. His mom first noticed a squeaky noise a month ago when he was crying or lying in his crib. The noise is progressively louder and persistent. He has no episodes of difficulty breathing or bluish discoloration during feeding, but spits up small amounts after feeds. When placed supine on exam table, inspiratory stridor is heard, which resolves quickly when he's in the prone position. What is the likely diagnosis?

laryngomalacia - most common cause of stridor in infants - supraglottic tissues appear floppy and collapse on inspiration to partially block airway - crying and feeding worsen the stridor due to increased airflow - prone positioning improves stridor because the tongue moves anteriorly, partially relieving the obstruction - usually resolves spontaneously by 18mos **choanal atresia, or congenital blockage of the posterior nasal aperture, presents shortly after birth with cyclic cyanosis that worsents during feeding and improves with crying (mouth breathing) - unilateral choanal atresia commonly presents during childhood as persistent, unilateral nasal obstruction and discharge

56 yo w/ severe dizziness, inability to walk, and stabbing pain on R side of face. BP is 144/90 and pulse is 92. He topples to the right when sitting w/out support. Left pupil is larger than right, and there is decreased corneal reflex on the right directly but not consensually. Partial ptosis of right eye. Horizontal and rotational nystagmus present. Voice is hoarse. Gag reflex is diminished. Loss of pain and temp sensation in R face and left trunk and limbs. Most likely location of brain lesion?

lateral medulla (Wallenberg syndrome) - most likely incracranial basilar a vestibular findings almost always present and include nystagmus, vertigo, and falling to side of lesion sensory findings include loss of pain an temp in ipsilateral face (spinal trigeminal nucleus and tract) and contralateral trunk/limbs (spinothalamic tract) despite loss of sensation, pain in face is sometimes prominent, also likely du to spinal trigeminal nucleus and tract lesions ipsilateral bulbar muscle weakness (eg, dysphagia, dysarthria, hoarseness) due to involvement of nucleus ambiguus and autonomic dysfxn (eg, ipsilateral horner syndrome) corneal reflex may be diminished due to disruption of the spinal trigeminal nucleus (located w/in lateral medulla)

32 yo woman w/ pelvic pain for past 2 days after menstrual period ended. She says pain feels like labor contractions. She is sexually active and doesn't use contraception. 5 yr ago has a spontaneous abortion tx by suction D&C. Pap was norm 2yrs ago. BP is 140/90 and pulse is 113. Pelvic exam shows irregularly enlarged uterus. Cervix is dilated w/ a spherical mass visible thru external os. Mass is firm and smooth, and slight bleeding is noted around it. Dx?

leiomyoma uteri submucosal fibroids arise from the myometrium immediately under the endometrial lining and protrude into the uterine cavity. they commonly cause heavy and prolonged menstrual bleeding and can prolapse thru the cervical os, presenting w/ a labor-like pain due to cervical distension by the solid mass

Patient is found to have calcium oxalate stones. Best recommendation for prevention of future stones in this pt?

limited sodium intake increased sodium intake enhances calcium excretion (hypercalciuria) and low sodium promotes sodium and calcium reabsorption through its effect on the medullary concentration gradient **increased vitamin c intake promotes hyperoxaluria, which can increase incidence of stones

52 yo has 6 mos of intermittent, epigastric pain that radiates to the back, is exacerbated w/eating and assoc w/nausea. Smoked cigs for 20 yrs and drinks 7-8 beers/day. Labs: albumin 3.7 tbili: 0.3 ast 32 alt 24 lipase 32 (normal: 10-140). CT shows pancreatic atrophy along w/ multiple calcifications of pancreatic parenchyma. MRCP reveals nondilated pancreatic duct w/out intraductal stones. Next best step in management?

lipase supplementation (chronic pancreatitis) - helps alleviate pain by reducing pancreatic hyperstimulation and improves nutrient digestion; additionally fat malabsorption and steatorrhea (if present) typically improve **a celiac plexus block (w/glucocorticoids or alcohol) is useful for pancreatic cancer pain but has limited efficacy in chronic pancreatitis - increased risk of hypotension and infection

woman at 30wks gestation w/ severe epigastric pain radiating to back w/elevated lipase (>3x normal), who doesn't drink alcohol or do drugs and has hx of hypertriglyceridemia. RUQ US reveals no biliar sludge or dilation of the biliary tree. Next step in management?

lipid panel - need lipid panel showing triglyceride level > 1000mg/dL to diagnose triglyceride-induced pancreatitis - in addition to IV fluids and pain control, may need insulin (limits fatty-acid release from adipocytes) and apheresis (removes triglyceride-rich plasma)

9 yo boy w/ dark brown urine after URI 3 days ago that has resolved. He also has bilateral hearing loss and an uncle who died of complications of a renal transplant 5yrs ago. What is most likely histological finding on renal biopsy?

longitudinal splitting of the glomerular basement membrane (Alport syndrome, mutation against type IV collagen) labs would show normal complement levels **mesangial and glomerular capillary deposition of C3 and IgG is seen in poststrep glomerulonephritis

27 yo g1p0 comes to office for first prenatal visit. She has not had any vaginal bleeding or pelvic pain and has no PMH. Abdominal exam reveals gravid uterus that is nontender, and the fundus is just below the umbilicus. US shows 2 intrauterine fetal poles measuring 14 wks gestation w/ HRs of 150 and 155. Cervical length is 4.5cm. Next step in management?

low-dose aspirin (preeclampsia ppx) pts w/ DM, chronic HTN, and multiple gestations are at high risk for preeclampsia and should be given daily low-dose aspirin to decrease risk **IM hydroxyprogesterone is indicated in pts w/ prior spontaneous preterm delivery to decrease the risk of recurrence

1 day old girl is in newborn nursery due to swollen hands and feet. Born via emergency c-section because mom has preeclampsia w/severe features complicated by pulmonary edema. Vitals normal, has webbed neck, dysplastic nails, and bilateral, nonpitting carpal and pedal edema. US shows horseshoe kidney. Most likely cause of edema?

lymphatic network dysgenesis (congenital lymphedema) - congenital lymphedema occurs in over half of patients with turner syndrome - dysfunctional lymphatic system causes accumulation of protein-rich interstitial fluid in hands, feet, and neck severe obstruction of lymphatic vessels can result in cystic hygroma of the neck and fetal hydrops

36 yo discovered 1cm firm, round mass in the upper outer quadrant of breast. No axillary lymph nodes are palpable. What is the next step in managment?

mammography (first-line imaging for women greater than 30) **all palpable breast masses in adult women require further evaluation

82 yo brought to ED for severe abdominal pain with cramping, bloating, and vomiting for past 5 days. She has PMH of DM2, HTN, gallstones, and diverticulitis treated medically 2 yrs ago without recurrence. She is hypotensive and tachycardic with WBC 11,000 and there is mild elevation of liber transaminases. Abdomen is soft, but distended, with hyperactive bowel sounds. Abdominal x-ray shows dilated loops of small bowel and air in the intrahepatic bile ducts. What is the most likely cause of her symptoms

mechanical bowel obstruction (gallstone ileus) - common symptoms: nausea and vomiting, pneumobilia (air in the biliary tree), hyperactive bowel sounds , and dilated loops of bowel - occurs when gallstone passes through biliary-enteric fistula in small bowel **emphysematous cholecystitis is more commonly characterized by fever and RUQ pain and can also cause ileus with decreased or absent bowel sounds (this person has hyperactive bowel sounds)

25 yo female has intermittent double vision, dizziness, and unsteady gait for last several days. During the past yr, she has had several episodes of numbness and dizziness. BP is 130/70 and pulse is 76. On attempted left gaze, her left eye abducts and exhibits horizontal nystagmus, but her right eye remains stationary. When she attempts to look to the right, her right eye abducts and exhibits horizontal nystagmus, but her left eye remains stationary. Pt can converge both eyes without any associated nystagmus. Most likely location of lesion?

medial longitudinal fasciculus (internuclear ophthalmoplegia) patient likely has MS and her pattern of eye movement is consistent with internuclear ophthalmoplegia, a disorder of the conjugate horizontal gaze in which the affected eye (ipsilateral to the lesion) is unable to adduct and the contralateral eye abducts with nystagmus (convergence and pupillary light reflex are preserved) internuclear ophthalmoplegia results from damage to heavily myelinated fibers of the medial longitudinal fasciculus (MLF) - MLF is a paired neural tract that mediates communication between CN III and CN VI, allowing for coordinated eye movements - unilateral MLF lesions occur with lacunar stroke in the pontine artery distribution; however, bilateral lesions are classically seen in MS

30 yo woman w/ left knee pain after colliding with another player during a hockey game. There is small left knee effusion and tenderness over the medial aspect of the knee at the joint line, but gait is normal. With the patient standing on the left leg and the knee bent slightly, internal rotation of the knee elicits a click with significant, sharp pain. Xray reveals no abnormalities

medial meniscus tear

72 yo was mechanically ventilated and tx w/ IV antibiotics, IV fluids, and vasopressor therapy for respiratory failure and septic shock. Hemodynamic monitoring was performed using right internal jugular venous catheter and a right brachial artery catheter. today he was extubated, and catheters were removed. since then, he reports right hand discomfort w/ tingling and numbness. both hands are warm w/ 2-sec capillary refill. b/l grip strength is diminished but symmetric, and sensation to light touch and pain is diminished in lateral palm, thumb, and index finger of the R hand. Likely cause?

median nerve injury - recent hx of brachial artery cannulation and adjacency of the brachial a to median n in distal upper arm w/ diminished sensation in distribution of median n

31 yo woman w/ 6wk hx of periorbital edema and abdominal distension. Presents w/moderate ascites and lower extremity edema. Urinalysis shows proteinuria and 24-hr protein excretion is 4g/day, total serum protein is 5 and serum albumin is 2.5. Renal US unremarkable and biopsy is performed. Pt starts to improve on low protein diet and diuretic tx but then suddenly develops right-sided abdominal pain, fever and gross hematuria. What is most likely diagnosis on renal biopsy?

membranous glomerulopathy (renal vein thrombosis (RVT) due to loss of antithrombin III in the urine) RVT can occur in any etiology of nephrotic syndrome, but is most commonly seen with membranous glomerulopathy

previously healthy 5 yo brought to ED with lethargy and altered mental status. 5 days ago was treated for a URI with aspirin. Now has nausea, vomiting, and hepatomegaly. What is percutaneous liver biopsy likely to reveal?

microvesicular fatty infiltration (Reye's syndrome) (macrovesivular fatty changes are seen in alcoholics (alcoholic hepatitis) and obese patients (fatty liver disease))

patient with 2 wk hx of low-grade fever and progressive weakness w/hx of heart murmur. exam shows splinter hemorrhages, small petechiae on the palatal mucosa, and an audible murmur. What is the most likely valvular dysfunction?

mitral regurgitation mitral valve disease, usually mitral valve prolapse with coexisting mitral regurgitation, is the most common valvular abnormality detected in patients with infective endocarditis

23 yo female w/ pain in right forefoot that started 6wks ago and has worsened over pas wk, making it hard to do regular exercise. there was not trauma or inciting event. PE of right foot elicits a clicking sensation when the 3rd and 4th metatarsal heads are squeezed together; this also produces burning pain over plantar surface of foot. Likely diagnosis?

morton neuroma - mechanically induced neuropathic degeneration of interdigital nerves - frequently seen in runners and is characterized by numbness, aching, and burning in distal forefoot from metarsal heads to 3rd and 4th toes **tarsal tunnel syndrome pain is reproduced by percussion over nerve (tinel sign), not by metatarsal compression

18mo boy developed fever, achiness, and fatigue yesterday and today has vomited 3x but has had no cough, rhinorrhea, or diarrhea. He did not receive routine 12- to 15-mo vaccines. Temp is 102.9. Pt is alert but irritable and closes his eyes during fundoscopic exam. When examiner flexes the neck, the child flexes knees. CSF: glucose: 60, protein: 75, WBC 150 RBC 2. Most likely cause of illness?

mumps virus CSF shows elevated WBC count (pleocytosis), elevated protein, and normal glucose, which is highly suggestive of viral meningitis **bacterial meningitis presents w/ marked pleocytosis, low glucose, and high protein

60 yo man w/ fatigue and muscle weakness in his extremities. Has also lost weight. Smokes 2 packs/day. Muscle strength is 3/5 in proximal muscle groups symmetrically. Reflexes are 2+ bilat and has no sensory abnormalities. There are erythematous violaceous papules involving dorsum of his fingers. Chest x-ray reveals ill-defined mass in right lower lobe. His muscle weakness is most likely due to lesion involving what?

muscle fibers (paraneoplastic dermatomyosisitis) - symmetrical proximal muscle weakness and erythematous rash on dorsum of fingers (Gottrons sign) **myasthenia gravis is due to autoantibodies against acetycholine receptors in postsynaptic membrane - absence of eye findings and presence of skin findings makes this unlikely **lambert-eaton syndrome is due to antibodies against presynaptic membrane voltage-gated ca2+ channels - would have diminished or absent DTRs and no skin findings

62 yo man who undergoes partial pancreactomy for exocrine pancreatic cancer. Surgery is complicated by excessive bleeding and he is given packed RBC transfusion and normal saline. He is transferred to surgical ICU and 12 hrs later he has decreased O2 sat. He has been given morphine regularly for pain. Temp is 99.5, BP 80/40, pulse 112, resp 28, puls ox 84% on 4L. He has bilateral basal crackles and abdomen is tender w/decreased bowel sounds. Pulmonary artery catheter shows cardiac index (CI) of 2 (normal 2.8-4.2) and a PCWP of 20 (normally 6-15). Most likely cause?

myocardial infarction leading to cardiogenic shock perioperative MI is common in pts undergoing noncardiac surgery; intraoperative hemorrhage requiring transfusion increases risk (likely due to reduced oxygen delivery to myocardium) patients lack chest pain due to postop pain control (morphine) CI is a measure of cardiac output (stroke volume x HR) and is low in LV infarction as tachycardia can't make up for decrease in stroke volume The increased pressure in the LV is transmitted back to left atrium and the lungs; therefore, PCWP is increased **TRALI refers to development of fever, hypotension, and noncardiogenic pulmonary edema w/in 6hrs of blood product admin. Because LV fxn is typically normal, there will be normal or high CI and normal or low PCWP

25 yo who was bit on upper right thigh and now has small ulcer. The pain is now increasing. PE shows erythematous and painful ulcer and remaining exam is unremarkable. What complication is she most likely to develop?

necrosis and eschar - small ulcer developing at the site of recent bite is highly suggestive of brown recluse bite **unlike brown recluse bites, black widow bites lead to more pronounced local and systemic manifestations due to effects of the toxin, like muscle pain, abdominal rigidity, and muscle cramps - wound ulceration is uncommon

2 yo girl w/ irritability, intermittent fevers, and poor appetite for 2 wks. The family recently went camping, but hasn't travelled internationally. Pt is tired-appearing and pale. The right upper eyelid is drooping and the right pupil is constricted. When the patient cries, only the left side of the face appears flushed. Abdomen is soft, nontender, and nondistended with no palpable masses. She has no dysmetria and has full strength in extremities. MRI reveals a cervical paravertebral mass. What is the most likely diagnosis?

neuroblastoma (horner syndrome w/ cervical paravertebral mass in a 2yo) **HSV and lyme disease can affect the facial nerve and cause Bell palsy, resulting in unilateral facial droop. Pupillary response and sweating/flushing aren't affected in Bell palsy.

32 yo man hospitalized for MDD is started on Sertraline and given Olanzapine on day 3 due to worsening agitation. After this, he is noticeably calmer but is unable to get out of bed. He appears confused and doesn't respond to questions appropriately. He is diaphoretic, temp 104.1, BP 167/97, and pulse 112. Arms and legs are stiff and difficult to bend. Most likely cause of symptoms?

neuroleptic malignant syndrome from onlanzapine differentiated from serotonin syndrome, which is characterized by hyperthermia, autonomic instability, mental status changes, prominent GI symptoms, neuromuscular irritability (including hyperreflexia and myoclonus), but not lead pipe rigiditiy **malignant hyperthermia is a specific disorder that occurs w/use of volatile anesthetics or succinylcholine

19 yo female w/ abnormal vaginal discharge for 2 wks. Had an episode of postcoital vaginal bleeding a few days ago, followed by return of abnormal yellow discharge. She is sexually active and has had the same partner for 6 mos, using no protection. PE shows yellow cervical discharge. Cervix is friable and bleeds easily on cotton tip manipulation. UPT -. What is likely to be found on microscopic examination of the discharge?

no organisms (acute cervicitis) acute cervicitis is most commonly caused by chlamydia trachomatis and neisseria gonorrhoeae and classical findings include mucopurulent cervical discharge and an edematous, friable cervix that bleeds w/manipulation no organisms are visualized w/ light microscopy since c trachomatis is an obligate intracellular pathogen

36 yo has postop incisional pain a week after abdominal hysterectomy. Since leaving the hospital she has incisional pain and light vaginal bleeding. She also had pulling sensation to the left of her incision that worsens when she bend over or strains to have a bowel movement. Vitals are normal. Abdomen is soft w/normoactive bowel sounds. The low, horizontal abdominal incision is closed with staples, and the skin edges are intact. There is mild incisional induration but no erythema, drainage, or fluctuance. Incision is diffusely tender to deep palpation, and no palpable masses or defects are present. Best next step in management?

observation and reassurance - afebrile and normal WBCs w/mild induration and diffuse tenderness are all normal **incisional exploration and packing is for superficial wound dehiscence (ie, skin separation with intact fascia) or subq fluid collection (eg, seroma, hematoma)

An 86-year-old female presents to the emergency department with a 2-day history of nausea, vomiting, and abdominal pain. Physical examination reveals a softly distended abdomen with minimal tenderness without evidence of masses. You note inhalation somatic dysfunction of the right pelvic diaphragm. Range of motion testing exhibits pain along the medial thigh with right hip internal rotation and extension. Which of the following is the most likely cause of her symptoms?

obturator hernia - positive Howship-Romberg sign w/ medial thigh pain elicited by internal rotation and extension of the hip is suggestive of obturator hernia **femoral hernia would present w/ a bulge beneath inguinal ligament on the affected side

23 yo man has lower extremity fxs, abdominal bruising, and scalp lacerations after MVA. He complains of pain and SOB. Initially, his BP is 95/60 and pulse is 120 and reg. Spine is immobilized and peripheral IV access is obtained. He is given 1L IV fluids. On the way to hospital, he becomes progressively drowsy and develops progressive weakness on R side of body. BP is 160/90 and pulse is 50. What cranial nerve function is most likely to be lost?

oculomotor (transtentorial (uncal) herniation secondary to right-sided epidural hematoma) epidural hematomas result from rupture of middle meningeal artery, and this high arterial pressure can rapidly expand the hematoma to compress the temporal lobe the fluid resuscitation in this patient likely increased the rate at which the hematoma expanded and the presence of HTN, bradycardia, and resp depression (Cushing's reflex) indicated elevated ICP The uncus is the innnermost part of the temporal lobe and herniates through the tentorium to cause pressure on ipsilateral oculomotor nerve (mydriasis and eventually down and out gaze), ipsilateral posterior cerebral artery (contralateral homonymous heminopsia), and contralateral cerebral peduncle (ipsilateral hemiparesis)

29 yo man w/ progressive lower extremity weakness. Had trigeminal neuralgia 3 mo ago and self-limited URI 2 wks ago. He has increased resistance to passive flexion and extension of lower limbs. DTR are 3+ bilat and plantar reflexes are upgoing b/l. There is decreased vibratory and positional sensation in his left upper extremity but no other sensory loss. Most likely finding on CSF analysis of this pt?

oligoclonal bands (Multiple sclerosis) - neurologic deficits disseminated in space and time (eg, trigeminal neuralgia, spastic lower limb paralysis, left upper limb sensory loss) **albuminocystologic dissociation is seen in Guiilain-Barre, which presents w/ rapidly progressive ascending paralysis. associated symptoms often include reduced/absent reflexes and paresthesias

78 yo has 6 mos of progressive bilat buttock pain that radiates to thighs and calfs. Worse w/ ambulation and now can only walk a few blocks. Better with leaning on cane and sitting. He has no pain at night and no bowel or bladder dysfunction. Had CABG 6 yrs ago and has DM2 and HTN. PE shows normal strength, reflexes, and sensation in the legs. Straight leg test doesn't reproduce pain. ABI 1.1 on right and 1.2 on left. HBA1C is 8%. Most likely dx?

osteoarthritis of the spine (lumbar spinal stenosis) - narrowing of spinal canal results in compression lumbar nerve roots - pain is relieved w/flexion (shopping cart sign) **vertebral compression fractures typically present w/ acute back pain following a minor injury (eg, ground level fall) and pain usually diminishes w/time

54 yo female fell on her hand suddenly while walking her dog and landed forcefully on her palm. PE reveals left wrist tenderness at the dorsoradial wrist lateral to the tendon of the extensor pollicis longus. Radiograph shows radiolucent line across the waist of the scaphoid bone. She should be closely monitored for what complication?

osteonecrosis (scaphoid fracture) - displaced fractures should be considered for surgical intervention **lunate dislocation following fall on outstretched hand can cause compressive neuropathy of the median nerve. nerve injury isn't common following scaphoid injury

21 yo requesting refill of opioid med for new-onset left hip pain. Pain started 3 days ago, but has progressively worsened and is now present at rest and overnight. No hx of trauma but has sickle cell disease. He is sexually active with a new partner. Temp is 99, bp is 100/70 and pulse is 80. PE shows no local tenderness, but restriction w/ abduction and internal rotation of left hip. X-ray and ESR are normal. Most likely diagnosis?

osteonecrosis of the proximal femur (avascular necrosis) - hip pain, limited internal rotation and abduction, unremarkable x-rays, and normal inflammatory markers - caused by occlusion of end arteries supplying the femoral head, leading to necrosis and collapse of periarticular bone and cartilage

28 yo female being evaluated for infertility comes to office w/ clear vaginal discharge for past 2 days. Menstrual cycles occur every 29 days with 4 days of flow. Last week she took penicillin for a sore throat. Pelvic exam shows clear mucus at the cervical os. What is the likely cause of pt's discharge?

ovulation - cervical mucus secretion close to ovulation (late follicular phase) increases in quantity to facilitate sperm transport into the uterus for conception ovulation marks transition from follicular phase to luteal phase midway through cycle and is preceded by LH surge **cervical mucus plug serves as barrier to ascending infection during pregnancy. this brown, red, or yellowish thick mucus is typically shed before or during labor

A 5-year-old male presents with penile swelling and discomfort. History reveals that he went to the restroom to urinate, retracted his foreskin, and now his glans is stuck outside of his foreskin. Proximal to the stricture the penis is flaccid, but distal to the stricture the glans is becoming swollen. The most likely diagnosis is what?

paraphimosis (paramedics!) a urological emergency that occurs when the retracted foreskin develops a fixed constriction proximal to the glans the penis distal to the constricting foreskin can become engorged in blood and edematous, which can lead to urinary retention, arterial compromise, and possible gangrene **Phimosis is the inability to retract the foreskin proximally to the glans penis. Phimosis can be a results of normal development or may be due to infection or poor hygiene. As most cases are not a result of a pathologic condition but a due to normal growth and development, reassurance is usually all that is needed, unless it leads to complications such as urinary retention. In these cases, the traditional form of treatment has been a circumcision.

24 yo w/ hx of heavy menstrual periods for which she had an IUD placed 2 yrs ago became amenorrheic, but for past mo has had light vaginal spotting with wiping. She also has intermittent pelvic pain that has now become constant and is unrelieved w/ibuprofen. She has had 3 sexual partners in the last yr. UPT is negative. Most likely cause of symptoms?

pelvic inflammatory disease - change in bleeding pattern in a sexually active woman w/ pelvic pain - patients can initially have intermittent pain; however, as infection and inflammation spread throughout the peritoneal cavity, constant pelvic pain develops - tx w/ broad-spectrum antibiotic therapy (ie, 2nd-gen cephalosporin plus doxycycline) **endometriosis can cause pain, but because it is an extrauterine disease, abnormal vaginal bleeding isn't present

54 yo woman g3p3 has difficulty voiding over past few wks. When she voids, she has difficulty initiating stream completely, but has no dysuria or hematuria. She has no involuntary leakage of urine. She has pelvic pressure that is worse w/standing. 10 mos ago, underwent total abdominal hysterectomy for epithelial ovarian cancer and completed course of chemo. Pelvic exam shows protruding soft, nontender mass at the level of the hymen that descends past the introitus w/ valsalva maneuver, which doesn't cause leakage of urine. Sensation and deep tendon reflexes are intact. What is the most likely cause of her symptoms?

pelvic organ prolapse - risk factors: multiparity, postmenopausal age, hysterectomy, and obesity - symptoms include pelvic pressure, voiding dysfunction (eg, urinary retention, difficulty starting stream, incontinence), and bowel dysfunction (constipation, fecal incontinence) **urethral diverticulum may present w/dysuria and postvoid dribbling, not urinary retention and pelvic pain. Pelvic exam would typically reveal a tender, anterior vaginal mass w/expression of urine or pus

22 yo w/ right-sided tension pneumothorax after tractor ran over chest has chest tube placed after BP and pulse ox normalize. Rest of trauma survey is notable for bruising over anterior chest wall and crackling of the neck and chest skin on palpation. Chest x-ray confirms appropriate endotracheal and chest tube placement and reveals multiple rib fractures, right pneumothorax, pneumomediastinum, and subcutaneous emphysema. 2 hrs later, chest tube drainage system has persistent large air leak, despite adequate seal at chest wall and connection points. What is best next step in management?

perform bronchoscopy - persistent air leakage after appropriate tx of tension pneumothorax suggest continuous, rapid, extrapulmonary air accumulation from tracheobronchial injury that allows (due to larger diameter of the proximal airway) a large quantity of air to escape w/each breath - bronchoscopy can directly visualized the lumen of the tracheobronchial tree and is definitive test for diagnosis - most patients require operative repair

63 yo w/ HTN collapses after sudden onset severe chest pain that radiated to back. Rushed to ED and BP is 90/50 and HR 110. Jugular veins are distended. The lungs are clear to auscultation. An intra-arterial catheter in radial artery shows variation in SBP related to respiratory cycle. X-ray shows widening of mediastinum. Most likely cause of syncope?

pericardial fluid accumulation (aortic dissection complicated by cardiac tamponade) - hypotension, tachycardia, distended jugular veins and respiratory variation in SBP (ie pulsus paradoxus)

34 yo man w/intermittent dizziness/spinning sensation, accompanied by nausea that resolve spontaneously after approx 1 min. Symptoms occur when he is lifting heavy weights, riding on an elevator, or after sneezing. He has no headache or ear pain but has trouble hearing out of right ear. He had a concussion 4mos ago. When performing a valsalva maneuver there is nystagmus and reoccurrence of his symptoms. what is the most likely cause?

perilymphatic fistula - rare, but debilitating complication of head injury - causes leakage of endolymph from semicircular canals and cochlea into surrounding tissues - sensorineural hearing loss is due to damage to cochlear hair cells from loss of endolymph - endolymph leakage due to pressure changes in ear causes episodic vertigo w/nystagmus

38 yo w/ 3 mos of episodes where her left foot tingles and becomes number and her entire foot becomes limp, causing her to trip. Symptoms resolve w/in a few hrs. She has been going through some stress and recently joined a meditation group, but has no other health issues. PE shows diminished pinprick sensation over the dorsum of the left foot and weakness of the left big toe on extension. She can walk on her left toes but not on her left heel. Most likely cause of her symptoms?

peripheral nerve compression - pt w/ intermittent foot drop likely has common fibular neuropathy common fibular n (also called common peroneal n) travels near the fibular head and is susceptible to compressive injuries from leg immobilization (eg, cast, bedrest), prolonged leg crossing (eg, during meditation), or protracted squatting manifestations are typically transient and reflect impaired functioning of the fibular n as follows: - unilateral foot drop - numbness/tingling over dorsal foot or lateral shin - impaired ankle dorsiflexion (walking on heels) and great toe extension - preserved plantar flexion (walking on toes) and reflexes **spinal nerve root entrapment can present w/ foot drop and impaired dorsiflexion when L5 is involved; however, patients usually have back pain radiating to foot. in addition, short, transient episodes over months would be uncommon

48 yo woman is having elective cholecystectomy. She is obese w/HTN. In prep for surgery, general anesthesia w/endotracheal intubation is administered. Immediately after a needle is placed into intraperitoneal space and CO2 gas insufflation is performed, severe sinus bradycardia and transient AV block is noted. What is the most likely cause of observed intraoperative finding?

peritoneal stretching - stretch receptors sense increase in intraabdominal pressure and respond by triggering increased vagal tone - close monitoring is required as patients may develop severe bradycardia, AV block, and sometimes asystole **CO2 gas embolization usually results from inadvertent insertion of insufflation needle directly into and artery, vein, or organ (eg, liver). affected patients typically develop end-organ infarction (arterial embolism) or hypotension and shock (venous embolism). severe bradycardia and AV block aren't expected

58 yo man w/ megaloblastic anemia, vitiligo, and shiny, smooth tongue, and subacute combined degeneration

pernicious anemia (autoimmune condition against intrinsic factor leading to B12 deficiency) **can present w/similar pancytopenia like in aplastic anemia but w/ macrocytosis and glossitis

34 yo g1p1 delivered baby 6 wks ago and delivery was complicated by severe postpartum bleeding requiring 5 units of RBCs. She now describes fatigue, poor appetite and is worried because she has little energy to bond w/ her child. She has been formula feeding baby due to lack of milk production. Prepreg weight was 140 and now weight is 124. BP is 90/60 and pulse is 88. Most likely cause?

pituitary ischemic necrosis (sheehan syndrome) - massive postpartum hemorrhage followed by lactation failure, hypotension and weight loss

34 yo g4p2a1 comes to ED at 24 wks gestation w/painless vaginal bleeding that began after intercourse. She woke up in a pool of bright red blood and continues to soak through a pad every hour. Hasn't had routine care but 8wk US showed singleton intrauterine pregnancy. Has had 2 CSs at term and a cervical conization earlier this year. Smokes a pack/day. Vitals normal. FHR 140 w/accelerations, absent decelerations, and moderate variability. Irregular contractions seen on tocodynamometry. Speculum exam shows large clots in vaginal vault, a closed cervix, and active bleeding form the os. Diagnsosis?

placentia previa (placenta covers cervix causing painless bleeding after 20 wks) - blood loss is primarily maternal, so initial fetal monitoring is typically reassuring **cervical insufficiency presents w/2nd trimester pregnancy loss w/out vaginal bleeding

A 19-year-old football player presents after twisting his right knee during a game. Physical examination reveals joint line tenderness and mild swelling in the area of injury. A clicking and popping sound is appreciated during passive flexion and extension of the right knee. The most appropriate initial step in the management is?

plain film radiography (need to rule out fx)

4 yo boy w/ bruising on chest, back, and legs, decreased energy, poor appetite, and oozing blood from gums last week. Paternal uncle had hemophilia. Exam shows scattered bruising, nontender, bilateral, anterior cervical lymphadenopathy, hepatosplenomegaly, with no joint swelling. Patient likely has a deficiency in what?

platelets (acute lymphoblastic leukemia) - cancer infiltrating bone marrow imparis platelet production leading to bruising, petechiae, and mucosal bleeding

51 yo man develops ARDS while hospitalized w/pancreatitis. On 3rd day in ICU, he is sedated, intubated, and ventilated w/ PEEP of 15cm water and fraction of inspired O2 of 0.6 (60%). His pulse acutely increases from 100 to 140, SBP goes from 120 to 90, and CVP increases from 10 to 15. On chest auscultation, breath sounds are absent on the left side. What is the most likely explanation for his sudden deterioration?

pneumothorax (PEEP in pt w/ARDS can rupture fragile lung parenchyma, resulting in air leakage into the pleural space) - causes absent breath sounds on affected side - can lead to compression of structures in mediastinum and impaired RV filling, leading to hypotension and tachycardia - when intrapleural space fills with air, intrathoracic pressure increases, leading to decreased venous compliance (cant stretch) leading to increased central venous pressure **if endotracheal tube shifts into right main-stem bronchi, atelectasis of left lung can result, leading to cough, dyspnea, and decreased O2 sat, but doesn't compress mediastinum (no hypotension and tachycardia)

51 yo w/ nausea and fatigue for past several weeks w/ occasional episodes of flank pain and nocturia 2-3 times per night. No change in appetite or weight. BP is 164/100. Liver is enlarged, and a mass is felt at the right flank on deep palpation. Prostate is mildly enlarged and nontender. Hgb is 10.4, BUN is 48 and creatinine is 3.1. What best explains these findings?

polycystic kidney disease - flank pain can frequently occur (cyst rupture or nephrolitiasis) - HTN from cyst enlargement leading to localized renal ischemia and increased secretion of renin - Nocturia due to urinary concentrating defect (cystic damage to nephron distal tubules impairing receipt of vasopressin signals) - liver cysts (hepatomegaly) and cerebral aneurysms also common **Renal cell carcinoma can present w/unilateral flank mass but wouldn't explain pt's advanced CKD and is less likely in the absence of hematuria, fever, and weight loss

22 yo w/ 6 mo hx of progressive pain and swelling of hands and bluish discoloration of the fingers when getting exposed to cold. She also describes frequent episodes of food getting stuck in chest while swallowing. There is an erythematous rash over malar area and nasal bridge. Skin of hands appears shiny and thick, w/small ulcers on the fingertips. Hand and wrist joints are swollen and tender. she has positive antinuclear antibody and anti-U1 ribonucleoprotein. She will most likely exhibit clinical manifestations of what condition in the future?

polymyolitis (mixed connective tissue disease) - variable features of SLE, systemic sclerosis, and polymyositis **TTP is a rare complication seen in SLE but a patient w/mixed connective tissue disease is much more likely to develop polymyositis than TTP

21 yo pt who had spontaneous vaginal delivery 2 days ago after induction of labor for gestation HTN now has nausea, vomiting, and severe headache. The headache is better when she lays down, but nothing else helps it. During delivery, she had epidural placed for analgesia that was removed immediately following deliver. She has neck stiffness and is unable to sit up due to severe nausea. There is no papilledema and the pupils are equal and reactive to light. B/L lower extremities have pitting edema to the knees. DTRs are 2+. Dx?

postdural puncture headache during placement of neuraxial anesthesia, unintentional dural puncture may occur, causing cerebrospinal fluid leakage, low cerebrospinal fluid pressure, and resultant slight herniation of the brain and brainstem. Therefore, patients can develop a positional headache (ie, worsens when sitting or standing and improves w/lying down), which correlates with the increased and decreased herniation/pressure on the brain, w/in 72 hrs of the procedure

A newborn boy has respiratory distress. Born at est 37 wks gestation via spontaneous vag delivery to 20yo primigravid mother w/no prenatal care. Puls ox is 84%, has flattened face, and there are intercostal retractions and bilateral lung sounds have decreased aeration. Abdomen is distended and there is a palpable, soft, suprapubic midline mass. He has cyanotic and bilateral clubfeet. Most likely underling cause?

posterior urethral valves hypoxia w/ decreased lung aeration (suggestive of pulmonary hypoplasia), flattened facies, and limb deformities are consistent w/ Potter sequence, which represents the pathophysiologic manifestations of any condition associated w/inadequate amniotic fluid levels (eg, urinary tract obstruction, renal agenesis, PCKD, preterm premature ROM) This patient's abdominal distension and suprapubic mass are consistent w/bladder distension resulting from urinary tract obstruction - most common cause in newborn boys is posterior urethral valves **congenital diaphragmatic hernia would present w/scaphoid abdomen and barrel-shaped chest (no abdominal mass)

45 yo has progressive weakness over past several mos. She has difficulty picking her child up and difficulty swallowing w/several episodes of choking while drinking water. She has normal sensation and DTRs. Strength is 4/5 in the deltoid and quadriceps and there is no limitation in ROM. ESR is 50mm/hr, and CK is elevated. TSH is normal. Best tx?

prednisone (polymyositis) - chronic proximal weakness w/out other neurological findings and an elevated CK - proximal muscle weakness manifests as diffic climbing stairs, getting out of car, and overhead arm activities (combing hair, lifting kid) - involvement of upper esophageal musculature can cause dysphagia **pyridostigmine is a cholinesterase inhibitor used to tx myasthenia gravis, which is characterized by weakness that worsens w/activity and involves facial and distal limb muscles as well as proximal muscles **riluzole is used for ALS, a progressive neurodegenerative disorder that causes asymmetric upper (spastic weakness, hypereflexia) and lower (muscle atrophy, fasciculations) motor neuron features

56 yo man w/ gradually enlarging, painless mass above right angle of the jaw. He has also recently experienced right-sided facial weakness. Had radiation to neck in 20's for Hodgkin disease. Mother had thyroid cancer. PE shows firm, nontender mass in R parotid and weakness of all right-sided facial muscles. What part of this presentation is most concerning for malignancy?

presence of facial droop - most parotid neoplasms are benign, but exposure to radiation is a risk factor for malignant disease - both CN VII and CN V are closely associated w/parotid gland - presence of facial droop (CN VII dysfunction) or facial numbness (CN V dysfunction) is very concerning for neural invasion due to malignant disease

75 yo w/ well-controlled HTN and DMII has normal extraocular exam, but fundoscopic exam showing enlarged optic cup w/increased cup/disc ratio. automated visual field testing reveals decreased peripheral vision in all axes. Likely dx?

primary open-angle glaucoma - manifests as loss of peripheral vision in assoc w/enlarged optic cup and increased cup/disc ratio - type of optic neuropathy characterized by atrophy of optic nerve - typically assoc w/increased intraocular pressure, as measured by tonometry, but finding isn't highly sens or spec **age-related macular degeneration presents w/ slowly progressive central (not peripheral) vision loss - exam shows subretinal drusen and pigment anomalies

36 yo female w/dyspnea on exertion. Lungs are clear to auscultation. Chest x-ray shows enlargement of the pulmonary vessels w/rapid tapering of distal vessels (pruning) and enlargement of RV. Dx?

primary pulmonary HTN

18 yo g2p1 at 38 wks gestation has severe abdominal pain and vaginal bleeding. Pregnancy 2 and a half yrs ago ended in a classical cesarean delivery at 24 wks gestation for placental abruption. She smokes cigarettes and uses cocaine multiple times a week. BP is 90/60 and pulse is 130. Abdomen is tender and has an irreg mass; no contractions are palpated. Bright red blood is noted on perinatal pad. FHR tracing shows multiple prolonged decelerations to 100/min. What risk factor most likely contributed to presentation?

prior uterine surgery (uterine rupture) uterine rupture can present w/ severe, sudden-onset abdominal pain and vaginal bleeding; patients may have a palpable, irregular abdominal mass (ie, fetal parts) and fetal decelerations due to fetal hypoxia and cord compression - additional findings may include loss of fetal station and change in contraction pattern (ie, decreased uterine tone) - significant risk factor is prior uterine surgery, particularly c-section **tobacco and cocaine increase risk of placental abruption via vasoconstriction, resulting in placental ischemia, infarction and premature placental separation from uterus - not associated w/ irreg abdominal mass

36 yo has syncopal episode at work. She is currently on day 10 of her menses, which have become heavier and longer over past yr. She has frequent light headedness and palpitations. She is sexually active w/bf and on OCPs. BP is 100/60 and pulse is 112. Pelvic exam shows irregularly enlarged uterus 12 wks in size. Hgb is 4.9 and MCV is 75. UPT is neg. Most likely cause of anemia?

proliferation of smooth muscle cells within the myometrium (fibroids) fibroids, especially those that are submucosal or intramural, are a common cause of heavy menstrual bleeding proliferation of smooth muscle cells w/in myometrium can cause profuse menses as well as irregular uterine enlargement **adnenomyosis is the proliferation of endometrial glands inside the uterine myometrium - clinical fx include bulky, tender uterus that is uniformly enlarged

6 yo boy w/congenital deafness is brought to ED after fainting while running. He regained consciousness quickly and without confusion. His brother drowned at age 7. Normal BP, HR and serum chemistry. EKG shows prolonged QT. What is most appropriate tx?

propanolol and pacemaker (Jervell and Lange-Nielson syndrome (autosomal recessive)) beta blockers are class II antiarrhythmics and are class of choice to blunt exertional HR and shorten QT interval (CCBs not effective) symptomatic patients require beta-blocker therapy plus long-term pacemaker placement

40 yo man comes to ED w/worsening RLQ pain that radiate to right groin w/fever and anorexia. Two weeks ago, he was treated for furunculosis of right thigh. Extension of the right hip increases pain, and flexion decreases pain

psoas abscess CT scan required to confirm the diagnosis

38 yo g5p5 has difficult and painful ambulation postpartum day 1. She has no pain while laying down, but reports sharp low midline abdominal pain that radiates down legs when walking. No difficulty voiding and is passing gas but hasn't had bowel movement yet. She has no numbness or foot drop. She received epidural anesthesia during labor and had vaginal delivery of 9lb8oz infant that was complicated by shoulder dystocia and McRoberts maneuver was used to dleiver. Neurologic exam is normal. Abdominal exam shows focal tenderness to palpation just inferior to the bladder. Pelvic exam reveals normal lochia, no vaginal masses, and a nontender uterine fundus just below the umbilicus. What is the most likely diagnosis?

pubic symphysis diastasis - during preg, increased levels of progesterone and relaxin increase pelvic mobility and promote physiologic pelvic widening (diastasis) - in traumatic delivery patients can develop symptomatic pubic symphysis diastasis - presents w/ suprapubic pain that radiates to back, hips, or legs and is exacerbated by walking, weight-bearing, or position changes - point tenderness to palpation over pubic symphysis **femoral nerve injury can result from hyperflexion of thigh (eg, McRoberts maneuver) but presents w/ numbness over anterior and medial thigh, inability to extend leg or flex hip, and diminished patellar reflexes

10 yo boy worked up in ED after MVC has significant bruising and severe pain over the right chest. X-ray is normal. Then, 6hrs after observation, he develops tachycardia and tachypnea and needs supplemental O2. pH is 7.42 and chest xray reveals patchy, irregular alveolar infiltrates of the peripheral right middle and lower lobes

pulmonary contusion

43 yo brought to ED 8 hrs ago, after high-speed MVC causing right femur fracture as well as bruising of face and chest without underlying fractures. She now has increased SOB (90% on 4l O2). Diffuse rales are present in anterior lung fields bilaterally, she has tachypnea and tachycardia, heart sounds are normal, and CT shows peripheral, anterior ground-glass opacities in both lungs

pulmonary contusion (subsequent resuscitative fluid administration can exacerbate the alveolar edema, leading to progressive dyspnea, tachypnea, and hypoxemia) Management includes pain control, pulmonary hygiene and respiratory support **Myocardial contusion can cause contractile dysfunction (eg, left ventricle) and cardiogenic shock with tachycardia and hypoxemia. However, these symptoms would be accompanied by hypotension and pulmonary edema would be diffuse, rather than anterior and peripheral

35 yo g2p1 at 32 wks w/HELLP is admitted to hospital and IV line and urinary catheter are placed. She receives corticosteroids to accel fetal lung maturity and mag sulf for fetal neural protection. She develops dyspnea and drop in arterial O2 sat 3hrs later. Afebrile, BP 150/80, pulse 112, resp 24, puls ox 91%. Exam shows bibasilar crackles, use of accessory muscles, and 2+ edema of lower extremities; otherwise, exam is normal. Urine output is 60ml/hr. What is most likely cause of symptoms?

pulmonary edema - preeclamptic patients have generalized arterial vasospasm leading to increased SVR and high cardiac afterload - Heart becomes hyperdynamic to overcome systemic HTN - decreased renal function, decreased serum albumin, and endothelial damage lead to increased capillary permeability and may contribute to pulmonary edema **mag sulf toxicity is characterized by decreased respiratory effort apnea (pt has increased effort), muscle paralysis, somnolence, visual disturbances, and decreased or absent DTRs

37 yo w/SOB and chest pain that began 2 days ago and have worsened. Has MS and is wheelchair bound due to spastic paraparesis and has saccadic speech. Temp 98, BP 120/70, pulse 110, and resp 18. Puls ox 94%. There are reduced breath sounds and dullness to percussion at the left lung base. Labs: WBC 7100 (neutrophils 80%). Xray shows small left-sided pleural effusion and no infiltrates are seen. Most likely cause of pleural effusion?

pulmonary embolism - patient w/prolonged immobilization w/ sudden-onset chest pain, dyspnea, and tachycardia - pts w/ PE commonly develop small pleural effusion due to hemorrhage or inflammation (tend to be exudative and grossly bloody and can be associated w/ pain due to pleuritic irritation) **aspiration pneumonia is common in pts w/ MS who have bulbar symptoms (eg, saccadic speech). It may also present w/ pleuritic pain and exudative pleural effusion, but typically causes fever, WBC increase, cough and lobar infiltrate on x-ray

A 72 y.o. man comes to the ER after being intubated by at the scene. Was found unconscious, with weak pulse and agonal respirations. He has HTN, DM2, CAD, and metastatic prostate cancer. BP is 80/40, pulse 120 and regular. Patchy rales are present and there is 2/6 apical systolic murmur. Upper and lower extremities are cold and clammy. There is a nonhealing wound on right foot. Right atrial pressure is 18 (norm: 0-18), pulmonary artery pressure is 43/21 norm: 15-28/5-16), and PCWP is 9 (norm: 6-12). Most likely diagnosis?

pulmonary embolism elevated pulmonary artery and right atrial pressures are consistent with pulmonary HTN PCWP reflects left atrial pressure and left sided heart function - patient's normal PCWP reflects normal left ventricular function, suggesting that elevated pulmonary pressures are seen due to intrinsic pulmonary process An acute massive pulmonary embolism can cause abrupt increases in RA pressure to >10 and PA pressure >40, leading to decreased venous return to left atrium, decreased cardiac output, hypotension, and obstructive shock

18 yo w/ mild dyspnea on exertion. Lungs are clear to auscultation. During expiration, there is an extra high-pitched sound heard after S1. There is a grade 3/6 systolic cresendo-decrescendo murmur that is loudest at the left upper sternal border. The S2 is split through the respiratory cycle and splitting increases w/inspiration. Murmur doesn't increase w/ standing. Most likely mechanism of murmur?

pulmonic stenosis - often asymptomatic until into adulthood - pulmonic ejection click = high pitched sound after S1 heard best after S1 is followed by crescendo-descrescendo systolic murmur heard over left intercostal space (murmur intensifies with inspiration when there is increased blood volume to right heart) - Stenosis causes pulmonic valve to close later than usual, resulting in widened splitting of the aortic and pulmonic components of S2; splitting is further increased during inspiration **ASD patients can have mid-systolic murmur due to increased flow across pulmonic valve; however, the S2 is widely split without variation with respiration (wide and fixed splitting)

5yo woman who started to have R UE and LE weakness while jogging that has gradually progressed over the last hr. Now has severe headache, nausea, and vomiting. PMH of HTN and stopped taking meds a week ago. BP is 174/102 and exam shows right hemiplegia, right hemisensory loss, and leftward deviation of eyes. Likely dx?

putaminal hemorrhage this pt has hypertensive intraparenchymal brain hemorrhage, which typically presents w/ sudden focal neurologic deficits that gradually worsen over minutes to hours symptoms often occur during routine activity but may be precipitated by exertion (eg, jogging). as the hemorrhage expands, headache, vomiting, seizures (if lobar), and altered mental status can occur hypertensive hemorrhages generally involve the same small, penetrating arteries that are responsible for lacunar stroke the most frequently affected locations include the basal ganglia (putamen), cerebellar nuclei, thalamus, and pons putaminal hemorrhage almost always involves adjacent internal capsule. this leads to c/l hemiparesis and hemianesthesia (due to disruption of corticospinal and somatosensory fibers in the posterior limb) and conjugate gaze deviation toward the side of the lesion (due to damage of the frontal eye field efferents in the anterior limb)

54 yo w/bright red blood per rectum. She passed 3 large, bloody stools over past 2 days, which have been associated w/mild abdominal cramps. She underwent radical hysterectomy and adjuvant therapy for cervical cancer over 12 mos ago. Hgb 9.8, WBC 6,200. A colonoscopy is performed and shows mucosal pallor, friability, and multiple telangiectasias, all of which are confined w/in the rectum. Scattered diverticula are present but limited to the descending and transverse colon. Most likely dx?

radiation proctitis (chronic) - pt w/ past radiation therapy w/ hematochezia and colonoscopy findings of mucosal pallor, friability, and multiple telangiectasias confined to the rectum **angidysplasia is an AV malformation that usually occurs in elderly pts as well as those w/ CKD and severe aortic stenosis. Although it morphologically resembles telangiectasia, the presence of colonic mucosal pallor and friability and hx of pelvic radiation is more consistent w/ radiation proctitis

5 yo boy brought to office for annual physical. Uncle died of MI at 56. Physical is within normal limits. Auscultation of chest shows grade II systolic ejection murmur at the left lower sternal border that is best heard when child is lying down and decreases when standing. What is the best next step in management?

reassurance maneuvers that decrease venous blood return to the heart (eg, standing, Valsalva) typically reduce the intensity of innocent murmurs

24 yo woman at 37 wk gestation is requesting induction of labor, She is very uncomfortable due to exhaustion and SOB when walking w/ leg swelling bilaterally and crampy leg pain that is relieved by massage. ROS neg for contractions, vaginal bleeding, or loss of fluid. Urinalysis shows trace protein and HR is 110. Cardiac auscultation shows 2/6 systolic ejection murmur. Lungs are clear to auscultation. Best next step in management?

reassurance and routine follow-up - Increased cardiac output results from an increase in stroke volume in early pregnancy and faster HR in late pregnancy and is frequently accompanied by a systolic ejection murmur - plasma volume expansion can cause peripheral edema **an echocardiogram would be indicated for eval of peripartum cardiomyopathy, which may cause peripheral edema and dyspnea. However, the absence of cough, hemoptysis, and a holosystolic mitral regurg murmur makes further workup unnecessary

9yo w/ lower extremity weakness and pain that began yesterday. Now has difficulty standing and tingling sensation in toes and fingers as well as lower back pain. Heart and lung exam are normal aside from bradycardia. Has decreased strength and diminished reflexes in both lower extremities. Pain and mildly decreased sensation with pinprick sensation. What is most associated with the patient's underlying diagnosis?

recent diarrheal illness (Guillain-Barre syndrome) - ascending weakness and hyporeflexia originating from the lower extremities - neuropathic pain is predominant in children, and autonomic dysfunction (eg, bradycardia) occurs in many cases **polio causes acute flaccid paralysis and is more likely in a child with an incomplete vaccination hx, but weakness is asymmetric and typically preceded by aseptic meningitis (eg, fever, headache, fatigue)

69 yo female w/ increasing abdominal pain for past several hours that is worse with cough or movement. She has no nausea, vomiting, or diarrhea. For 2 days she has been coughing and wheezing due to asthma exacerbation. She also has HTN and afib, for which she takes lisinopril, diltiazem, and apixabean. Had abdominal hysterectomy 5 yrs ago. Vitals are stable, expiratory wheezing, and localized fullness with tenderness and guarding just lateral to the umbilicus on the left side. Bowel sounds normal in all quadrants. WBC 12,000 and Hgb 10.8. Likely cause?

rectus sheath hematoma - pt taking apixaban w/acute abdominal mass and anemia makes this likely - is due to rupture of the inferior epigastric artery and can be seen with blunt trauma or forceful abdominal contractions in people on anticoagulation therapy **although paraumbilical hernias can occur after abdominal surgery and may cause a painful abdominal mass (particularly if strangulation occurs), patients typically have a hx of a palpable bulge that enlarges with increased intraabdominal pressure (eg, coughing) and anemia wouldn't be expected

64 yo w/ worsening SOB and rhinorrhea and cough over past 3 days. 40 pack yr hx and baseline SOB w/exertion. Also has bilateral ankle swelling and is overweight. Has bilateral wheezes and prolonged expiratory phase. WBC 14,5000 and Hgb is 16. pH 7.34 w/ PaO2 of 68 and PaCO2 of 65. The absence of marked acidosis in this pt is best explained by what?

renal tubular compensation Patients w/ chronic hypoventilation due to COPD, obesity hypoventilation syndrome, or neuromuscular causes have gradual increases in PaCO2 that result in chronic respiratory acidosis; to compensate, the kidneys increase bicarb retention, creating a secondary metabolic alkalosis

34 yo man w/ severe, sudden-onset penis pain following sex. He heard a cracking sound followed by pain and rapid loss of erection. He is unable to pass urine. Penis is swollen and ecchymotic. Following adequate analgesia, what would be the best next step in management?

retrograde urethrography - used anytime there is blood at meatus, hematuria, dysuria, or urinary retention prior to surgery **dont manage penile fx w/ cold compresses, pressure dressings, and splinting **color doppler US is used in cases of testicular torsion, not in penile fractures

60 yo alcoholic is hospitalized w/multiple generalized tonic-clonic seizures. On day 2 of hospitalization, temp 98.8, BP 155/92, pulse 108, and resp 14. K+ 5.6, BUN 36, Creatinine 2.4. Urinalysis shows 2+ blood and trace ketones. Urine microscopy shows wbcs 2-5/hpf, rbcs 0-1/hpf, and some epithelial cells. What is the most likely cause of his renal dysfunction?

rhabdomyolysis - renal insufficiency and discordant blood findings on urine testing following tonic-clonic seizures rhabomyolysis releases myoglobin (a skeletal muscle O2-storage protein that contains heme pigment) into the bloodstream in the kidneys, myoglobin is degraded and releases heme into the renal tubules; this can cause renal tubular obstruction and tubular epithelial injury, resulting in acute tubular necrosis (ie, renal insufficiency). Urinalysis detects heme rather than RBCs Potassium and phosphorus are released from lysed muscle cells, causing hyperkalemia and hyperphosphatemia

A 45-year-old female undergoes a successful exploratory laparotomy for acute abdominal pain. During the procedure a 2 cm mass is discovered on the appendix. Subsequent pathology reports indicate it to be malignant carcinoid with positive margins. Her past medical history is unremarkable and her only medication is an oral contraceptive pill. The most appropriate management is:

right hemicolectomy For appendiceal carcinoids a right hemicolectomy is indicated if the tumor is greater than 1.5 cm. If there are no signs of tumor spread and the tumor is less than 1.5 cm just an appendectomy can be performed. When a patient presents with carcinoid syndrome octreotide can be used as treatment.

A 54-year-old female presents to the emergency room with abdominal pain of one week's duration that has been intermittent but has not completely resolved. On exam you note RLQ tenderness and a tender nodule that is adjacent to the tip of the 12th rib on the right. A CT scan is performed, which shows a distended appendix with a 2.5 cm cystic lesion. The appendix is removed and pathology describes a cystadenocarcinoma. The most appropriate next step in management is what?

right hemicolectomy while cystadenomas can be treated w/ appendectomy alone, a right hemicolectomy is indicated for all cystadenocarcinomas greater than 2cm in diameter **not tx w/adjuvant chemo

53 yo woman brought to ED after collapsing suddenly and losing consciousness for approx 3 mins. Is now very weak and dyspneic with left-sided chest pain. Had no tonic-clonic activity or incontinence. Was diagnosed with colon cancer and is scheduled to undergo resection. Has HTN and doesnt take meds. BP is 86/50, pulse 120, pulse ox 80%, JVP 13. Lungs are clear to auscultation. Further workup would likely show what?

right ventricular dysfunction from acute massive PE malignancy w/ dyspnea, chest pain, tachycardia, hypoxia, and clear lungs is highly suggestive of PE - massive PE can present w/ syncope and hemodynamic collapse Acute massive PE results in abrupt increase in pulmonary vascular resistance and, subsequently, right ventricular pressures - elevated RV pressures cause increased RV wall tension, cardiac muscle stretching, and RV dilation. This increases RV oxygen demand and decreases coronary artery perfusion, which causes supply/demand mismatch and RV ischemia **pericardial tamponade wouldn't cause dramatic hypoxia

56 yo woman w/hx of asthma has had increased dyspnea over past several weeks and are more severe today. She is coughing up dark colored sputum. Puls ox is 93% and she is speaking in short sentences and is using accessory muscles of respiration. Trachea is deviated to right and there is dullness to percussion on the right. Scattered rhonchi are present bilaterally, and breath sounds are diminished over the right lower lung. What best explains exam findings?

right-sided atelectasis - mucus hypersecretion can form mucus plug in airways leading to obstruction w/ distal air trapping and additional air can't enter that portion of the lungs - trapped air molecules eventually diffuse into blood stream, creating a vacuum that leads to alveolar collapse (ie atelectasis) - trachea pulled toward atelectasis - because collapsed lung has greater density than air filled lung, there is dullness to percussion - lack of air flow causes decreased breath sounds **pneumothorax and pleural effusion present the same except trachea moves away from them

3 yo was playing outside when he started coughing uncontrollably and wheezing. He has no chronic medical conditions. Resp 48/min and pulse ox 91%. Right lung has diminished breath sounds and expiratory wheezing. Left lung is clear. Supplemental O2 is given. Chest x-ray reveals hyperinflation of the right lung with left mediastinal shift. Best next step in management?

rigid bronchoscopy (foreign body aspiration) **don't need CT scan because X-ray already shows foreign body aspiration

47 yo woke up this morning w/ a right-sided facial droop. He is unable to fully close his right eye. He has no headache, neck stiffness, extremity weakness, or numbness, but reports fatigue and low-grade fevers for the past several weeks. He is sexually active w/ multiple women. PE shows flattening of the R nasolabial fold, drooping of the R corner of the mouth, and R-sided hyperacusis. Sensation to touch on the face is normal b/l. Parotid glands are enlarged b/l, and scattered enlarged cervical lymph nodes are present. Lungs are clear to auscultation. Hgb 13, PLT 260,000, WBC 8200, Ca2+ 11. Most likely cause of facial paralysis?

sarcoidosis patient w/ new-onset facial nerve palsy, hypercalcemia, b/l parotid gland swelling, and constitutional symptoms has sarcoidosis w/ extrapulmonary manifestation - although lung involvement (eg, hilar lymphadenopathy) is common in sarcoidosis, it is often (>50%) asymptomatic, and pts may present w/extrapulmonary disease

64 yo woman w/ bitemporal headache that has slowly worsened over past week w/ dizziness, blurry vision, and tingling and numbness of her feet. No fever or jaw pain. Temp is 99, BP is 140/90, pulse is 70. B/L pupils are equal and reactive. Visual acuity is decreased in both eyes, and fundoscopic exam reveals dilated, segmented, and tortuous retinal veins. There is decreased pinprick sensation and loss of ankle reflex b/l. Hgb 11.2, Tprotein 10.5, albumin 3.7, ESR 80. Best next diagnostic study?

serum protein electrophoresis (Waldenstrom macroglobulinemia (WM)) excessive production of monoclonal IgM antibody causes hyperviscosity syndrome w/ peripheral neuropathy, headaches and vision changes PE typically shows "sausage-link" (dilated, segmented, tortuous) retinal veins, and lab studies reveal anemia, gamma gap, and elevated ESR

23 yo had anterior shoulder dislocation while playing basketball and right arm is held in slight abduction and external rotation. Distal pulses are full. If left untreated what is the most likely complication of the pt's injury?

shoulder abduction weakness (injury to anterior nerve) anterior dislocations are typically caused by a blow to an externally rotated and abducted arm when the head of the humerus is displaced anteriorly, there is flattening of the deltoid prominence, protrusion of the acromion, and anterior axillary fullness (due to humeral head's displacement into this location) The axillary n is most commonly injured and innervated teres minor and deltoid (weakened shoulder abduction) muscles - also provides sensory innerv over lateral shoulder **radial n is primarily injured during humeral mid-shaft fractures and use of improperly fitted crutches (weakened extensors of hand and wrist)

16 yo w/ daily, throbbing headaches in frontal region that are worse in the mornings and occasionally associated w/nausea and vomiting. He has also had several brief periods of blurriness in his left eye. He takes isotretinoin for severe acne. Temp 98.1, BP 130/80, pulse 70, and resp 15. Has bilateral optic disc edema and decreased visual acuity. Normal neurologic exam. Likely cause of symptoms?

side effect of isotretinoin (idiopathic intracranial HTN) - excessive vitamin A and its metabolites (eg, isotretinoin) are believed to impair CSF reabsorption, leading to increased ICP **optic chiasm glioma can cause insidious headache and vision changes. However, with a tumor compressing the optic chiasm, a visual field defect (eg, bitemporal hemianopsia) would be expected

42 yo w/ nonpruritic, nonpainful skin lesion on left upper arm that began 2mos ago. He also has tingling and numbness of left fingers. Emigrated from SE Asia to Connecticut a yr ago. Skin exam shows 4-cm, well-circumscribed, hypopigmented patch on the left upper arm w/ no sensation to pinprick. Ulnar n is thickened and tender at the left elbow. touch and pain sensation is absent in the left ulnar n distribution. What is most likely to confirm dx?

skin biopsy from edge of lesion (leprosy) - chronic, anesthetic (often hypopigmented) skin lesions - nodular, painful nearby nerves w/ loss of sensory/motor function

53 yo woman w/ right sided abdominal pain that started 2 days ago and is constant, burning, and interfering w/sleep. No associated nausea, vomiting, or diarrhea and no response to antacids or ibuprofen. Had breast cancer a yr ago w/ modified radical mastectomy followed by chemo w/ last course 3 wks ago. Lungs clear and abdomen soft and nondistended. Liver span is 8cm. Lightly brushing skin to the right of the umbilicus elicits intense pain. What additional feature is patient likely to develop?

skin lesions (herpes zoster virus (shingles)) - frequently triggered by severe physical stress (like cancer in this patient)

38 yo woman w/intermittent abdominal bloating and diarrhea for past 2 mos. She has 5-6 large volume, greasy stools a day. Often severe aft meals, but no specific foods that trigger it. Underwent gastric bypass 2 yrs ago. Abdominal exam shows diffuse, mild tenderness to palpation w/out rebound tenderness. Bowel sounds are increased. Stool testing shows increased fecal fat. Hgb 10.6, MCV 106 potassium 3.5, bicarb 22, glucose 110. Most likely cause of symptoms?

small intestinal bacterial overgrowth - bloating, steatorrhea (elevated fecal fat), and macrocytic anemia in setting of gastric bypass proximal small intestine normally contains minimal bacterial colonization due to gastric acidity and peristalsis; but gastric bypass allows for bacterial growth - nutritional deficiencies of B12 and fat-soluble vitamins often seen **dumping syndrome is seen w/ high-carb foods rapidly emptying into small bowel, leading to osmotically driven fluid shifts from plasma to intestine - have abdominal pain and diarrhea soon after eating, but fluid shift also results in sympathetic activation, leading to tachycardia, diaphoresis, and flushing - not assoc w/steatorrhea or vitamin B12 defic

57 yo man comes to ED w/ right arm and leg weakness that began 2 hrs ago. He is unable to shake hands and walks w/ a limp. He has had a mild, constant headache for past several days. PMH of HTN and hyperlipidemia. BP is 180/100 and pulse is 80. There is mild asymmetry of lower face, decreased muscle strength of R arm, and pos babinski on R side. sensory exam is normal. blood glucose is 210. ECG shows sinus rhythm w/occasional PVCs. Noncontrast CT reveals no abnormalities. Most likely cause?

small-vessel lipohyalinosis pt shows signs of lacunar stroke in the internal capsule lacunar infarcts occur due to occlusion of deep penetrating arteries in the brain these vessels are located in highly turbulent region of the cerebrovascular system, and therefore are most susceptible to vascular disease related to HTN, diabetes, hyperlipidemia and smoking due to their small size, lacunes are often not appreciated on noncontrast CT scans obtained lacunar infarcts commonly occur in the internal capsule and may lead to pure motor hemiparesis due to injury of the corticospinal (posterior limp) and corticobulbar (genu) tracts - pts typically have weakness involving c/l face, arm, and leg - sensory deficits, mental status changes, seizure, and cortical signs are typically absent

17 yo girl w/ lower extremity weakness that started w/ tingling in feet 5 days ago. She now cannot walk or stand. Had URI 2 wks ago but is otherwise healthy. Vitals are normal. Bilateral ankle and knee flexion/extension strength is 1/5 and hip flexors are 2/5. Reflexes are absent in lower extremities. CSF shows protein 200, glucose 65. Next step in management?

spirometry (Guillain-Barre syndrome) - need to assess pulmonary function because neuromuscular respiratory failure is a life-threatening complication - decline in FVC indicated impending respiratory failure warranting endotracheal intubation

19 yo was found by roommate 30 mins ago "convusling" on bed. EMS was called and pt had another brief seizure in ambulance. No hx of seizures. Roommate says he went to bed late last night after frat party. Temp is 99, BP is 148/90, and pulse is 94. On PE, pt is somnolent, groans to painful stimuli, and doesn't follow instructions. There is a small tongue laceration. During exam, he has another tonic-clonic seizure, which terminates after admin of IV lorazepam. Most approp next step in management?

start fosphophenytoin infusion (generalized convulsive status epilepticus (GCSE)) GCSE is defined as a seizure lasting greater than 5 mins or as more than 2 seizure events in which the pt doesn't completely regain consciousness (as in this pt) - IV benzos should be administered for seizure termination; midazolam may be administered IM if IV access unavailable - a nonbenzo antiepileptic should be administered to prevent seizure recurrence (fosphophenytoin, phenytoin, leveteracetam, or valproic acid) **after stabilization, neuroimaging (eg, brain MRI, head CT) should be performed to eval for any underlying structural abnormality, hemorrhage, or area of ischemia

68 yo female w/ COPD has increased SOB and cough for 12 hrs that hasn't improved w/ 2 extra nebulizer txs. Temp is 98, BP is 110/65, pulse is 110, resp 28. Puls ox is 84% of 2L by nasal cannula. She is given additional supplemental oxygen, IV antibiotics, methylprednisolone, and 2 txs of nebulized ipatropium and albuterol. She remains dyspneic and uses accessory muscles of respiration but is alert and follows commands. On 4L/min O2 ph is 7.32, PCO2 is 60, and PO2 is 52. Chest x-ray reveals emphysema but no pulmonary edema, consolidation, pleural effusion, or pneumothorax. Next step in management?

start noninvasive positive-pressure ventilation (NPPV) - acute exacerbation of COPD that has failed to improve w/medical management should be considered for NPPV - NPPV is ventilatory support delivered by facemask rather than endotracheal tube - decreases work of breathing, improves alveolar ventilation, and is preferred support - decreases RR and arterial CO2 w/ an increase in tidal volume, minute ventilation, and arterial O2 **invasive mechanical ventilation may be required in hypercapnic patient s w/ poor mental status, hemodynamic instability, or profound acidemia (pH <7.1) - better to try NPPV before this

pt who develops blurry vision and glare while driving after beginning steroid eye drops for acute anterior uveitis

steroid-induced open-angle glaucoma - steroids can increase intraocular pressure due to decrease outflow of aqueous humor via drainage channels in the anterior chamber, possibly worsened by swelling of structures (eg, choroid, lens) in the posterior chamber - test with tonometry

6 hr old boy w/ scalp swelling. Was born vis c-section. Delivery was complicated by 18hrs of active labor followed by arrest of descent, necessitating the c-section. Maternal uncle w/ severe hemophilia A. Pulse is 190 and appears pale. There is diffuse fluctuant edema extending from right ear across the vertex to just above the left ear. Labs: Hgb 9, plt 192,000, WBC 10,000, PT 13, PTT 38. Most likely cause of presentation?

subgaleal vein shearing - rare but potentially fatal neonatal injury caused by rupture of emissary veins upon scalp traction during delivery - these sheared veins, which connect the dural sinuses to the scalp, can cause massive blood accumulation between the periosteum and galea aponeurotica **cephalohematoma is characterized by bleeding between the skull and periosteum due to subperiosteal vessel rupture - unlike this pt, exam will show a firm, nonfluctuant swelling that doesn't cross suture lines or lead to significant blood loss

A 71-year-old male with a prolonged hospital course for pneumonia developed foul-smelling diarrhea and fevers. A colonoscopy showed pseudomembranes and the patient was started on metronidazole. Two days later, the patient continues to have high fevers and worsening white blood cell count of 20,000 cells/microL. Vitals reveal a heart rate of 125/min and a blood pressure of 82/50 mmHg. Physical examination reveals a diffusely tender and distended abdomen. A follow-up abdominal flat plate radiograph is obtained and shows a severely dilated large bowel. The most appropriate management is:

subtotal colectomy w/end ileostomy (complete colon removal with mid to lower rectum remaining) - pt has toxic megacolon secondayr to c diff w/ fulminant colitis and for this surgical emergency, this is the tx of choice

22 m who is tall, thin and a smoker presents w/ sudden onset of dyspnea and right sided chest pain that is worse with deep inspiration or cough. BP 140/90, pulse 86, resp 18, puls ox 98%. Chest x-ray reveals small right apical pneumothorax. What is the most appropriate management of this patient?

supplemental O2 (small pneumothoraces under 2cm can be managed with observation and supplemental O2 (regardless of O2 saturation), which enhances speed of resorption) stable patients with large pneumothoraces should undergo decompression with a large-bore needle (eg, 14- to 18-gauge) inserted in the 2nd or 3rd intercostal space in the midclavicular line o at the 5th intercostal space in the mid or anterior axillary Patients who are unstable should undergo emergent placement of a tube thoracostomy (if not available, urgent needle decompression can be performed)

28 yo female w/ R shoulder pain and weakness after returning from 2-week hiking trip. She had no falls or trauma. PE reveals weakness of R shoulder abduction and external rotation. passive ROM is full. R UE sensation and DTRs are normal. Most likely cause of symptoms?

suprascapular nerve entrapment (SNE) suprascapular n provides motor supply to supraspinatus and infraspinatus muscle SNE can develop due to external compression of nerve at the suprascapular notch, such as from excessive use of a heavy backpack when hiking or direct blow to nerve from fall or repetitive motion at shoulder (eg, weight lifting, baseball) symptoms: shoulder pain, weakness of abduction (supraspinatus) and external rotation (infraspinatus) w/ preserved passive ROM **AC sprain is usually caused acutely by a fall or direct blow to shoulder. As in SNE, pain may be exacerbated by cross-body adduction of arm; however, weakness of shoulder in abduction isn't seen

32 yo woman comes to EE w/ abdominal pain and nausea that began 2 days earlier and has become increasingly severe over past 3 hrs. She has passed several blood clots vaginally over last hr. Has hx of irregular menses and doesn't remember date of LMP. Has "heart-shaped uterus." BP 90/55, pulse 120. Abdominal exam shows guarding and decreased bowel sounds. Speculum exam shows moderate bleeding with clots. UPT +. TVUS shows a gestational sac at the upper left uterine cornu and free fluid in the posterior cul-de-sac. Most approp next step?

surgical exploration - pt has acute abdomen (eg, guarding and decreased bowel sounds) that is likely due to hemoperitoneum from a ruptured ectopic pregnancy in the left uterine cornu - a gestational sac normally implants in the upper fundal region; implantation in the outer quadrants (cornual areas) is abnormal **dilation and curettage is performed to remove uterine contents for spontaneous or incomplete abortion - spontaneous abortion doesn't present as acute abdomen

65 yo comes to ED aft having seizure. PMH of non-small cell lung carcinoma (NSCLC) dx 2 yrs ago and tx surgically. He works as college professor and usually walks 1-2 miles a day. MRI of brain shows solitary cortical mass (2cm x 3.5 cm) in R hemisphere at the grey-white matter jxn that is most consistent w/ metastasis. CT scan of chest and abdomen shows no evidence of recurrent malig. Pt is admitted for further management and started on phenytoin. Most approp next step in management?

surgical resection of mass - brain mets is freq seen in pts w/ NSCLC - favorable survival outcomes include age < 65 yrs, good functional performance status, and stable extracranial disease - for pts w/ a single brain metastasis in a surgically accessible location and good performance status, surgical resection offers the best therapeutic option

42 yo female g2p2 w/pelvic pain. Over past 2yrs menses have become painful, w/heavy bleeding. The pelvic pain used to subside w/menses but has become constant over last few months. She has no dysuria, urinary frequency, or constipation. She had tubal ligation after her last delivery. Hasn't had a cervical cancer screening w/in past 5 yrs and isn't sexually active. Exam shows boggy uterus that is tender to palpation. What additional finding is likely present?

symmetrically enlarged uterus (adenomyosis) **enlarged, irregularly shaped uterus is common w/ uterine leiomyomata (fibroids). Fibroids cause cause heavy menstrual bleeding; however, the uterus is typically firm (not boggy) and nontender

Newborn boy w/ weight in 3rd percentile, length in 25th percentile, and 50th percentile for head circumference. Hepatosplenomegaly is present and over the next 48 hrs, infant develops jaundice, clear rhinorrhea, and a maculopapular rash on feet and buttocks that later desquamates. Likely congenital infection?

syphilis presents w/ rhinorrhea, abnormal long-bone radiographs, and desquamating or bullous rash **all congenital infections present w/ IUGR, hepatosplenomegaly, jaundice, and blueberry muffin spots

67 yo woman had sudden-onset R sided numbness, and eval revealed ischemic stroke 4 wks ago. She feels like some of sensation is returning but has transient burning pain in R UE and LE that can be induced by light touch. PMH of HTN and DM2 and 30 pack-yr hx. PE shows R side hemiesthesia and mild athetosis of R hand. There is hyperesthesia on R side demonstrated by exaggerated pain on light touch. What was the likely location of the stroke?

thalamus lacunar stroke of the thalamus is typically due to atherothrombotic occlusion of the small, penetrating (thalamogenticulate) branches of the posterior cerebral a the ventral posterolateral and ventral posteromedial nuclei of the thalamus transmit sensory info from C/L body and face - consequently, pts w/ thalamic stroke in this region present w/ sudden C/L sensory loss involving all sensory modalities - sx can be accompanied by transient hemiparesis, athetosis, or ballistic mvmts due to disruption of neighboring basal ganglia structures and corticospinal fibers in the posterior limb of the internal capsule several wks following the stroke, sensory deficits improve; however, some pts develop thalamic pain syndrome (Dejerine-Roussy syndrome) - characterized by severe paroxysmal burning pain over affected area that is classical exacerbated by light touch (allodynia)

2 hr old boy has developed worsening cyanosis over past few minutes. Birth weight was 9lb and delivery was uncomplicated. Oxygen is administered by hood, but doesn't relieve the cyanosis. Further exam shows tachypnea, subcostal retractions, a normal S1, a single and loud S2, and no murmur. Most likely cause of cyanosis?

transposition of great vessels abnormal rotation of the great vessels during cardiac devel results in aorta arising from RV and pulmonary artery from LV. as a result, deox blood coming from body goes to RA and RV and is cycled to the body thru the aorta. ox blood from lungs is returned to the lungs by left side of heart through pulmonary artery. characteristic findings include cyanosis in first 24hrs of life, single S2 (absent pulmonary component of S2 because the aorta is anterior to the pulmonary artery), and a narrow mediastinum ("egg on a string")

35 yo man w/ 5-day hx of back pain, which began after carrying heavy boxes while moving. The pain is sharp and radiates to posterior aspect of left foot and calf. Physical exam is normal w/exception of lifting left leg to 70 degrees causing burning pain radiating from low back to left foot. What is the best next step in management?

trial of NSAIDs (acute lumbosacral radiculopathy (sciatica) w/positive straight leg test) MRI is recommended only for patients with severe, progressive, or bilateral neurological deficits and X-ray has limited use

40 yo woman w/ abdominal pain for past 2 wks that has become worse of past day and isn't relieved by ibuprofen. LMP 2 wks ago and was regular w/ painful cramping and heavy bleeding on first day. She is sexually active and uses condoms for contraception. Temp is 102, bp 100/60, and pulse 92. Abdomen has diffuse tenderness to palpation that is worst in RLQ. No rebound or guarding. Has small, mobile, tender uterus and an enlarged, tender right adnexal mass. WBC 22,000. CRP and CA-125 are elevated. US reveals large, thick-walled multiloculated mass filled w/debris obliterating the right adnexa. Most likely dx?

tubo-ovarian abscess (complication of PID that presents w/ fever, abdominal pain, and complex multiloculated adnexal mass w/thick walls and internal debris) **a mature cystic teratoma may present w/abdominal pain or symptoms of torsion but doesn't cause fever and leukocytosis - US reveals ovarian cyst w/solid components, hyperechoic nodules, and calcifications **ovarian torsion presents w/ sudden onset of unilateral pain w/out fever or leukocytosis - US findings include an adnexal mass w/absent doppler flow to ovary

32 yo primigravida woman at 24wks has been having 3-4 loose, bloody stools per day and intermittent lower abdominal pain for past 4wks. On occasion she experiences urgency to use bathroom, but when she sits, she strains to produce very little stool. 2 yrs ago she had intermittent bloody stools that resolved w/out meds. On abdominal exam, the uterus is at the umbilicus and FHR is 160; there is mild diffuse abdominal pain to palpation w/out rebound or guarding. Dx?

ulcerative colitis - pt w/ hematochezia, abdominal pain, and tenesmus (eg, fecal urgency followed by straining and inability to defecate) - fertility rates are lower in women w/UC - pregnancy is a high-risk period for pts w/ UC due to placental cytokines worsening colonic inflammation - fetal risks include preterm delivery and SGA **diverticuluar bleeding typically presents w/ hematochezia in the absence of abdominal pain and is generally diagnosed in those over 60 - preg isn't a risk factor

65 yo w/slowly progressive, right-hand clumsiness for several month. Pt has ESRD and undergoes dialysis 3x/week. Exam shows decreased sensation over 4th and 5th fingers of right hand and weaker grip compared to left side. Overall muscle mass is decreased in all 4 extremities. What is the most likely site of the nerve injury?

ulnar nerve at the elbow injury to ulnar nerve at the wrist (eg, hamate fracture, compression from bike handlebar) can cause numbness and paresthesia at the medial side of the hand along with intrinsic hand weakness that patients may describe as "clumsiness" injury at the elbow will cause similar symptoms along with decreased grip strength due to denervation of the flexor muscles of the forearm

41 yo w/ loss of urine w/coughing and sneezing for 4 mos. She is sexually active and used condoms for contraception. Menses are regular and LMP 2wks ago. She has never been pregnant. At 25, she was tx for chlamydia. She smokes 10 cigs a day. PE shows irregularly enlarged anterverted and anteflexed uterus. What would most likely reveal cause of her urinary symptoms?

ultrasound of the pelvis - given her enlarged uterus with an irregular contour, the most likely cause of stress urinary incontinence is leiomyomata uteri (fibroids) - enlarged, anteverted and anteflexed uterus will put direct pressure on bladder and decrease capacity - suberserosal and pedunculated fibroids are especially likely to compress **speculum exam can examine pelvic organ prolapse (cystocele), which can cause similar symptoms, but this is unlikley in a nulliparous woman

4 yo boy said he couldn't see out of left eye when patch was on right eye in pirate halloween costume. Eye exam shows symmetric corneal reflexes. Red reflex is equal bilaterally. Cover test shows no eye deviation, and extraocular movements are intact. Visual acuity is 20/30 in right eye. In left eye he is unable to see visual acuity chart. Fundoscopic exam shows sharp disc margins and normal vessel caliber and course. Most likely cause of findings?

uncorrected refractive error (amblyopia) - during early childhood, decreased visual stimuli from one eye results in preferentially enhanced development of visual cortex associated w/ the other, normally fxning eye and therefore further decreased visual acuity of the affected eye causes of amblyopia include strabismus (ocular malalignment), uncorrected refractive error, and less commonly, vision deprivation due to conditions such as cataracts, ptosis, or corneal opacities - patient doesn't have eye deviation that would suggest strabismus or a dull red reflex due to a lens opacity

16 yo w/ knee pain that began 2 days ago w/ swelling and a deep ache. The swelling worsened yesterday and she is limping today. On palmar side of each hand, there are a few nontender pustules w/surrounding erythematous rim, each 3-4mm in diameter. Knee has effusion w/out erythema and ROM is decreased due to pain. Joint aspiration reveals 30,000 WBC. What hx factor is most commonly assoc w/ this pt's presentation?

unprotected sexual intercourse (disseminated gonococcal infection) - either presents w/purulent monoarthritis or triad of tenosynovitis, dermatitis, migratory polyarthralgia **lyme disease can present w/ rash and arthritis but rash isn't pustular and often precedes arthritis

33 yo woman noticed vaginal mass 2 wks ago after pain with intercourse. She has no irregular menstrual bleeding or postcoital bleeding. She uses OCPs and has had 4 sexual partners in the last year. She had 3 term vaginal deliveries w/largest weighing 10lb. Had an abnormal cervical cancer screening 2yrs ago but normal follow-up colposcopy. Pelvic exam shows 3-cm mass on anterior aspect of vaginal wall that doesn't change in size w/valsalva and expresses purulent discharge. What is most likely diagnosis?

urethral diverticulum (abnormal localized outpouching of the urethral mucosa and surrounding tissues) - likely arose from recurrent periurethral gland infections, which can develop an abscess that eventually breaches the mucosa - can cause dyspareunia and associated purulent discharge **bartholin gland abscesses can present as a tender vulvar mass expressing purulent discharge and causing dyspareunia, but bartholid glands are located at the posterior vulvar vestibule, not along the anterior wall

49 yo woman, g5p5, has intermittent, involuntary loss of urine over past 5 mos. She has to wear pads when she jogs now, but has no dysuria, urgency, or hematuria. She usually awakens once in the night to void. 3 yrs ago, she was diagnosed w/ DM2 and is tx w/ metformin. BMI is 31. Pelvic exam shows normal external genitalia, a well-rugated vagina, and an anterior vaginal bulge. Patient loses a small amount of urine when asked to cough. Postvoid residual vol is 40mL. Serum HbA1c is 7.7%. Likely cause of symptoms?

urethral hypermobility (stress urinary incontinence) - chronic pressure or injury to pelvic floor muscles (due to childbirth, obesity, or high-impact exercise like jobbing) can lead to pelvic floor muscle weakness - substantial weakness of pelvic floor muscles can result in urethral hypermobility, in which urethra abnormally moves w/increased abdominal pressure and is unable to fully close **vesicovaginal fistula is a complication following prolonged labor, pelvic surgery, or pelvic radiation therapy - these patients have constant (rather than intermittent) leakage of urine because the bladder continuously drains through the vagina

57 yo post menopausal women pt with vaginal spotting and increased pelvic pressure and some constipation. Dx w/ breast cancer 3 yrs ago and has been taking tamoxifen for past 2 yrs (couldn't tolerate aromatase inhibitors). Had 3 CSs in her 30s. A large, irregular mass is palpable above the pubic symphysis. On pelvic exam, there are no cervical or vaginal lesions; a small amount of blood is seen at the os. The uterus appears well supported. What is the most likely diagnosis?

uterine sarcoma (uterine mass w/ bulk symptoms (eg, pelvic pressure, constipation) and abnormal postemenopausal bleeding) - Tamoxifen increases risk - often indistinguishable form benign leiomyomata (ie uterine fibroids) **adenomyosis can present w/ enlarged uterus and abnormal bleeding (eg, heavy menses); but patients have symmetrically enlarged, globular uterus

29 yo primigravid woman at 30 wks gestation says she has had minimal fetal movement for the past few hours w/out vaginal bleeding or contractions. She has sickle cell disease and is taking low-dose aspirin. Preg dated by 10-wk US and she had fetal anatomy US at 18 wks gestation. At 24 wks, she has abnormal glucose challenge, but normal glucose tolerance. BMI is 32. FHR is 140, fundal height is 26 and US reveals cephalic fetus measuring at the 16th percentile and an amniotic fluid index of 4cm (normal > 5). What is the most likely cause of patient's presentation?

uteroplacental insufficiency - patients w/ sickle cell are at risk of multiple obstetric complication (eg, preeclampsia, fetal growth restriction) due to vasocclusion that can result in placental infarction, ischemia, and uteroplacental insufficiency - w/ decreased fetal perfusion, fetus has decreased movement and growth restriction and preferential shunting of blood flow from kidneys to brain (less urine production = oligohydramnios) **maternal obesity doesn't typically cause oligohydramnios

63 yo woman who initially only had post-coital bleeding, now has more frequent bleeding. For the past month, bleeding has been daily spotting and she is beginning to have pain w/intercourse. Just became sexually active 6 mos ago aft no sex for 10yrs. Smoke a pack a day. The vagina appears atrophic w/minimal rugation and has a 1-cm lesion in the upper 1/3rd of the posterior wall. There is a malodorous, watery discharge in the vagina. Numerous squamous epithelial cells and rare leukocytes are seen on wet mount microscopy and KOH is neg. Pelvic US shows anteverted uterus w/ a 3mm endometrial stripe. Next step in management?

vaginal biopsy - vaginal bleeding, malodorous discharge, and ulcerated vaginal lesion in a smoker are concerning for vaginal squamous cell carcinoma **endometrial biopsy is for suspected endometrial cancer in women w/postmenopausal women w/ endometrial lining > 4mm on US

A 50-year-old male is undergoing a laparoscopic Nissen fundoplication for gastroesophageal reflux disease. Which of the following structures should be isolated and handled carefully while performing distal esophageal mobilization?

vagus nerve - vagal nerve runs along the esophagus and can be inadvertently transected or injured during mobilization of the esophagus

32 yo woman starts experiencing painful contractions as she transitions into active labor. FHR is 130 w/ moderate variability, positive accelerations, and no decelerations. She requests epidural anesthesia for pain control. After induction of anesthesia, she begins to feel light-headed and BP is 90/55, HR is 120, and resp 12. She has normal strength and sensation in upper and lower extremities. Most likely cause of hypotension?

vasodilation and venous pooling hypotension is a common side effect of epidural anesthesia. the cause is blood distribution to lower extremities and venous pooling from sympathetic blockade

A 1 mo old boy becomes diaphoretic during feeding and for the past wk has only fed for 5mins at a time before falling asleep. Exam shows thin infant who is sleeping but is easily arousable. He has tachycardia and a 1/VI holosystolic murmur over left lower sternal border w/ a diastolic rumble over the cardiac apex. S2 is prominent. The patient is tachypneic, but lung fields are clear to auscultation. Most likely cause of symptoms?

ventricular septal defect producing holosystolic murmur at left lower sternal border - an apical diastolic rumble may also be heard from increased flow across mitral valve (from increased left-to-right shunting across the VSD) - as ventricular pressure increases due to increased flow, the S2 will become more prominent due to increased pressure closing the pulmonary valve **tetralogy of fallot consists of VSD, right ventricular outflow obstruction (eg, pulmonic stenosis), overriding aorta, and right ventricular hypertrophy. The pulmonic stenosis component causes a harsh, systolic ejection murmur and left upper sternal border

21 yo man has a generalized tonic-clonic seizure. Has never had a seizure before but has had progressive headaches, fever, nasal congestion, and rhinorrhea over the last 3 wks. Temp is 102, BP 130/85, pulse is 96. He is drowsy and postictal. CT scan of the head w/IV contrast reveals 3-cm ring-enhancing lesion in the left frontal lobe. There are air-fluid levels and mucosal edema in the paranasal sinuses. Rapid HIV testing is negative. Tissue bx of brain lesion is most likely to demonstrate what?

viridans streptococci or staph aureus (brain abscess) a single brain abscess is usually the result of direct extension from an adjacent tissue infection (eg, otitis media, sinusitis, dental infection) this pt w/ rhinorrhea, nasal congestion, and CT scan evidence of air fluid levels/edema in the paranasal sinuses likely had sinusitis that progressed to brain abscess (seizures, ring-enhancing lesion) **primary brain neoplasm (eg, lymphoma, glioblastoma) or metastasis could cause a seizure and a single-ring enhancing lesion on CT. However, a young patient w/ fever and several weeks of sinusitis manifestations is more likely to have a brain abscess

50 yo woman w/ progressive right shoulder pain radiating to her forearm. Hx of well-controlled RA and 25 pack yr hx. PE shows mild drooping of the right upper eyelid and right pupillary constriction. ROM over right shoulder is normal and doesn't elicit arm pain. What is the best next step in management?

x-ray of chest (superior pulmonary sulcus tumor) - right shoulder pain with ipsilateral miosis and ptosis w/ long hx os smoking raises strong suspicion of Horner syndrome due to superior pulmonary sulcus (Pancoast tumor) **CT scan would reveal a CNS lesion (eg, medullary infarction), which can sometimes cause Horner syndrome. However, this wouldn't account for progressive arm and shoulder pain (due to invasion of C8/T1 nerve roots)

53 yo man comes to ED w/ SOB and right-sided pleuritic chest pain. He was seen in urgent care 5 days ago and diagnosed with pneumonia and was given antibiotic that has not helped. X-ray shows large, loculated pleural fluid collection

complicated parapneumonic effusion (empyema) will have low ph, low glucose and high protein due to increased microvascular permeability and cellular destruction

28 yo w/painful breast mass that is 1.5cm, soft, tender, and mobile with no skin changes or galactorrhea. US shows single, thin-walled, fluid-filled cyst. FNA reveals clear, yellow tinged fluid and the mass is still palpable after aspiration. What is next step in management?

core needle biopsy ***if the aspirated fluid was clear (ie, not bloody) and the breast mass resolved completely, the patient could undergo observation and repeat breast examination in 4-6 weeks

46 yo woman w/lower extremity swelling for past 2 yrs. Has had several episodes of cellulitis involving the left leg w/ most recent one, 6 mos ago in which she was treated w/IV antibiotics, complicated by catheter-related axillary vein thrombosis requiring 3 mos of anticoag therapy. Has DMII, BMI of 34. PE shows firm edema of left lower extremity w/out erythema or warmth and examiner cant lift skin from dorsum of toes on the left foot (can on right foot). What is the most likely cause of symptoms?

disruption of lymphatics (lymphedema due to chronic inflammation from recurrent cellulitis, chronic venous insufficiency, and obesity) early disease: soft skin w/ pitting edema later in disease: progressive deposition of subq collagen and adipose leads to firm, thickened skin and nonpitting edema **patient's unilateral symptoms and hx of recurrent cellulitis make venous incompetence an unlikely underlying cause

50 yo man w/ 2 days of malaise, abdominal pain, and bloody stools who underwent kidney transplant 4mos ago. Atypical lymphocytes are seen on peripheral smear and a colonoscopy reveals multiple, large, shallow ulcerations throughout the colon. What's the most likely diagnosis?

CMV infection

Pt w/ 4 weeks of constant, progressive epigastric pain that is worse with eating and weight loss. Hx of GERD, but meds not working for this and 10-pack-yr hx. Stool occult blood is negative, upper GI endoscopy shows mild esophagitis but no gastritis, US shows 2 large gallstones w/no gallbladder wall thickening or ductal dilation. Pancreas is poorly visualized due to overlying gas. What is the next best test?

CT scan of abdomen to examine pancreas for a mass or inflammation

44 yo man w/ depression, loss of interest, impaired concentration, fatigue, and poor sleep (not suicidal) is diagnosed w/MDD and treated w/50mg sertraline. After a month of tx, he still has some sadness, so dose is increased to 100mg. At 2 mo f/u he has no symptoms and asks when he can stop taking sertraline. What is the best response?

He should continue it for an additional 6 mos (for remitted single-episode, unipolar major depression) **people w/more than 3 episodes, chronic episodes (>2yrs), or severe episodes (suicide attempt) should continue treatment indefinitely

52 yo male complains of low energy, poor sleep (waking up to early), trouble concentrating at work, not enjoying visiting his kids, and that food is tasteless. What sleep abnormalities if he likely to have?

Increased serum cortisol concentration decreased REM sleep latency (ie, decreased time from sleep onset until start of first REM sleep period) decreased slow-wave sleep

A 5-year-old child presents with increasing irritability, colicky abdominal pain, and rectal bleeding with stools that have a currant jelly appearance. A tubular mass is palpated in the right lower quadrant. Upright abdominal x-ray shows air-fluid levels with a stepladder pattern. Think:

Intussusception Barium enema is both diagnostic and therapeutic.

A patient presents with pigmented spots on his lips and a history of recurrent colicky abdominal pain. What is the cause of his abdominal pain? Think:

Peutz-Jeghers syndrome. The hamartomatous polyps are likely causing intermittent intussusception.

A 70-year-old male with a history of peripheral vascular disease and hyperlipidemia presents to the emergency department with severe, diffuse abdominal pain. His blood pressure is 170/100 and his pulse is 90 bpm. Supine abdominal radiograph shows free air in the abdomen and within the wall of the small intestine. What is the most likely diagnosis?

Small bowel infarction

28 yo with C6 paraplegia from MVC a year ago presents to ED w/ severe HTN, flushing, diaphoresis, bradycardia, and urinary retention (clear lungs and normal heart sounds)

autonomic dysreflexia (potentially life-threatening complication of spinal cord injury above T6) noxious stimuli (eg, urinary retention, constipation, pressure ulcers) can precipitate an unregulated sympathetic response, leading to vasoconstriction and severe HTN Above the lesion, a compensatory parasympathetic response causes diaphoresis, flushing, bradycardia, and nasal congestion; vasodilation occurs but can't overcome the sympathetic drive to normalize pressure Management includes placing the patient upright, removing tight clothing, and removing stimulus

58 yo fell on an outstretched right hand and injured her wrist. There is gross deformity of the right wrist with extensive swelling and bruising (colles' fracture). Patient has no sensory symptoms initially, but develops paresthesia in her hand while in ED. What is likely to be abnormal due to the associated nerve injury in this patient?

abduction of thumb against resistance median nerve injury (provides sensation to lateral 3.5 digits and motor innervation to the thenar muscles (opponens pollicis and abductor pollicis brevis)) **injury to the radial nerve at the elbow can cause weakness of hand/finger extensor muscles and sensory loss over the posterior forearm/dorsolateral hand (typically seen with supracondylar humerus fractures)

36 yo female presents to ED w/worsening fever, sore throat, neck pain, and stiffness after accidentally swallowing a fishbone 2 days ago. Temp is 102.2 and pulse is 106/min. There is pooling of saliva in the hypopharynx and the posterior pharyngeal wall is red and bulging. Lateral radiographs of the neck reveal increased thickness of the prevertebral soft tissues with an air-fluid level. Due to the contiguous spread of the disease process, the patient is at greatest risk of developing what?

acute necrotizing mediastinitus (patient has retropharyngeal abscess, which drains inferiorly to the superior mediastinum) **ludwig angina is a rapidly progressive bilateral cellulitis of the submandibular and sublingual spaces, most often arising from an infected mandibular molar

57 yo man comes to office w/ several months of difficulty walking and now has due to weakness and stiffness in both legs. He also has numbness and tingling in his arms and legs. Patient has used IV drugs, in past, 30 yr pack hx, and has had several sex partners. PE shows normal mental status and cranial nerves with weakness and atrophy of the upper arm muscles with decreased triceps reflex, b/l lower extremity 4/5 strength and vibratory and pain sensation is decreased in both the leags and hands. Babinski is upgoing bilaterally. Further evaluation is likely to reveal what?

advanced spondylosis of the cervical spine on radiography causing cervical myelopathy (spinal cord compression) patients usually develop slowly progressive gait dysfunction, extremity weakness, and vibratory/pain sensation changes w/LMN signs at level of lesion (arms) and UMN signs below the level (legs) **intramedullary cavity of cervical spine on MRI is consistent with syringomyelia (will have loss pain and temperature sensation in a "cape-like" distribution and is far less common than spondylosis)

62 yo man that had weakness in right arm and leg 6 hrs ago that resolved w/in 30 mins. Has HTN and 25 pack yr smoking hx. Has mild right-sided pronator drift, CT of brain w/out contrast shows no abnormalities, and MRI is pending. What medication is best to give him at this point?

aspirin - most likely had an ischemic stroke that is probably thrombotic due to multiple vascular risk factors - aspirin is only antiplatelet agent that is effective in reducing risk of early occurence of ischemic stroke **this patient is beyond the therapeutic window of 3-4.5 hours for alteplase therapy (thrombolysis)

22 yo g1p1 is evaluated postpartum for lower abdominal pressure and pain. Underwent induction at 41 wks due to oligohydramnios with forceps delivery due to prolonged 2nd stage of labor. Received epidural. Had 2nd degree perineal laceration repaired after delivery and PPH that resolved w/uterotonics. She is 6 hrs postpartum and is unable to void w/persistent urine dribbling. PE shows firm uterine fundus and diffuse lower abdominal tenderness. Perineal repair is intact and there is moderate lochia. What is the likely cause of urinary symptoms?

bladder atony - due to reduced sensory and motor sacral spinal cord impulses from regional neuraxial anesthesia (eg, epidural anesthesia), which can suppress micturition reflex and decrease detrusor tone - forceps-assisted delivery is another risk factor management w/ intermittent urethral catheterization and reassurance (typically self-limited and resolves in under a week) **vesicovaginal fistula can occur due to obstetric complications and cause acute urinary dribbling; however, it often takes days to develop and there is typically no associated bladder distention as urine continuously drips through the fistula

59 yo male w/ 1 mo of dysuria, urinary urgency, and frequency and dull, nonradiating suprapubic pain for the past few days. 40 yr smoking pack hx. Rectal exam shows smooth, firm enlargement of the prostate w/ no tenderness, induration, or asymmetry. No bacteria or casts on urinalysis but moderate blood

bladder cancer common manifestations: - hematuria (gross or microscopic) - voiding symptoms - suprapubic pain (usually in more advanced tumor that has penetrated the muscle and invaded soft tissue or nerves) **urethral stricture can cause voiding symptoms and urinary retention w/subsequent suprapubic pain. However, most patients have decreased urinary stream, and hematuria is uncommon

20 yo fell headfirst into hockey rink boards and did not lose consciousness, but now is unable to move arms an legs. placed in rigid cervical collar and large-bore IV catheters inserted. BP is 128/70, 102bpm, resp 14/min. Pinprick and temperature sense are absent below clavicles and pt cant move extremities; vibratory sense is intact. CT scan shows burst fracture of C5 w/posterior displaced fragments on the spinal cord. Neurosurgery consulted. What is the best next step in management?

bladder catheterization - anterior cord syndrome can disrupt the autonomic tracts involved in bladder control, leading to retention, so catheterization is done to prevent bladder distension and possible injury **he is alert w/normal respirations so doesn't need further airway protection (eg, orotracheal intubation)

patient who fell 20 ft while hanging christmas lights and has sternal bruising, severe chest and. back pain, bp 162/90, pulse 118, and respirations 24 w/symmetric breath sounds and normal heart sounds

blunt thoracic aortic injury CT angiography of chest to make diagnosis

4 day old full-term boy brought to physician for his first office visit after uncomplicated birth. Mother's blood type is A+ and had normal prenatal lab/US results. Birth weight 7lb 8oz, length 18in. Exclusively breastfed since birth and nurses for 10 min each breast every 4 hours. Mother just began producing milk today. Infant passes several meconium stools in first 2 days of life but last stool was yesterday and dark green. He now weights 6lb 10oz and has scleral icterus and jaundice. Tbili 14, direct 0.9

breast feeding jaundice due to insufficient intake of breast milk (should be feeding for 10-20 minutes every 2-3 hours and number of wet diapers should equal infants weight in days in first week) need to increase breastfeeding to every 2-3 hrs **galactosemia would have vomiting vomiting and growth failure in addition to jaundice from inability to digest galactose in breast milk or cow's milk formulas

33 yo at 39 wks gestation is set to be induced. She has had uncomplicated pregnancy except for positive GBS test w/resistance to erythromycin. She developed a nonpruritic maculopapular after taking penicillin in her 20's. What is the most appropriate pharmacotherapy?

cefazolin (lower risk of cross-reactivity but achieve the same high bactericidal concentrations in amniotic fluid without fetal toxicity) **GBS is often resistant to both erythromycin and clindamycin, but if sensitive, good drugs for woman w/severe penicillin allergy

A patient, 38 weeks gestation with sudden onset of vaginal bleeding and severe lower abdominal pain. Has 5-pack-year smoking hx and BP is 158/96 and pulse is 112. 3-cm dilated cervix w/fetus in vertex position at 0 station. FHR is 140 w/ accelerations and moderate variability. Contractions occur every 2 minutes and last for 20 secs. Urinalysis shows 3+ protein. Patient is at greatest risk of developing what complication?

disseminated intravascular coagulation (due to tissue factor released by decidual bleeding) and hypovolemic shock (fetal complications = hypoxia and preterm birth) placental abruption is retroplacental bleeding between the placenta and the uterine decidua that can cause placental detachment and vaginal bleeding **uterine rupture is associated w/prior uterine surgery and presents w/abdominal pain and heavy bleeding but patient would lose contraction pattern (rather than contractions every 2mins) and there would be loss of fetal station

29 yo woman, gravida 2 para 0, at 33 wks gestation by in vitro fertilization is inpatient for management of known vasa previa. Her membranes rupture with leakage of blood-tinged fluid. She has no contractions or heavy vaginal bleeding, vitals are stable, and FHR is 100/min. What is the best next step in management?

emergency cesarean delivery vasa previa is an aberrant condition in which the fetal vessels overlie the cervix surrounded only by a thin fetal membrane, making them prone to tear with rupture of membranes or contractions

29 yo has difficulty swallowing pork, chicken and steak occasionally over the past 2 yrs. The frequency of which this dysphagia is occurring is increasing. He has no trouble swallowing liquids. He has seasonal allergies. Abdomen is soft, nontender and nondistended. What is the most likely explanation for his symptoms?

eosinophilic esophagitis - common in younger men and associated with other atopic conditions - intermittent solid food dysphagia that's worse with meat **dysphagia in achalasia is typically progressive (rather than intermittent) and involves both liquids and solids

35 yo female w/ 5 mo hx of episodic retrosternal pain that is often precipitated by emotional stress and hot or cold food and radiates to interscapular region. Pain typically lasts 15 mins (occasionally for a few hrs). ECG is normal and pain is alleviated w/sublingual nitroglycerine. Chest x-ray, upper GI endoscope, and echocardiogram show no abnormalities/ What the is most appropriate next step in management?

esophageal motility studies (manometric recordings) - likely diffuse esophageal spasm **24 hr pH monitoring is for GERD

17 yo w/ excessive tiredness and feverishness for past 6 wks. Generalized itching but no rash. No nasal congestion, sore throat, cough, or abdominal pain. Has several enlarged, nontender, and firm cervical and supraclavicular lymph nodes. No organomegaly. Best way to establish diagnosis?

excisional lymph node biopsy (hodgkin lymphoma) **mononucleosis would present w/sore throat and tender lymphadenopathy (cervical chain) and almost all cases resolve in a month

patient right after thyroid surgery who is in no acute distress, has clear lungs, normal heart sounds, with neck incision well-approximated and a 4-cm ballotable swelling under the incision that increases in size during the examination

expanding neck hematoma (life threatening due to the possibility of either direct airway compression or from vascular compression causing venous congestion leading to laryngeal edema) do immediate wound exploration

25 yo w/abdominal pain, nausea and vomiting x 2mo that has progressed w/bilious vomiting and bowel movements in 2 days. Hx of crohns disease w/ileal involvement tx w/ infliximab. Abdomen is distended, tympanic, and tender to palpation (no rebound tenderness or guarding). What likely explains his presentation?

fibrotic intestinal stricture (common complication of crohns due to poorly controlled, sever inflammation) leading to SBO **adynamic ileus presents similarly to SBO but only develops after exposure or insult that "stuns" the bowel, like recent intra-abdominal surgery or high-dose opioids

26 yo comes to ED after being hit in the eye with small tree branch. right eye reveals no foreign bodies, but there is mild conjunctival erythema, especially around the limbus. What is best next step in management?

fluorecein staining high-velocity injury to the globe can lead to a corneal abrasion or open globe laceration (OGL). Small injuries may not be visible on routine inspection, but further assessment can be obtained with fluorescein instillation.

A 75-year-old female with a past history of cholelithiasis presents complaining of RUQ pain that radiates to her back, with nausea, vomiting, and abdominal distention. Abdominal plain films show air in the biliary tree and a "stepladder"appearance of the small bowel

gallstone ileus (ileocecal valve is most common site of obstruction)

17 yo girl w/ 2 mos of colicky left lower quadrant pain that worsen w/intercourse and exercise but resolves spontaneously. UPT and STI testing is negative and US reveals 8-cm left ovarian cyst with calcifications and hyperechoic nodules. What is the best next step in management?

laparoscopic ovarian cystectomy to reduce risk of ovarian torsion (mature cystic teratoma)

38 yo woman w/ increased nausea and substernal discomfort, which often occurs after eating and cause patient to self-induce vomiting. 2 wks ago had difficulty swallowing w/sensation of food stuck in chest. Had GERD during pregnancy 10yrs ago, but no other medical conditions. Chest exam reveals retrocardiac air-fluid level. What is the most likely underlying cause of these symptoms?

gastric herniation into the thoracic cavity (paraesophageal hiatal hernia w/retrocardiac air-fluid level (due to stomach bubble within the thoracic cavity)) **achalasia results from degeneration of esophageal ganglion cells. patients have dysphagia, but regurg of undigested food is typicial and retrocardiac air-fluid level would not occur

45 yo presents to ED w/colicky, RUQ pain, hyperbilirubinemia, vomiting; and melena for 24 hrs. Was diagnosed with nonalcoholic steatohepatitis by percutaneous liver biopsy 5 days ago

hemobila (bleeding into biliary tract that usually occurs as a complication of hepatic or bilopancreatic interventions (eg, liver biopsy, cholecystectomy, endoscopic cholangiopancreatography)) **liver abscesses can form after liver biopsy and presents with RUQ pain, leukocytosis, and possibly hyperbilirubinemia when they compress the biliary tree; however, fevers are typical, and UGIB wouldn't occur

54 yo woman w/fatigue, anorexia and weight loss. PMH of HTN and type 2 DM. BMI of 31. Abdomen w/shifting dullness, hepatomegaly and single palpable liver nodule. ALP 370, AST 134, ALT 124, alpha fetoprotein 12 (normal: 0-20)

hepatocellular carcinoma (only 50% of cases have elevated alpha fetoprotein) **focal nodular hyperplasia is a benign liver lesion that arises in young women

54 yo woman w/difficulty walking, weakness and pain in thighs. Has HTN treated with hydrochlorothiazide and metropolol. Her brother died of a neurological disease at 20. Neurological exam reveals hyporeflexia and decreased strength in all muscles. ECG shows flat and broad T waves with occasional PVCs. What is the likely cause of her complaints?

hypokalemia from thiazide diuretic use

26 yo woman w/ right knee pain who is training for marathon. Pain is sharp and most prominent on lateral aspect of knee that is worse when descending stairs . Knee ligament testing shows no instability and there is normal range of motion. There is an area of tenderness 2cm proximal to right lateral joint line

iliotibial band syndrome **patellofemoral pain syndrome is an anterior knee pain that is reproduced by squatting **pes anserinus pain syndrome causes medial (not lateral) knee pain

87 yo bed-bound female w advanced dementia has 3 days of fever and foul-smelling breath. PMH of hypertension, hypercholesterolemia, and hemorrhagic stroke 8 mos ago. Has been admitted twice for pneumonia over the last 6 mos. Chest imaging reveals infiltrate in the posterior segment of the right upper lobe. What is the most important predisposing factor for this condition in this patient?

impaired swallowing and cough reflex (aspiration pneumonia) **impaired mucociliary clearance occurs in smokers and patients with either cystic fibrosis or kartagener syndrome - increases risk of pulmonary infections, but not aspiration pneumonia specifically

65 yo man w/ right shoulder pain over past few weeks. Works as house painter and has pain when reaching for or lifting objects above his head. There is no tenderness over the AC joint. Strength, sensation, and arterial pulses are normal in upper extremities. When shoulder is internally rotated, the thumb is pointed toward the floor, and the arm is passively raised in front of the patient, he experiences reproduction of pain at 60 degrees of flexion and above. What is likely responsible for his condition?

inflammation of the rotator cuff tendons (normal ROM w/positive impingement test) - common to have rotator cuff tendinopathy with repetitive activity above shoulder height **AC arthritis would present with pain lifting shoulder over head but patients would typically have tenderness, swelling, or deformity in the AC space

patient w/ blunt abdominal trauma after night of heavy drinking now has inability to void, abdominal distention w/ascites, and elevated BUN and creatinine

intraperitoneal bladder rupture

4 yo boy brought to ED due to nausea, diarrhea, and abdominal pain after ingesting unknown pills. Temp 99, BP 76/38, pulse 160, resp 34/min. patient tired and not responsive to painful stimuli, peripheral pulses weak, and capillary refill is 4 seconds. Begins to develop hematemesis after exam. Has anion gap acidosis and x-ray shows several small opacities in the stomach and duodenum. What did he most likely ingest?

iron (leads to abdominal pain, vomiting diarrhea, and bleeding, w/anion gap metabolic acidosis) tx w/IV deferoxamine chelation therapy **salicylate (eg, aspirin) poisoning presents w/nausea, vomiting and metabolic acidosis w/respiratory compensation. However, tinnitus is common early sign, and aspirin tablets aren't visualized on x-ray

13 yo girl who complains of lethargy. Patients family is Greek and mother and paternal uncles have chronic anemia. Menarche was 1yr ago and pt has heavy bleeding w/ passage of clots. Pt is a vegetarian. Family house was built in the 1980s. Systolic ejection murmur is heard in the pulmonic area, Hgb 7.9 and MCV 65. What is the next step in evaluation?

iron studies (prolonged bleeding and poor dietary intake raises suspicion of iron deficiency anemia (IDA)) **correction of IDA is necessary before diagnosing thalassemia because iron deficiency can lower hemoglobin A2 and mask the diagnosis **lead poisoning in houses built before 1978

55 yo woman presents to ED w/ acute onset midepigastric pain that radiates to her back w/nausea and vomiting. ALP 148, AST 111, ALT 160, amylase 940, lipase 2155. US shows several gallstones w/out gallbladder thickening. She recovers quickly w/symptomatic care. What is next best step in management?

laparoscopic cholecystectomy (gallstone pancreatitis and at increased risk for it to occur again) **ERCP recommended for patines with gallstone pancreatitis who have, cholangitis, visible common bile duct dilation/obstruction, or increasing liver enzymes

80 yr old man w/pneumonia and a hx of hypertension, coronary artery disease, and ascending aortic aneurysm with moderate aortic insufficiency should not be treated with what antibiotic?

levofloxacin fluroquinolones upregulate cell matrix metalloproteases, including increased collagen degradation; this mechanism is likely responsible for several associated adverse effects, including achilles tendon rupture, retinal detachment, and aortic aneurysm rupture

62 yo male with 30 pack year smoking history, HTN, and dyslipidemia who drinks alcohol occasionally presents with sudden-onset abdominal pain, symptomatic hypotension, weakness, diaphoresis, and flank ecchymosis

likely a ruptured abdominal aortic aneurysm do a focused abdominal US

51 yo man w/frequent itching, especially after bathing, occasional headaches and dizziness, ruddy cyanotic appearance, and spleen palpable 4cm below left lower ribs, with platelets 521,000, Hgb 21.5, and WBC 14,800. What is likely to be seen on further workup?

low serum erythropoietin level (polycythemia vera) normally RBC production is dependent on EPO, a cytokine released by kidneys (and liver) in response to tissue hypoxia; EPO activates the JAK2 tyrosine kinase, which differentiated late myeloid cells from erythrocytes In PV, RBC production is driven by constitutively active JAK2, gene rather than tissue hypoxia, so EPO tends to be low **serum iron low in most patients due to increase hematopoiesis w/subsequent increased iron utilization

38 yo female w/progressive muscle weakness over last 6 mos w/diffic climbing stairs and combing hair and inability to walk short distances w/out rest. No muscle pain. Has hypothyroidism, demineralization of vertebral bones, BP 155/100, BMI 32, facial hirsuitism, and mild proximal weakness of the extremities. Muscles aren't tender to palpation and no fasciculations are noted. What is most likely responsible for patient's muscle weakness?

muscle atrophy (cushing syndrome) **hypokalemia, hypomagnesemia, and hypercalcemia cans cause muscle weakness, but none would be associated with hirsuitism

52 yo female w/gradual onset of weakness in proximal thigh muscles. She has increasing difficulty arising from her chair and climbing stairs w/out problems chewing food, changes in bowel or bladder function, morning stiffness, or joint pain. Meds are lisinopril and amlodipine for HTN. 4/5 strength in thighs w/ mild tenderness. Rest of exam normal. What would establish diagnosis in this patient?

muscle biopsy, which will show endomysial infiltrate and patchy necrosis (polyomyositis with slowly progressive weakness of lower extremities) will have elevated CK, aldolase, aST (muscle enzymes), ANA autoantibodies

7 day old girl in neonatal intensive care w/ truncus arteriosus has poor feeding and abdominal distension. Pt hospitalized since birth at 38 wks and is awaiting adequate weight gain prior to heart repair. Temp 95, BP 60/30, pulse 180, resp 65, pulse ox 82%. PE: lethargic, cyanotic, moderate abdominal distension and absent bowel sounds. BRB visible ins tool and nasogastric tube placed for decompression recovers bilious fluid. What is the most likely diagnosis?

necrotizing enterocolitis (temperature instability, abdominal distension, and bloody stools) although majority of affected infants are premature or have very low birth weight, term infants with reduced mesenteric oxygen delivery from cyanotic congenital heart disease and/or hypotension are also at risk for intestinal ischemia and infarction

most appropriate treatment for patient with 3 day hx of fever, chills, and sore throat with muffled voice and enlarged, tender cervical lymph nodes on the left, and uvula deviated to the right?

needle peritonsillar aspiration (peritonsillar abscess) **emergent laryngoscopy is for patients with epiglottitis

32 yo male with Crohn's disease comes to ED with left flank pain, hematuria, and vomiting

nephrolithiasis oxalate absorption is increased in Crohn disease and all other interstinal diseases causing fat malabsorption increased absorption is the most common cause of symptomatic hyperoxaluria and oxalate stone formation **dehydration and prolonged immobilization predispose to calcium stone formation

14 yr old girl who hasn't yet menstruated w/ corrective glasses for myopia. Height is in the 5th percentile and BP is 140/90 in right upper arm and 90/40 in lower extremities. Multiple pigmented nevi are scattered on the face and chest. What is she at increased risk for?

osteoporotic fracture due to estrogen deficiency from ovarian dysgenesis (turner syndrome) **most patients w/turner syndrome have normal cognitive abilities, but there is an increased risk of impaired nonverbal skis (eg, mathematics), ADHD, and problems w/executive functioning

patient who fell, heard a pop and is holding knee in flexed position with significant swelling and tenderness over the medial side with large, painful, immobile deformity over lateral side

patellar dislocation (laterally)

22 yo w/ new onset foot pain that is localized to the heel and is most pronounced when getting up from sitting after a long period of time. She enjoys running barefoot on the beach. Foot has moderately high arch and direct pressure to the bottom elicits a sharp pain, but mediolateral squeezing of the heel causes no discomfort

plantar fasciitis (high impact exercise to barefeet leading to repetitive microtrauma to the aponeurosis and its insertion point at the calcaneus) **stress fractures typically seen in inexperienced athletes after initiation of high impact exercise program and there would be a positive squeeze test

patient w/ severe COPD exacerbation (eg, wheezing, dyspnea, cough) requiring mechanical ventilation who develops abrupt-onset hypoxemia associated with elevated peak and plateau pressure (measured on the ventilator) and decreased right-sided breath sounds

pneumothorax positive pressure ventilation (eg, mechanical ventilation) poses risk of pulmonary barotrauma, leading to alveolar rupture and pneumothorax formation and patients w/COPD are at increased risk - increased plateau pressure is a measure of the lung'd decreased ability to expand (compliance) **large PE can cause acute hypoxemia but wouldn't cause elevated ventilatory pressures or decreased unilateral breath sounds

52 yo w/ intermittent RUQ pain that generally lasts 30-60mins (similar to pain episodes before surgery) and nausea. She had elective cholecystectomy for gallstones a year ago. US of abdomen reveals mild dilation of the CBD and pancreas appears normal (ALP 185, AST 84, ALT 72)

postcholecystectomy syndrome (PCS) can be due to biliary (eg, retained common bile duct or cystic stone, biliary dyskinesia) or extrabiliary (eg, pancreatitis, peptic ulcer disease, coronary artery disease) causes Findings suggest common bile duct stones or biliary sphincter of Oddi dysfunction Next step involves endoscopic US, endoscopic retrograde cholangiopancreatography (ERCP), or magnetic resonance cholangiopancreatography

64 yo patient brought into ED after high-speed motor vehicle collision with internally rotated right leg. Hip is flexed and adducted, right knee is flexed, and he is unable to dorsiflex or straighten the right leg

posterior hip dislocation

29 yo flight attendant with generalized weakness and lightheadness, nausea, diffuse abdominal pain and 22 lb weight loss over last 3 months who has hypothyroidism, and hyperpigmented mucous membranes, (afebrile, BP of 80/50, HR of 110, Hgb of 11, and 10% eosinophils)

primary adrenal insufficiency (autoimmune inflammation) **would likely have fever in parasitic infection and wouldn't have hyperpigmented mucous membranes

34 yo presents to ED w/ right lower leg swelling, redness, and pain. Mother has hx of PE. She has tenderness to palpation in the right calf and US shows DVT. Further eval shows elevated plasma homocysteine level. She is started on heparin and warfarin therapy. What additional therapy is indicated?

pyridoxine (vitamin B6) - lowers homocysteine levels by acting as a cofactor for the enzyme cystathionine B-synthase, which metabolizes homocysteine into cystathionine - homocysteine is highly reactive and increased levels predisposed to venous thrombosis and atherosclerosis

patient who recently had a UTI and now has several days of unilateral flank pain, weight loss, fever, and leukocytosis without UTI symptoms or bacteriuria

renal abscess **acute interstitial nephritis can cause AKI but is typically associated with rash, pyuria, and urine eosinophilia and WBC casts - weight loss and flank pain would be unexpected **acute papillary necrosis can cause AKI, fever, and flank pain; however, it typically causes hematuria and is most common in patients with analgesic overuse or sickle cell anemia

38 yo male complains of SOB that is worse with exertion and cough with mucoid sputum over the past 6mo. He smoked cigarettes for 5yrs but quit 13 years ago. Physical exam is unremarkable, as well as CBC and CMP. Xray shows bilateral basilar lucency. What is best next test?

serum alpha-1 antitrypsin level smoking affects upper lobes where AAT deficiency classically causes destruction of lower lobes

sepsis in a child with sickle cell anemia should make you think of what organism?

streptococcus pneumoniae (even with vaccination after asplenia, it remains the most common cause)

25 yo delivered a baby 2 weeks ago and is in clinic to discuss birth control. She takes a daily iron supplement for iron deficiency anemia due heavy menstrual bleeding. Vitals are normal. What is the best contraceptive option for her?

subdermal progestin-releasing implant **cant use copper IUD because may increase menstrual bleeding and worsen anemia **no estrogen <1mo postpartum because of increased risk of thromboembolism

65 yo w acute right flank pain that is crampy and waxes and wanes. Says he has passed "sand" in urine before. PMH of obesity, hyperlipidemia, HTN, and gout. BP 160/100, mild tenderness over right flank, urine pH of 5 w/hematuria. US shows right-sided hydronephrosis and dilation of proximal ureter/ What is the most likely cause of symptoms?

supersaturation of urine with uric acid (uric acid kidney stones) - stones radiolucent, so not visible on x-ray, but visible on CT **renal clearance of excess oxalate leads to calcium oxalate kidney stones. However, decreased oxalate absorption wouldn't increase risk for stones

65 yo man was evaluated for a dry cough (30 pack yr hx) and CT found 2-cm peripheral, round lesion in the right lower lung lobe. Borders are irregular and no calcification is present. What should you recommend next?

surgical excision nodules <0.6cm are unlikely to be malignant and generally do not require follow-up; however, nodules >0.8cm require additional management or surveillance

48 yo w/progressive loss of visual acuity. pt cant read computer screen and has stopped driving at night due to excessive glare of oncoming headlights. has well controlled HIV. examination reveals opacification of lenses bilaterally. what is the best next step in management?

surgical removal of the lens (cataracts) **lanoprost eye drops are used to treat open-angle glaucoma, which presents with chronic, insidious loss of peripheral vision

patient who sustained a whiplash cervical spine injury 7 years ago how complains of upper extremity weakness and loss of sensation in the hands (light touch, vibration, and position sense are intact)

syringomyelia

54 yo man comes to ED after 2 days of fever, chills, dysphagia, and drooling with inability to eat or drink due to pain in neck and mouth. Hx of heave alcohol use. Febrile, tachycardic, and tachypneic, and is drooling with muffled voice. Tongue is displaced posteriorly and superiorly due to swelling at floor of mouth. Bilateral tenderness, induration, and palpable crepitus of submandibular area. Most likely source of infection?

teeth roots - ludwig angina is a rapidly progressive cellulitis of the submandibular space and most cases arise from dental infections in the mandibular molars that spread contiguously down the root into the submylohyoid (and then sublingual) space - tx: iv antibiotics (ampicillin-sulbactam, clindamycin) and removal of inciting tooth **the parotid gland is located on the lateral face superficial to the masseter muscle, not affected in ludwig angina

7 yo girl slipped and fell onto outstretched right hand and was in immediate pain and would not move arm. She is holding the arm in flexion and winces when it is moved. X-ray shows supracondylar fracture w/posterior displacement of the distal humerus fragment. She is at greatest risk of injury to what?

the median nerve and brachial artery, which are located anterior to the humerus (anterior displaced proximal fragment can entrap them)

patient with weeks of upper abdominal pain, blood diarrhea, and fecal urgency likely has undiagnosed IBD (eg, ulcerative colitis). Acute worsening with fever, abdominal distension, leuukocytosis, hypotension, and tachycardia suggests what?

toxi megacolon initial test is abdominal x-ray to confirm the diagnosis and exclude perforation

58 yo pt presents to ED due to several hours of left-sided headache, nausea, and vomiting. Has blurred vision and sees halos around lights. Started a new med recently for hand tremors. BP is 150/90 and pulse is 96/min. PE reveals left eye circum-corneal infection w/dilated and sluggish pupil and decreased acuity. She has increased resistance to passive movements of the extremities. What medication is likely to have caused her symptoms?

trihexylphenidyl (likely has acute-closure glaucoma (headache, nausea, and sluggish/dilated pupil)) - anticholinergic used in tx of symptomatic parkinson disease - anticholinergics can cause mydriasis, which can precipitate acute angle closure glaucoma

37 yo in ED w/ abdominal pain, nausea, and vomiting. Underwent laparoscopic hysterectomy for severe endometriosis a week ago and for the past 2 days has had increased pain and bloating. Voiding normally and passing flatus, but has noticed increased vaginal discharge. Febrile w/moderately distended, nontender abdomen. Laparoscopic incisions intact w/out palpable masses or defects. Pelvic exam shows watery vaginal discharge and vaginal cuff appears closed. Hgb 12.2, Sodium 136, Potassium 3.6, normal creatinine. Urinalysis is normal. Abdominal US shows large amount of fluid with no internal echoes. What is the most likely diagnosis?

unilateral ureteral laceration (uroperitoneum in someone who has undergone hysterectomy, with distorted pelvic anatomy (eg, endometriosis, prior surgery), the most likely cause is from a ureteral laceration) - normal creatinine because other kidney and ureter functioning normally **hemoperitoneum unlikely because her BP, Hgb, and pulse were normal

45 yo in ED w/ abdominal pain, nausea, and 2 episodes of vomiting over the last 2 hrs. For past several weeks has had episodic epigastric and RUQ pain associated with nausea. PMH of MVP, obesity, and migraine headaches relieved by NSAIDs. Febrile w/pulse of 102/min, diffuse abdominal tenderness and guarding over the upper quadrant. Stool guaiac is positive. What is best next step in management?

upright x-ray of the chest and abdomen to look for subdiaphragmatic free air presentation concerning for peptic ulcer disease (eg, postprandial nausea/upper abdominal pain, NSAID use, positive stool guaiac) complicated by perforation **stool guaiac unlikely to be positive in cholecystitis **mesenteric ischemia: severe abdominal pain w/benign abdominal exam (no diffuse tenderness and guarding)

62 yo man hospitalized for elective CABG who has severe penicillin allergy causing anaphylactic shock. What is the most appropriate preoperative antibiotic for him?

vancomycin or clindamycin (good gram-positive coverage) cant use first and second generation cephalosporins (shared allergenic epitopes to penicillin) **ciprofloxacin is used prophylaxis in some genitourinary procedures

patient w/ mucosal candidiasis who now has dyspnea, fever, and hypoxemia w/bilateral crackles on lung examination and 2/6 midsystolic murmur at left upper sternal border. No JVD or lower extremity edema present. What is the most likely underlying mechanism responsible for the patient's hypoxemia?

ventilation/perfusion mismatch (likely pneumonia) in pneumonia, the alveoli become filled with inflammatory exudate, leading to marked impairment of alveolar ventilation in the affected portion of the lungs. the result is right-to-left intrapulmonary shunting, and extreme form of V/Q mismatch in which there is perfusion of lung tissue in the absence of alveolar ventilation (V=0) **patient doesn't have signs of HF consistent w/endocarditis and fever induces hyperdynamic state, which most likely accounts for his murmur (flow murmur)

patient who gets in a car accident, experiences neck hyperextension, and has a C5 right lamina fracture shown on CT will have what symptoms if unilateral joint subluzation occurs?

weak risk extension and thumb sensory loss (C6 radiculopathy)


संबंधित स्टडी सेट्स

Real Estate Private Equity Questions

View Set

Ch.15 The West Between the Wars, Ch.14 World War I and The Russian Revolution, Ch.13 Challenge and Transition in East Asia (1800-1914), Ch. 9 The French Revolution and Napoleon, Ch.12 The Reach of Imperialism (1800-1914), Ch.11 Mass Society and Democ...

View Set

Exam 6: Ch. 38 & 40 Musculoskeletal

View Set

Entrepreneurship Review Chapter 1

View Set

NC Statutes and Regulations Pertinent to Property and Liability

View Set

2ND TB (CHPT 5): Health Belief Model (HBM)

View Set

Systematic Review and Meta-analysis

View Set